27.11.2016 Views

Behavioral Science

Create successful ePaper yourself

Turn your PDF publications into a flip-book with our unique Google optimized e-Paper software.

DEJA REVIEW TM<br />

<strong>Behavioral</strong> <strong>Science</strong>


NOTICE<br />

Medicine is an ever-changing science. As new research and<br />

clinical experience broaden our knowledge, changes in treatment<br />

and drug therapy are required. The authors and the<br />

publisher of this work have checked with sources believed to<br />

be reliable in their efforts to provide information that is complete<br />

and generally in accord with the standards accepted at<br />

the time of publication. However, in view of the possibility of<br />

human error or changes in medical sciences, neither the<br />

authors nor the publisher nor any other party who has been<br />

involved in the preparation or publication of this work warrants<br />

that the information contained herein is in every respect<br />

accurate or complete, and they disclaim all responsibility for<br />

any errors or omissions or for the results obtained from use of<br />

the information contained in this work. Readers are encouraged<br />

to confirm the information contained herein with other<br />

sources. For example and in particular, readers are advised to<br />

check the product information sheet included in the package<br />

of each drug they plan to administer to be certain that the<br />

information contained in this work is accurate and that<br />

changes have not been made in the recommended dose or in<br />

the contraindications for administration. This recommendation<br />

is of particular importance in connection with new or<br />

infrequently used drugs.


DEJA REVIEW TM<br />

<strong>Behavioral</strong> <strong>Science</strong><br />

Second Edition<br />

Gene R. Quinn, MD, MS<br />

Resident Physician<br />

Department of Medicine<br />

University of California, San Francisco<br />

San Francisco, California<br />

University of Washington School of Medicine<br />

Seattle, Washington<br />

Class of 2009<br />

New York Chicago San Francisco Lisbon London Madrid Mexico City<br />

Milan New Delhi San Juan Seoul Singapore Sydney Toronto


Copyright © 2010, 2007 by The McGraw-Hill Companies, Inc. All rights reserved. Except as permitted<br />

under the United States Copyright Act of 1976, no part of this publication may be reproduced or distributed<br />

in any form or by any means, or stored in a database or retrieval system, without the prior written<br />

permission of the publisher.<br />

ISBN: 978-0-07-163293-5<br />

MHID: 0-07-163293-X<br />

The material in this eBook also appears in the print version of this title: ISBN: 978-0-07-162728-3,<br />

MHID: 0-07-162728-6.<br />

All trademarks are trademarks of their respective owners. Rather than put a trademark symbol after every<br />

occurrence of a trademarked name, we use names in an editorial fashion only, and to the benefit of the<br />

trademark owner, with no intention of infringement of the trademark. Where such designations appear in<br />

this book, they have been printed with initial caps.<br />

McGraw-Hill eBooks are available at special quantity discounts to use as premiums and sales<br />

promotions, or for use in corporate training programs. To contact a representative please e-mail us at<br />

bulksales@mcgraw-hill.com.<br />

TERMS OF USE<br />

This is a copyrighted work and The McGraw-Hill Companies, Inc. (“McGrawHill”) and its licensors<br />

reserve all rights in and to the work. Use of this work is subject to these terms. Except as permitted under<br />

the Copyright Act of 1976 and the right to store and retrieve one copy of the work, you may not decompile,<br />

disassemble, reverse engineer, reproduce, modify, create derivative works based upon, transmit, distribute,<br />

disseminate, sell, publish or sublicense the work or any part of it without McGraw-Hill’s prior consent.<br />

You may use the work for your own noncommercial and personal use; any other use of the work is strictly<br />

prohibited. Your right to use the work may be terminated if you fail to comply with these terms.<br />

THE WORK IS PROVIDED “AS IS.” McGRAW-HILL AND ITS LICENSORS MAKE NO GUAR-<br />

ANTEES OR WARRANTIES AS TO THE ACCURACY, ADEQUACY OR COMPLETENESS OF OR<br />

RESULTS TO BE OBTAINED FROM USING THE WORK, INCLUDING ANY INFORMATION THAT<br />

CAN BE ACCESSED THROUGH THE WORK VIA HYPERLINK OR OTHERWISE, AND EXPRESS-<br />

LY DISCLAIM ANY WARRANTY, EXPRESS OR IMPLIED, INCLUDING BUT NOT LIMITED TO<br />

IMPLIED WARRANTIES OF MERCHANTABILITY OR FITNESS FOR A PARTICULAR PURPOSE.<br />

McGraw-Hill and its licensors do not warrant or guarantee that the functions contained in the work will<br />

meet your requirements or that its operation will be uninterrupted or error free. Neither McGraw-Hill nor<br />

its licensors shall be liable to you or anyone else for any inaccuracy, error or omission, regardless of cause,<br />

in the work or for any damages resulting therefrom. McGraw-Hill has no responsibility for the content<br />

of any information accessed through the work. Under no circumstances shall McGraw-Hill and/or its<br />

licensors be liable for any indirect, incidental, special, punitive, consequential or similar damages that<br />

result from the use of or inability to use the work, even if any of them has been advised of the possibility<br />

of such damages. This limitation of liability shall apply to any claim or cause whatsoever whether such<br />

claim or cause arises in contract, tort or otherwise.


To my family


This page intentionally left blank


Contents<br />

Faculty Reviewers/Student Reviewers<br />

Preface<br />

xi<br />

xiii<br />

SECTION I HUMAN DEVELOPMENT AND PSYCHOLOGY 1<br />

Chapter 1 EARLY STAGES OF LIFE: INFANCY TO CHILDHOOD 3<br />

Theories of Development / 3<br />

Infant Morbidity and Mortality / 5<br />

Neonatal Reflexes / 5<br />

Developmental Milestones / 6<br />

Attachment / 11<br />

Early Adolescence (11 to 14 Years) / 12<br />

Clinical Vignettes / 13<br />

Chapter 2 MIDDLE STAGES OF LIFE: ADOLESCENCE TO ADULTHOOD 15<br />

Early Adulthood / 15<br />

Middle Adulthood / 17<br />

Clinical Vignettes / 17<br />

Chapter 3 LATE STAGES OF LIFE: AGING, DEATH, AND BEREAVEMENT 19<br />

Aging / 19<br />

Dying, Death, and Bereavement / 21<br />

Clinical Vignettes / 23<br />

Chapter 4 PSYCHOANALYTIC THEORY 25<br />

Clinical Vignettes / 33<br />

Chapter 5 LEARNING THEORY 35<br />

Associative Learning vs Nonassociative Learning / 35<br />

Clinical Vignettes / 42<br />

Chapter 6 SLEEP SCIENCE AND DISORDERS 43<br />

Normal Sleep / 43<br />

Abnormal Sleep / 45<br />

Other Sleep Changes / 47<br />

Clinical Vignettes / 48


viii<br />

Contents<br />

Chapter 7 SEXUALITY 49<br />

Sexual Development / 49<br />

Sexual Development and Physiologic Abnormalities / 50<br />

Hormones and Their Influence on Behavior / 51<br />

Sexual Response Cycle / 51<br />

Sexual Dysfunction / 52<br />

Paraphilias / 54<br />

Influence of Medical Conditions on Sexuality / 54<br />

Effects of Drugs and Neurotransmitters on Sexuality / 55<br />

Clinical Vignettes / 56<br />

Chapter 8 ABUSE AND AGGRESSION 57<br />

Children and Elder Abuse and Neglect / 57<br />

Domestic Partner Abuse / 60<br />

Sexual Violence / 61<br />

Aggression / 62<br />

Clinical Vignettes / 63<br />

Chapter 9 SUICIDE 65<br />

Suicide / 65<br />

Clinical Vignettes / 68<br />

Chapter 10 GENETIC BASIS OF BEHAVIOR 69<br />

Genetic Studies / 69<br />

Psychiatric Disorders Genetics / 69<br />

Neuropsychiatric Disorder Genetics / 71<br />

Alcoholism Genetics / 72<br />

Clinical Vignettes / 72<br />

Chapter 11 NEUROCHEMISTRY IN BEHAVIORAL SCIENCES 75<br />

Neuroanatomy / 75<br />

Brain Lesions / 76<br />

Neurotransmitters / 77<br />

Amines / 78<br />

Neuropeptides / 81<br />

Clinical Vignettes / 82<br />

SECTION II PSYCHIATRIC DISORDERS AND TREATMENT 83<br />

Chapter 12 PSYCHOTIC DISORDERS 85<br />

Introduction / 85<br />

Disorders / 85<br />

Clinical Vignettes / 92


Contents<br />

ix<br />

Chapter 13 MOOD DISORDERS 93<br />

Clinical Vignettes / 101<br />

Chapter 14 ANXIETY DISORDERS 103<br />

Panic Disorder / 104<br />

Obsessive-Compulsive Disorder / 105<br />

Social Phobia / 106<br />

Posttraumatic Stress Disorder / 107<br />

Specific Phobia / 108<br />

Adjustment Disorder with Anxiety / 109<br />

Clinical Vignettes / 110<br />

Chapter 15 COGNITIVE DISORDERS 111<br />

Delirium / 111<br />

Dementia / 112<br />

Amnestic Syndromes / 115<br />

Clinical Vignettes / 115<br />

Chapter 16 SOMATOFORM DISORDERS 117<br />

Somatization Disorder / 118<br />

Conversion Disorder / 119<br />

Hypochondriasis / 120<br />

Body Dysmorphic Disorder / 121<br />

Factitious Disorder / 122<br />

Malingering / 123<br />

Clinical Vignettes / 123<br />

Chapter 17 PERSONALITY DISORDERS 125<br />

Cluster A: The Mad / 126<br />

Cluster B: The Bad / 127<br />

Cluster C: The Sad / 130<br />

Clinical Vignettes / 133<br />

Chapter 18 DISSOCIATIVE DISORDERS 135<br />

Clinical Vignettes / 136<br />

Chapter 19 SUBSTANCE ABUSE DISORDERS 139<br />

Clinical Vignettes / 151<br />

Chapter 20 EATING DISORDERS 153<br />

Clinical Vignettes / 156


x<br />

Contents<br />

Chapter 21 CHILD PSYCHIATRY 159<br />

Pervasive Development Disorders / 159<br />

Disruptive Behavior Disorders / 161<br />

Attention-Deficit Hyperactivity Disorder / 162<br />

Other Neuropsychiatric Disorders of Childhood / 162<br />

Clinical Vignettes / 164<br />

Chapter 22 PSYCHOPHARMACOLOGY 165<br />

Clinical Vignettes / 174<br />

SECTION III ETHICS, HEALTH CARE, AND STATISTICS 175<br />

Chapter 23 CLINICAL PRACTICE AND DIFFICULT SITUATIONS 177<br />

Clinical Vignettes / 180<br />

Chapter 24 MEDICAL ETHICS AND LEGAL ISSUES 181<br />

Clinical Vignettes / 188<br />

Chapter 25 HEALTH CARE IN THE UNITED STATES 191<br />

Health-Care Insurance / 191<br />

Health-Care Costs / 193<br />

Health-Care Delivery Systems / 194<br />

Health Status and Determinants / 195<br />

Clinical Vignettes / 196<br />

Chapter 26 EPIDEMIOLOGY AND RESEARCH DESIGN 199<br />

Epidemiology / 199<br />

Research Study Designs / 201<br />

Testing / 203<br />

Measures of Association / 205<br />

Clinical Vignettes / 206<br />

Chapter 27 BIOSTATISTICS 209<br />

Statistical Distribution / 209<br />

Statistical Hypothesis and Error Types / 212<br />

Statistical Tests / 214<br />

Clinical Vignettes / 215<br />

Index 217


Faculty Reviewers<br />

Debra L Klamen, MD, MPHE<br />

Associate Dean, Education and Curriculum<br />

Professor and Chair, Department of Medical<br />

Education<br />

Southern Illinois University School of<br />

Medicine<br />

Springfield, Illinois<br />

Student Reviewers<br />

Jessica Bury<br />

Mayo Medical School<br />

MD/MPH Candidate<br />

Class of 2010<br />

Daniel Marcovici<br />

Sackler School of Medicine<br />

Tel Aviv University<br />

Class of 2011<br />

Sarah Fabiano<br />

SUNY Upstate Medical University<br />

Class of 2010


This page intentionally left blank


Preface<br />

<strong>Behavioral</strong> science is an extremely high-yield, yet often overlooked, subject on the<br />

USMLE Step 1. Understanding important behavioral science concepts and being able<br />

to apply them to test questions can significantly increase your score. Deja Review:<br />

<strong>Behavioral</strong> <strong>Science</strong>s is the perfect format to study behavioral science material; it helps<br />

you rapidly review material you know as well as fill-in gaps in your knowledge.<br />

In this second edition of Deja Review: <strong>Behavioral</strong> <strong>Science</strong>s I have highlighted high-yield<br />

concepts such as psychiatric disorders and developmental milestones while still keeping<br />

the content comprehensive enough to use during your preclinical course and clerkship.<br />

Epidemiology and biostatistics are often tested on the USMLE—this new edition has<br />

fully revised and expanded sections on these subjects that will review and teach you<br />

core concepts needed to work through epidemiology questions on the USMLE. With a<br />

little bit of work and Deja Review: <strong>Behavioral</strong> <strong>Science</strong>s, you’ll be well-prepared for the<br />

multitude of behavioral science questions on the USMLE.<br />

ORGANIZATION<br />

All concepts are presented in a question and answer “flashcard” format that covers key<br />

facts on commonly tested topics in behavioral medicine. <strong>Behavioral</strong> science is often<br />

conceptual, and questions are designed to review and teach these concepts instead of<br />

simply memorizing material.<br />

The question and answer format has several important advantages:<br />

• It provides a rapid, straightforward way for you to assess your strengths and<br />

weaknesses.<br />

• It serves as a quick, last-minute review of high-yield facts.<br />

• It allows you to efficiently review and commit to memory a large body of<br />

information.<br />

At the end of each chapter, you will find clinical vignettes that expose you to the type<br />

of questions classically tested on the USMLE Step 1. These board-style questions put<br />

the basic science into a clinical context, allowing you apply the facts you have just<br />

reviewed in a clinical scenario and make the diagnosis.<br />

HOW TO USE THIS BOOK<br />

This text was assembled with the intent to represent the core topics tested on course<br />

examinations and USMLE Step 1. Remember, this text is not intended to replace comprehensive<br />

textbooks, course packs, or lectures. However, it may serve as a supplement


xiv<br />

Preface<br />

to your studies during your coursework and Step 1 preparation. You may use the book<br />

to quiz yourself or classmates on topics covered in recent lectures and clinical case discussions.<br />

A bookmark is included so that you can easily cover up the answers as you<br />

work through each chapter. The compact, condensed design of the book is conducive<br />

to studying on the go, especially during any downtime throughout your busy day.<br />

However you choose to study, I hope you find this resource helpful throughout your<br />

preparation. Good luck and good studying!<br />

Gene R. Quinn


SECTION I<br />

Human Development<br />

and Psychology


This page intentionally left blank


CHAPTER 1<br />

Early Stages of Life:<br />

Infancy to Childhood<br />

THEORIES OF DEVELOPMENT<br />

Describe Erik Erikson’s theories of<br />

development.<br />

Describe Sigmund Freud’s theories of<br />

development.<br />

Describe Jean Piaget’s theories of<br />

development.<br />

Critical periods at which achievement<br />

of social goals need to be achieved,<br />

otherwise they won’t be achieved.<br />

Organized by parts of the body from<br />

which pleasure is derived at each age of<br />

development. Each stage is part of the<br />

development into the sexual maturity of<br />

adulthood, characterized by ego<br />

formation and the ability to delay<br />

gratification.<br />

Learning capabilities of the child at<br />

various ages during development;<br />

children must move through four stages<br />

of development. There is a specific set<br />

of skills that must be mastered at each<br />

stage of development before<br />

progression to the other stages.<br />

Erik Erikson’s Theory of Development<br />

Which stage of development is<br />

characterized by an infant establishing<br />

faith in their caregiver?<br />

Which stage of development is<br />

characterized by a child learning<br />

physical skills such as walking and<br />

learning to use the bathroom?<br />

Which stage of development is<br />

characterized by a child becoming<br />

assertive in their learning?<br />

Trust vs Mistrust: birth to 18 months<br />

Autonomy vs Shame and Doubt:<br />

age 18 months to 3 years<br />

Initiative vs Guilt: age 3 to 6 years<br />

3


4 Deja Review: <strong>Behavioral</strong> <strong>Science</strong><br />

Which stage of development is<br />

characterized by a child acquiring<br />

new skills at a rapid rate?<br />

Which stage of development is<br />

characterized by a teen who achieves a<br />

sense of identity in politics, sex roles,<br />

or occupation?<br />

Which stage of development is<br />

characterized by an adult determining<br />

whether or not they want to establish<br />

an intimate relationship with another<br />

individual?<br />

Which stage of development is<br />

characterized by an adult finding ways<br />

to support and encourage the next<br />

generation?<br />

Which stage of development is<br />

characterized by an adult reflecting<br />

on their experiences to derive<br />

meaning from their life?<br />

Industry vs Inferiority: age 6 to 12 years<br />

Identity vs Role Confusion: age 12 to<br />

18 years<br />

Intimacy vs Isolation: age 19 to 40 years<br />

Generativity vs Stagnation: age 40 to<br />

65 years<br />

Ego Integrity vs Despair: age 65 years<br />

to death<br />

Sigmund Freud’s Theory of Development<br />

Which stage of development is<br />

characterized by focus on receiving<br />

pleasure through food consumption or<br />

sucking on pacifiers?<br />

Which stage of development is<br />

characterized by focus on receiving<br />

pleasure through potty training?<br />

Which stage of development is<br />

characterized by focus on identifying<br />

with adult role models and the oedipal<br />

complex?<br />

Which stage of development is<br />

characterized by focus on expanding<br />

social interactions?<br />

Which stage of development is<br />

characterized by focus on establishing<br />

a family?<br />

Oral phase present from birth to<br />

age 1 year<br />

Anal phase present from age 1 to<br />

3 years<br />

Phallic phase present from age 3 to<br />

6 years<br />

Latency phase present from age 6 to<br />

12 years<br />

Genital phase present from age 13 to<br />

adulthood<br />

Jean Piaget’s Theory of Development<br />

Which stage of development is<br />

characterized by infants and toddlers<br />

focusing on their eyes, ears, hands,<br />

and other senses?<br />

Sensorimotor period from birth to<br />

age 2 years


Early Stages of Life: Infancy to Childhood 5<br />

Which stage of development is<br />

characterized by children acquiring<br />

representational skills in the area of<br />

mental imagery and language?<br />

Which stage of development is<br />

characterized by children being more<br />

logical, flexible, and organized than in<br />

early childhood?<br />

Which stage of development is<br />

characterized by being able to think<br />

logically, theoretically, and abstractly?<br />

Preoperational thought present from<br />

age 2 to 7 years<br />

Concrete operational present from<br />

age 7 to 12 years<br />

Formal operational from age 12 to<br />

adulthood<br />

INFANT MORBIDITY AND MORTALITY<br />

Define premature birth.<br />

What are the potential outcomes of<br />

being a premature infant?<br />

In the United States, what percentage<br />

of births is premature?<br />

What are the common risk factors<br />

associated with premature births?<br />

Less than 34 weeks gestation or birth<br />

weight less than 2500 g<br />

Increased infant mortality<br />

Delayed physical and social development<br />

Emotional and behavioral problems<br />

Dyslexia<br />

Child abuse<br />

6% for white women and 13% for<br />

African American women (an average<br />

of 7.2 per 1000 live births)<br />

Low socioeconomic status<br />

Teenage pregnancy<br />

Poor maternal nutrition<br />

NEONATAL REFLEXES<br />

What are the six important neonatal<br />

reflexes?<br />

Which neonatal reflexes are present<br />

at birth?<br />

1. Moro<br />

2. Palmar grasp<br />

3. Rooting<br />

4. Stepping<br />

5. Asymmetric tonic neck<br />

6. Parachute<br />

Moro<br />

Palmar grasp<br />

Rooting<br />

Stepping


6 Deja Review: <strong>Behavioral</strong> <strong>Science</strong><br />

What is the Moro/”startle” reflex?<br />

What is the palmar grasp?<br />

What is the rooting reflex?<br />

What is the stepping reflex?<br />

At what age do the Moro,<br />

palmar, rooting, and stepping<br />

reflexes disappear?<br />

What is the asymmetric tonic neck?<br />

At what age does asymmetric tonic<br />

neck appear and then disappear?<br />

What is the parachute reflex?<br />

Head extension causes extremity<br />

extension followed by flexion.<br />

If a finger is placed in an infant’s palm,<br />

it is grasped.<br />

If an object is placed around an infant’s<br />

mouth, the infant will pursue it.<br />

When held upright and leaning<br />

forward, an infant will make walking<br />

motions with their legs.<br />

4 to 6 months<br />

While supine, turning of the head<br />

causes ipsilateral extremity extension<br />

and contralateral flexion.<br />

Present at 2 weeks and disappears<br />

by 6 months<br />

While sitting and tilted to one side, an<br />

infant extends the ipsilateral arm to<br />

support the body.<br />

DEVELOPMENTAL MILESTONES<br />

What are the key categories of<br />

development?<br />

What are the developmental milestones<br />

at 1 month of age?<br />

What are the developmental milestones<br />

at 2 to 3 months of age?<br />

Gross motor<br />

Fine/visual motor<br />

Language<br />

Social<br />

Gross motor: when prone lifts head<br />

slightly<br />

Fine/visual: with eyes tracks objects to<br />

midline; tight grasp<br />

Language: startles to sound<br />

Social: fixes on face<br />

Gross motor: steadily holds head up;<br />

when prone lifts chest up<br />

Fine/visual: hands open at rest<br />

Language: smiles responsively; coos<br />

Social: recognizes parents; reaches for<br />

familiar objects or people


Early Stages of Life: Infancy to Childhood 7<br />

What are the developmental milestones<br />

at 4 to 5 months of age?<br />

What are the developmental milestones<br />

at 6 months of age?<br />

What are the developmental milestones<br />

at 9 months of age?<br />

What are the developmental milestones<br />

at 12 months of age?<br />

What are the developmental milestones<br />

at 15 months of age?<br />

What are the developmental milestones<br />

at 18 months of age?<br />

Gross motor: rolls front to back and back<br />

to front; sits well supported<br />

Fine/visual: grasps with both hands<br />

Language: orients to voice<br />

Social: laughs; enjoys observing<br />

environment<br />

Gross motor: sits well unsupported; sits<br />

upright<br />

Fine/visual: transfers hand to hand;<br />

reaches with either hand<br />

Language: babbles<br />

Social: recognizes strangers and has<br />

stranger anxiety<br />

Gross motor: crawls, pulls to stand<br />

Fine/visual: uses pincer grasp; finger<br />

feeds<br />

Language: says “dada/mama”;<br />

understands “no”<br />

Social: waves bye-bye; plays<br />

pat-a-cake<br />

Gross motor: walks alone<br />

Fine/visual: throws, releases objects<br />

Language: one to eight words other<br />

than “dada/mama”; one-step<br />

commands<br />

Social: imitates actions; comes when<br />

called; cooperates with dressing<br />

Gross motor: walks backward; creeps<br />

upstairs<br />

Fine/visual: builds two-block towers;<br />

scribbles; uses a cup<br />

Language: uses four to eight words<br />

Social: throws temper tantrums<br />

Gross motor: runs; kicks a ball<br />

Fine/visual: feeds self with utensils<br />

Language: points to body parts<br />

when asked; names common<br />

objects<br />

Social: plays around but not with other<br />

children; start of toilet training


8 Deja Review: <strong>Behavioral</strong> <strong>Science</strong><br />

What are the developmental milestones<br />

at 21 months of age?<br />

What are the developmental milestones<br />

at 24 months of age?<br />

What are the developmental milestones<br />

at 30 months of age?<br />

What are the developmental milestones<br />

at 3 years of age?<br />

What are the developmental milestones<br />

at 4 years of age?<br />

What are the developmental milestones<br />

at 5 years of age?<br />

Gross motor: squats and recovers<br />

Fine/visual: builds five-block towers<br />

Language: two-word combinations<br />

Social: toilet training<br />

Gross motor: walks well up and down<br />

stairs; jumps<br />

Fine/visual: removes clothing; copies a<br />

line<br />

Language: 50-word vocabulary;<br />

stranger understands half of<br />

speech<br />

Social: follows two-step commands;<br />

engages in parallel play<br />

Gross motor: throws ball over hand<br />

Fine/visual: removes clothes; copies<br />

lines<br />

Language: appropriate pronoun use<br />

Social: knows first and last names<br />

Gross motor: pedals tricycle; goes<br />

up and down stairs with alternating<br />

feet<br />

Fine/visual: draws a circle; eats with<br />

utensils<br />

Language: three-word sentences;<br />

uses plurals and past tense;<br />

stranger understands three-fourths<br />

of speech<br />

Social: group play; shares toys<br />

Gross motor: hops and skips<br />

Fine/visual: catches ball; dresses alone;<br />

copies a cross<br />

Language: knows colors; counts to 10<br />

Social: imaginative play<br />

Gross motor: hops and skips<br />

Fine/visual: ties shoes<br />

Language: prints first name<br />

Social: plays cooperative games;<br />

understands rules and abides by them


Early Stages of Life: Infancy to Childhood 9<br />

Table 1.1 Developmental Milestones<br />

1 month old<br />

Gross Fine/Visual<br />

Motor Motor Language Social<br />

When With eyes Startles to Fixes on face<br />

prone lifts tracks sound<br />

head<br />

objects to<br />

slightly midline;<br />

tight grasp<br />

2 to 3 months old<br />

Steadily holds Hands open Smiles Recognizes<br />

head up; at rest responsively; parents;<br />

when prone coos reaches for<br />

lifts chest<br />

up<br />

familiar<br />

objects or<br />

people<br />

4 to 5 months old<br />

Rolls front to Grasps Orients to Laughs;<br />

back and with both voice enjoys<br />

back to front; hands observing<br />

sits well<br />

environment<br />

supported<br />

6 months old<br />

9 months old<br />

Sits well Transfers Babbles Recognizes<br />

unsupported; hand to strangers<br />

sits upright hand; and has<br />

reaches with<br />

stranger<br />

either hand<br />

anxiety<br />

Crawls, Uses pincer Says Waves<br />

pulls to grasp; “dada/mama”; bye-bye;<br />

stand finger understands plays<br />

feeds “no” pat-a-cake<br />

12 months old<br />

Walks alone Throws, One to eight Imitates<br />

releases words other actions;<br />

objects than “dada/ comes when<br />

mama”; called;<br />

one-step<br />

commands<br />

cooperates<br />

with<br />

dressing<br />

(Continued)


10 Deja Review: <strong>Behavioral</strong> <strong>Science</strong><br />

Table 1.1 Developmental Milestones (Continued )<br />

Gross Fine/Visual<br />

Motor Motor Language Social<br />

15 months old<br />

Walks Builds two- Uses four to Throws<br />

backward; block eight words temper<br />

creeps towers; tantrums<br />

upstairs scribbles;<br />

uses a cup<br />

18 months old<br />

Runs; kicks a Feeds self Points to body Plays around<br />

ball with parts when but not with<br />

utensils asked; names other<br />

common<br />

objects<br />

children;<br />

start of<br />

toilet<br />

training<br />

21 months old<br />

Squats Builds Two-word Toilet<br />

and five-block combinations training<br />

recovers towers<br />

24 months old<br />

Walks well Removes 50-word Follows twoup<br />

and clothing; vocabulary; step<br />

down stairs; copies a stranger commands;<br />

jumps line understands engages in<br />

half of speech parallel<br />

play<br />

30 months old<br />

Throws ball Removes Appropriate Knows first<br />

over hand clothes; pronoun use and last<br />

copies<br />

names<br />

lines<br />

3 years old<br />

Pedals Draws a Three-word Group play;<br />

tricycle; circle; eats sentences; shares toys<br />

goes up with uses plurals<br />

and down utensils and past<br />

stairs with<br />

tense;<br />

alternating<br />

feet<br />

stranger<br />

understands<br />

three-fourths<br />

of speech


Early Stages of Life: Infancy to Childhood 11<br />

Table 1.1 Developmental Milestones (Continued )<br />

Gross Fine/Visual<br />

Motor Motor Language Social<br />

4 years old<br />

5 years old<br />

Hops and Catches Knows colors; Imaginative<br />

skips ball; counts to 10 play<br />

dresses<br />

alone;<br />

copies a<br />

cross<br />

Hops and Ties shoes Prints first Plays<br />

skips name cooperative<br />

games;<br />

understands<br />

rules and<br />

abides by<br />

them<br />

ATTACHMENT<br />

What is anaclitic depression?<br />

What occurs without proper<br />

mothering or attachment?<br />

What occurs during failure<br />

to thrive?<br />

Toddlers who are hospitalized<br />

are most likely to fear what?<br />

Physical, psychological, and social<br />

problems caused by prolonged removal<br />

of parental nurturing during ages 6 to<br />

12 months. This is reversed if nurturing<br />

is reintroduced.<br />

Failure to thrive<br />

Developmental retardation<br />

Poor health and growth<br />

High death rates, even with adequate<br />

physical care<br />

Separation from parents or care<br />

providers more than bodily harm or<br />

pain


12 Deja Review: <strong>Behavioral</strong> <strong>Science</strong><br />

In what age group is elective<br />

surgery best tolerated?<br />

By age 3, how does separation from<br />

parents or care providers affect<br />

children?<br />

How do toddlers understand death?<br />

At what age do children begin to<br />

understand the concept of death?<br />

In what age group do children<br />

begin to form relationships with<br />

other adults?<br />

At what point should adopted<br />

children be told that they<br />

are adopted?<br />

7 to 11 years<br />

Separation from parents has no<br />

long-term negative effects on children.<br />

In fact, children at age 3 are able to<br />

spend significant portions of the day<br />

with other adults.<br />

It is an incomplete understanding of the<br />

meaning of death and the child may<br />

expect a friend, relative, or pet to come<br />

back to life.<br />

At the age of 8 years<br />

7 to 11 years<br />

At the earliest age possible, when they<br />

are able to understand language<br />

EARLY ADOLESCENCE (11 TO 14 YEARS)<br />

How is the start of puberty marked?<br />

How is the development through<br />

puberty measured?<br />

How many Tanner stages are there?<br />

What are the three categories of<br />

measurement?<br />

In girls: the onset of menstruation,<br />

beginning at age 11 to 12 years<br />

In boys: the first ejaculation, occurring at<br />

age 13 to 14 years<br />

By the Tanner stages<br />

There are five Tanner stages.<br />

1. Male genitalia<br />

2. Female breasts<br />

3. Pubic hair


Early Stages of Life: Infancy to Childhood 13<br />

Table 1.2 Tanner Stages of Development<br />

Stage I<br />

Stage II<br />

Male Genitalia Female Breasts Pubic Hair<br />

Childhood-sized Preadolescent breasts Absent.<br />

penis, testes, and with elevation of the<br />

scrotum<br />

papilla only<br />

Enlargement of the Breast buds with Pubic hair is<br />

testes and scrotum elevation of breast sparse and<br />

and papilla<br />

straight with<br />

downy hair on<br />

labia/penis base.<br />

Stage III<br />

Penis enlargement Breast and areola Pubic hair is<br />

enlargement with curled, darker,<br />

ingle contour<br />

and coarse.<br />

Stage IV<br />

Stage V<br />

Scrotal skin Areola and papilla Pubic hair is<br />

darkening and projection with adult- type hair<br />

rugations are separate contour limited to the<br />

present. (secondary mound). genital area.<br />

Adult-sized and Mature breasts Pubic hair is<br />

-shaped penis,<br />

adult-quantity<br />

testes, and scrotum<br />

and -pattern and<br />

spreads to the<br />

thighs.<br />

CLINICAL VIGNETTES<br />

A worried mother brings her 6-month-old son to your office for a checkup. She<br />

states that his highest achieved milestones are: he can recognize strangers, wave<br />

bye-bye, use a pincer-grasp, smile responsively, coo, crawl, and pull to stand. In<br />

which developmental areas is he behind?<br />

He is delayed in his language skills. By 6 months he should be babbling, and he<br />

should have oriented to voice before that. His language skills are that of a 2-3-<br />

month-old. His gross motor, fine motor, and social skills are all at a 9-month level.


14 Deja Review: <strong>Behavioral</strong> <strong>Science</strong><br />

Your medical school classmate is a braggart; he loves to tell everyone how<br />

advanced his 12-month-old daughter is. He claims she can walk backward, uses a<br />

cup, builds a two-block tower, uses six different words, and throws the cutest<br />

temper tantrums you’ve ever seen. How far advanced is she in developmental<br />

milestones?<br />

She is at a 15-month level in all areas—fine motor, gross motor, language, and<br />

social.<br />

A 35-year-old-male patient comes to your office worried that he never finished<br />

going through puberty. His pubic hair is sparse, but it is dark and curled. His penis<br />

has enlarged, but there is no scrotal darkening or rogations. At what Tanner stage<br />

is he?<br />

Tanner stage III


CHAPTER 2<br />

Middle Stages of Life:<br />

Adolescence to<br />

Adulthood<br />

EARLY ADULTHOOD<br />

What age range constitutes early<br />

adulthood?<br />

What are the primary characteristics<br />

of this stage of life?<br />

Which Erikson stage is prevalent<br />

during early adulthood?<br />

Which life events often occur during<br />

this stage?<br />

20 to 40 years<br />

Role in society is defined.<br />

Physical development peaks.<br />

Sense of independence.<br />

Intimacy vs Isolation<br />

Marriage<br />

Having children<br />

Occupation<br />

What percentage of Americans is 60% to 70%<br />

married by age 30?<br />

What are postpartum blues?<br />

What is the percentage of women who<br />

suffer from postpartum blues?<br />

How long do the symptoms of<br />

postpartum blue last?<br />

Mild mood swings that may switch<br />

quite rapidly and occur after delivery—<br />

usually within a few days.<br />

40% to 80% (very common and<br />

considered normal)<br />

They usually resolve within 2 weeks.<br />

15


16 Deja Review: <strong>Behavioral</strong> <strong>Science</strong><br />

What can a physician do to assist<br />

a patient with postpartum blues?<br />

What is the percentage of women who<br />

suffer from postpartum depression?<br />

What are the diagnostic criteria for<br />

postpartum major depression?<br />

What complications for the mother/<br />

child relationship may result from<br />

postpartum depression?<br />

When do the symptoms of postpartum<br />

major depression occur?<br />

What can a physician do to assist a<br />

patient with postpartum major<br />

depression?<br />

Support and reassurance<br />

Advise to get help with child care<br />

Watch for development of postpartum<br />

depression<br />

5% to 25% (making it important to<br />

screen for!)<br />

The same as a major depressive episode,<br />

but occurring after delivery.<br />

Note: The Diagnostic and Statistical<br />

Manual of Mental Disorders, fourth edition<br />

(DSM-IV-TR) does not have a separate<br />

diagnosis for postpartum depression—<br />

only a “postpartum onset” modifier of<br />

major depression.<br />

Failure to form bond with the baby<br />

Thoughts of failure as a mother<br />

Feeling unable to take care of baby<br />

Interference with child development<br />

Suicide of mother or homicide of infant<br />

Within 4 weeks after delivery (DSM-IV-TR<br />

criteria for “postpartum onset”), though<br />

some say up to 12 months after<br />

Psychosocial therapy<br />

Antidepressant medication<br />

May require hospitalization if severe<br />

What percentage of women suffer from 0.1% to 0.2%<br />

postpartum psychosis?<br />

What are characteristics of postpartum<br />

psychosis?<br />

When do the symptoms of postpartum<br />

psychosis occur?<br />

What can a physician do to assist a<br />

patient with postpartum psychosis?<br />

Which postpartum emotional reaction<br />

is considered normal?<br />

Delusions<br />

Hallucinations<br />

Mother may harm infant<br />

2 to 3 weeks after delivery<br />

Antipsychotic medication<br />

Hospitalization<br />

Note: This is a psychiatric emergency!<br />

Postpartum blues


Middle Stages of Life: Adolescence to Adulthood 17<br />

MIDDLE ADULTHOOD<br />

What age range constitutes middle<br />

adulthood?<br />

What are the primary characteristics of<br />

this stage of life?<br />

Which Erikson stage is prevalent during<br />

middle adulthood?<br />

What term describes the decrease in<br />

physiologic function that occurs<br />

in midlife?<br />

What physiologic functions decrease<br />

in men?<br />

What major reproductive change occurs<br />

in women?<br />

What are the characteristics of<br />

menopause?<br />

What medical intervention has been<br />

used to treat acute menopausal<br />

symptoms?<br />

How long should contraceptive<br />

measures continue after the<br />

last menstrual period in<br />

menopause?<br />

40 to 65 years<br />

Power<br />

Authority<br />

Generativity vs Stagnation<br />

Climacterium<br />

Endurance<br />

Muscle strength<br />

Sexual performance<br />

Menopause<br />

Lack of menstrual cycles<br />

Hot flashes<br />

Sexual Dysfunction/Vaginal Dryness<br />

Osteoporosis (long-term)<br />

Short-term estrogen replacement<br />

therapy (long-term not recommended)<br />

1 year


18 Deja Review: <strong>Behavioral</strong> <strong>Science</strong><br />

CLINICAL VIGNETTES<br />

A woman who delivered a healthy baby boy 4 days ago comes into your clinic. She<br />

states that she has been having mood swings—sometimes laughing, sometimes<br />

crying. She feels it is impacting her care of her baby.<br />

What is the most likely diagnosis?<br />

Postpartum blues<br />

What should your treatment be?<br />

You should screen her for postpartum depression and stay vigilant for this<br />

diagnosis—but reassure her that her symptoms are normal and will likely resolve<br />

within a couple of weeks. Also, recommend that she have someone help her with<br />

child care. Be sure to distinguish this from postpartum depression—a common<br />

disorder with more severe symptoms that should be treated more aggressively.<br />

A 58-year-old woman complains of less interest in sex over the last few years. She<br />

says it is a combination of less desire and pain with intercourse. What is the most<br />

likely cause of the patient’s symptoms?<br />

Postmenopausal women have a significant decrease in estrogen, leading to vaginal<br />

atrophy, dryness, and sexual dysfunction. Topical vaginal estrogen preparations<br />

may help, as can lubricants.<br />

A 32-year-old married woman develops hallucination and delusions 2 weeks after<br />

the delivery of a healthy baby boy. She informs the physician that she has had<br />

thoughts about harming the infant. The physician decides to hospitalize the<br />

mother immediately and place her on antipsychotic medication. What postpartum<br />

reaction has the woman experienced?<br />

Postpartum psychosis


CHAPTER 3<br />

Late Stages of Life:<br />

Aging, Death, and<br />

Bereavement<br />

AGING<br />

What percentage of the US population 15%<br />

will be 65 years and older by the<br />

year 2020?<br />

Which is the fastest growing age group<br />

in the United States today?<br />

What is the average life expectancy in<br />

the United States?<br />

What factors within a population can<br />

have an effect on differences in life<br />

expectancy?<br />

How are the life expectancies changing<br />

in regard to different ethnic groups?<br />

What physical changes are associated<br />

with aging?<br />

85 years and older<br />

77 years<br />

Gender (females tend to live longer than<br />

males; an average difference of 7 years)<br />

Race (whites tend to live longer than<br />

blacks)<br />

White female (80 years) > black female<br />

(74 years) = white male (74 years) ><br />

black male (66 years)<br />

Males and African Americans are living<br />

longer, thus the gap between them and<br />

the longest living (white female) is<br />

decreasing.<br />

Sensory: impaired vision and hearing<br />

Visceral: decreased pulmonary, renal,<br />

and gastrointestinal function<br />

Extremities: increased fat deposits,<br />

osteoporosis, and decreased muscle<br />

mass and strength<br />

19


20 Deja Review: <strong>Behavioral</strong> <strong>Science</strong><br />

What changes in the brain usually<br />

accompany aging?<br />

Less cerebral blood flow<br />

Decreased brain weight<br />

Enlarged ventricles and sulci<br />

Increased amount of senile plaques and<br />

neurofibrillary tangles (also found in<br />

older brains from patients who did not<br />

suffer from Alzheimer disease [AD])<br />

Psychological<br />

Does an individual’s level of<br />

intelligence change throughout life?<br />

Which Erikson stage of development<br />

is characteristic of the last stage of life?<br />

No. Slight decreases in cognitive ability<br />

are usually compensated for by life<br />

experience and wisdom—so called<br />

“crystallized intelligence.”<br />

Ego Integrity vs Despair. During this<br />

time there is an evaluation of the choices<br />

made in one’s life and a reconciliation<br />

of those choices with one’s values. A<br />

positive evaluation leads to contentment,<br />

whereas a negative evaluation leads to<br />

discontent.<br />

Psychopathology<br />

What is the most common psychiatric<br />

illness of the elderly?<br />

What factors can lead to depression in<br />

the elderly?<br />

What common disease process involving<br />

cognitive decline may mimic depression<br />

in the elderly?<br />

What three methods can be used to<br />

successfully treat depression?<br />

What changes in sleep patterns occur<br />

in the elderly?<br />

Major depression<br />

Loss of spouse, friends, and/or family<br />

Loss of prestige and income<br />

Decline of health<br />

Retirement<br />

Dementia<br />

1. Pharmacotherapy<br />

2. Psychotherapy<br />

3. Electroconvulsive therapy (this is<br />

still the most effective treatment for<br />

depression)<br />

Decreased quality of sleep with less<br />

time in deep sleep and more time in<br />

light sleep<br />

Decreased rapid eye movement (REM)<br />

sleep


Late Stages of Life: Aging, Death, and Bereavement 21<br />

What substance-use disorders are<br />

common in the elderly population but<br />

often unidentified?<br />

Do psychiatric drugs produce the same<br />

effects in the elderly as they do in<br />

young adults?<br />

Alcohol-related disorders<br />

No. There are differences in drug<br />

bioavailability, metabolism, and<br />

response, and the elderly have a greater<br />

susceptibility to side effects. Therefore,<br />

when you prescribe, you should start<br />

low and go slow.<br />

Longevity<br />

What factors are associated with<br />

longevity?<br />

Family history of longevity<br />

Continuation of occupational and<br />

physical activity<br />

Higher education<br />

Social support system including marriage<br />

(only men benefit from marriage)<br />

DYING, DEATH, AND BEREAVEMENT<br />

Which author describes five stages of<br />

death and dying, which may or may<br />

not occur in consecutive order?<br />

At what stage would a patient refuse<br />

to believe he or she is dying and say,<br />

“You must be reading the<br />

wrong chart”?<br />

At what stage would a patient become<br />

upset with the hospital staff and say,<br />

“You should have made the diagnosis<br />

sooner! It is your fault I am in this<br />

situation”?<br />

At what stage would a patient plead<br />

with a higher power for forgiveness<br />

and healing?<br />

At which stage would a patient<br />

become tearful, withdrawn, and<br />

apathetic?<br />

At what stage would a patient gain the<br />

understanding that life has come to<br />

an end?<br />

Elizabeth Kubler Ross<br />

Denial<br />

Anger<br />

Bargaining<br />

Depression<br />

Acceptance


22 Deja Review: <strong>Behavioral</strong> <strong>Science</strong><br />

What is the difference between<br />

bereavement and major depression?<br />

How long does bereavement last?<br />

Bereavement tends to be less severe<br />

in symptoms and less pervasive<br />

(ie, centers only around the loss).<br />

It is also self-limited and has a shorter<br />

duration.<br />

However, major depressive episodes<br />

may be superimposed onto, or<br />

precipitated by a major loss with<br />

bereavement.<br />

Normal bereavement should begin to<br />

resolve within a few months, though<br />

some grief may never go away.<br />

A DSM-IV-TR diagnosis of major<br />

depression should not be made within<br />

the first 2 months after a loss.<br />

Table 3.1 Differences Between Bereavement (Normal Grief) and Major Depression<br />

(Abnormal Grief)<br />

Bereavement<br />

Depression<br />

Minor sleep disturbances<br />

Some feelings of guilt<br />

Illusions<br />

Expressions of sadness<br />

Minor weight loss (8 lb)<br />

Poor grooming<br />

Few attempts to return to normal<br />

routine<br />

Severe symptoms subside in<br />

Severe symptoms continue for<br />

2 months<br />

Moderate symptoms subside in<br />

Moderate symptoms subside in<br />

1 month<br />

Tx: support groups, increased<br />

contact with physician, counseling,<br />

short-acting sedatives if needed<br />

Abbreviation: Tx, treatment.<br />

Tx: may include antidepressants,<br />

psychotherapy, antipsychotics,<br />

and electroconvulsive therapy


Late Stages of Life: Aging, Death, and Bereavement 23<br />

CLINICAL VIGNETTES<br />

You diagnose a 53-year-old man who has a long history of heavy smoking with<br />

metastatic lung cancer. What stage of grief would he likely be in if he:<br />

Says “I’ll be fine,” and lights up another cigarette.<br />

Denial<br />

Goes to church and prays to live another 3 years, just long enough to see his<br />

daughter graduate college.<br />

Bargaining<br />

Throws your reflex hammer at a nurse.<br />

Anger<br />

Says “I don’t care—there’s no point in living anyway.”<br />

Depression<br />

Starts to make plans for his hospice care and his surviving family.<br />

Acceptance<br />

An 83-year-old man lost his wife to cancer 1 month ago. He feels sad all the time,<br />

always thinking about his wife. He cries at night and cannot sleep. He has not<br />

been doing any of his normal activities. He also believes he hears his wife<br />

whispering in his ear at night before bedtime. What is your diagnosis?<br />

Bereavement—symptoms have only been present for 1 month after a major loss.<br />

If symptoms were to become more severe and last over 2 months, he would need<br />

to be screened for a mood disorder using DSM-IV-TR criteria.<br />

A 76-year-old man finds that he has left little impact on the world. He did not<br />

spend much time with his children, he is divorced, and he is not happy with any<br />

of the big decisions in his life. Which Erickson stage of life is this man most likely<br />

experiencing?<br />

Ego Integrity vs Despair


This page intentionally left blank


CHAPTER 4<br />

Psychoanalytic<br />

Theory<br />

What are the two major theories of the<br />

mind developed by Freud?<br />

Which part of Freud’s topographic mind<br />

contains information that one is<br />

unaware of, but can be accessed<br />

with prompting?<br />

Which part of the mind contains<br />

thoughts of which a person is aware?<br />

How is the expression of the id<br />

regulated?<br />

Which part of the mind helps a person<br />

maintain relationships?<br />

1. Topographic theory of the mind<br />

(includes three parts):<br />

a. Conscious<br />

b. Preconscious<br />

c. Unconscious<br />

2. Structural theory of the mind<br />

(also known as the tripartite<br />

theory):<br />

a. Ego<br />

b. Superego<br />

c. Id<br />

Preconscious<br />

The conscious—this part of the mind has<br />

no access to the unconscious. The<br />

unconscious includes what we are not<br />

aware of such as sexual drives,<br />

aggressive drives, impulses, or<br />

fantasies.<br />

The ego exerts the learned rules of the<br />

world over the id to control its overt<br />

expression of primitive drives, such as<br />

sexual urges and aggression. The ego<br />

manages and negotiates the drives from<br />

the id as well as moral restrictions (from<br />

the superego).<br />

The ego<br />

25


26 Deja Review: <strong>Behavioral</strong> <strong>Science</strong><br />

Which part of the mind incorporates<br />

acquired moral and ethical concepts?<br />

How does the ego deal with conflicts<br />

between the id and the superego?<br />

What type of thinking is primary<br />

process?<br />

Which part of the mind is controlled<br />

by primary process thinking?<br />

What is psychotherapy?<br />

What is the purpose of psychotherapy?<br />

What are the differences between<br />

psychodynamic psychotherapy and<br />

psychoanalysis?<br />

The superego (our conscience)<br />

Defense mechanisms<br />

It is associated with pleasure and<br />

instincts. This process does not involve<br />

logic or time and may not be linear.<br />

The id—this is not influenced by reality<br />

and focuses on pleasure and instinct<br />

(primary processes).<br />

A treatment technique. This treatment<br />

ranges from strengthening useful ego<br />

defenses (in supportive therapy) to<br />

challenging detrimental ego defenses<br />

and uncovering unconscious conflicts<br />

(in expressive and analytic<br />

psychotherapy), improving self-esteem,<br />

and improving relationships.<br />

To improve functionality. It helps to<br />

support defenses and functioning,<br />

reduce anxiety/stress, work on specific<br />

conflicts, improve coping skills, have<br />

healthier relationships, and enhance<br />

enjoyment/fulfillment in life.<br />

Time:<br />

• Psychoanalysis: 1 hour,<br />

3 to 5 × per week for 3 to<br />

8 years<br />

• Psychotherapy: 1 hour,<br />

1 to 2 × per week for 2 to<br />

20 years<br />

Level of therapist’s participation:<br />

• Psychoanalysis: The therapist mainly<br />

listens to the patient who lies on the<br />

couch and helps guide the patient,<br />

clarify, question, gently confront,<br />

encourage, interpret, analyze,<br />

and so forth.<br />

• Psychodynamic: The therapist<br />

is more interactive while both<br />

he or she and the patient sit in chairs<br />

facing each other, though the<br />

therapist still plays a guiding role.


Psychoanalytic Theory 27<br />

What are the three main<br />

techniques used in psychoanalysis<br />

and psychodynamic<br />

psychotherapy to uncover<br />

the unconscious?<br />

Which technique is used to describe<br />

the patient expressing “whatever<br />

comes to mind”?<br />

How does the unconscious mind<br />

manifest impulses, wishes,<br />

and fears?<br />

What is the purpose of a defense<br />

mechanism?<br />

What are the mature defense<br />

mechanisms?<br />

What are the immature defense<br />

mechanisms?<br />

Which defense mechanism causes a<br />

patient to take his uncomfortable<br />

feelings toward an unacceptable<br />

target and aim it at a more acceptable<br />

target?<br />

Which defense mechanism allows one<br />

to use a socially acceptable way to<br />

combat an unacceptable impulse?<br />

1. Free association<br />

2. Dream interpretations<br />

3. Transference interpretations<br />

Free association<br />

Through dreams, slips of the tongue,<br />

and forgetting significant things<br />

Maintaining the ego or decreasing<br />

anxiety and maintaining a sense<br />

of self<br />

Humor<br />

Altruism<br />

Sublimation<br />

Suppression<br />

Acting out<br />

Denial<br />

Displacement<br />

Dissociation<br />

Fixation<br />

Identification<br />

Intellectualization<br />

Isolation<br />

Projection<br />

Rationalization<br />

Reaction formation<br />

Regression<br />

Repression<br />

Splitting<br />

Displacement<br />

Sublimation


28 Deja Review: <strong>Behavioral</strong> <strong>Science</strong><br />

How does sublimation differ from<br />

displacement?<br />

Which defense mechanism allows one<br />

to find amusement in an otherwise<br />

difficult situation?<br />

If a patient uses intellectualization<br />

and isolation of affect as ways to<br />

deal with their discomfort, what<br />

personality disorder might<br />

he have?<br />

What defense mechanisms are highly<br />

associated with OCPD?<br />

How does rationalization differ from<br />

intellectualization?<br />

Which immature defense is used<br />

extensively by patients with borderline<br />

personality disorder who cannot<br />

integrate the good and bad aspects<br />

of the same person?<br />

In displacement, the object or person<br />

receiving the negative attention is<br />

“more tolerable” to the individual<br />

(kick dog instead of boss), while in<br />

sublimation, the negative attention<br />

is channeled into an activity that is<br />

socially acceptable (vigorous<br />

exercise to relieve stress caused<br />

by boss).<br />

Humor<br />

Obsessive-compulsive personality<br />

disorder (OCPD)<br />

Intellectualization (focus on facts about<br />

painful things instead of the painful<br />

things—eg, learning about treatments<br />

for cancer when diagnosed rather than<br />

feeling grief over the diagnosis.)<br />

Rationalization (making excuses that<br />

seem like a reasonable explanation—eg,<br />

I failed out of medical school because<br />

I didn’t really like biology anyway.)<br />

Isolation of affect (eg, I refuse to have<br />

emotions about this topic because<br />

emotions are too difficult.)<br />

Reaction formation (eg, I can’t deal with<br />

the fact that I hate you so I’ll give you a<br />

gift instead.)<br />

Rationalization occurs when a person<br />

uses excuses to explain an uncomfortable<br />

feeling related to an event or person.<br />

Intellectualization is when a person<br />

defers to factual information in order to<br />

deal with or understand uncomfortable<br />

feelings.<br />

Splitting


Psychoanalytic Theory 29<br />

Which defense mechanism leads a<br />

patient to deal with an uncomfortable<br />

situation by “placing it on the back<br />

burner”?<br />

How does repression differ from<br />

suppression?<br />

Which defense mechanism is associated<br />

with a person dealing with stressful<br />

situations in a childlike manner?<br />

In which personality disorder is<br />

repression, regression, and<br />

somatization used the most?<br />

Which defense mechanism allows us to<br />

place our bad feelings about ourselves<br />

onto others?<br />

How does displacement differ from<br />

projection?<br />

Which defense mechanism causes us<br />

to take on the positive and negative<br />

behaviors of others?<br />

Which personality disorders use<br />

projection and denial as their primary<br />

defense mechanisms?<br />

How does denial differ from splitting?<br />

Suppression<br />

Repression is an immature defense<br />

mechanism that occurs when the<br />

unconscious causes us to “forget”<br />

painful information while suppression<br />

is a mature defense mechanism that<br />

allows us to consciously put off painful<br />

information and “deal with it later” in<br />

order to maintain our composure.<br />

Regression<br />

Histrionic<br />

Projection<br />

Displacement allows us to place our<br />

negative emotion about someone else<br />

onto another target, while projection<br />

allows us to place our negative<br />

emotions about ourselves onto another<br />

target.<br />

Identification<br />

Paranoid personality disorder<br />

Schizotypal personality disorder<br />

Antisocial personality disorder<br />

Borderline personality disorder<br />

In denial, patients ignore all the bad<br />

aspects about something entirely (Bob’s<br />

dog died but he is still feeding him<br />

every night). This is not a conscious<br />

action. With splitting, patients cannot<br />

see the positive and the negative at the<br />

same time (their doctors are either<br />

“horrible” or “wonderful” but never<br />

“so-so”).


30 Deja Review: <strong>Behavioral</strong> <strong>Science</strong><br />

If a man “forgets” to return calls to all<br />

of the clients he dislikes today, which<br />

defense mechanism is at work?<br />

Which two personality disorders<br />

primarily utilize avoidance?<br />

If a man who recently embezzled<br />

money donates funds to a homeless<br />

shelter, what defense mechanism is<br />

he using?<br />

Which defense mechanism causes an<br />

individual to express unacceptable<br />

thoughts and feelings in a socially<br />

inappropriate manner?<br />

Which defense mechanism allows one<br />

to avoid a painful situation by acting as<br />

if it never happened?<br />

What is the emotional reaction a patient<br />

has toward their physician?<br />

What is the emotional reaction a<br />

physician has to a patient?<br />

What are the merits of recognizing<br />

countertransference?<br />

Avoidance<br />

Avoidant and dependent personality<br />

disorders<br />

Altruism—assisting others to avoid<br />

feeling bad about oneself<br />

Acting out<br />

Denial<br />

Transference. It occurs when patients<br />

unconsciously reexperience relationships<br />

from their past in their current<br />

relationship. This can occur in any<br />

relationship (not just with therapists or<br />

physicians). It can be a useful tool in<br />

therapy.<br />

Countertransference. It has two parts:<br />

1. Feelings the physician has toward<br />

the patient that are related to their<br />

own past (like transference).<br />

2. Feelings that a physician has toward<br />

the patient that resembles how the<br />

patient causes most people to feel<br />

(patient specific).<br />

The physician’s countertransference can<br />

influence the way the patient is treated.<br />

It is also a useful insight in psychotherapy<br />

to recognize how the patient makes<br />

others feel about them.


Table 4.1<br />

Defense<br />

Mechanism<br />

Description<br />

31<br />

Immature Mature<br />

Altruism<br />

Humor<br />

Sublimation<br />

Suppression<br />

Acting Out<br />

Denial<br />

Displacement<br />

Dissociation<br />

Identification<br />

Intellectualization<br />

Projection<br />

Rationalization<br />

Reaction formation<br />

Regression<br />

Repression<br />

Splitting<br />

Guilty feelings are alleviated by kindness to others<br />

Recognizing the amusing nature of an anxiety-provoking or adverse situation<br />

Channeling unacceptable impulses into acceptable actions<br />

Voluntary blocking of an idea or feeling from conscious awareness<br />

Undesirable feelings and thoughts are expressed through actions<br />

Refusing to accept facts about an unpleasant reality<br />

Transferring feeling about one thing to something else<br />

Detaching from oneself; emotionally or mentally<br />

Modeling one’s behavior after someone more powerful<br />

Using facts and logic to avoid emotions<br />

Attributing one’s feelings to an external source<br />

Proclaiming logical reasons for actions actually performed for other reasons<br />

Process whereby a warded-off idea or feeling is replaced by an unconsciously derived<br />

emphasis on its opposite<br />

Returning to a less mature stage of development<br />

Involuntary blocking of an idea or feeling from conscious awareness<br />

Belief that people are either all good or all bad<br />

(Continued)


Table 4.1 (Continued )<br />

Defense<br />

Mechanism<br />

Example<br />

32<br />

Immature Mature<br />

Altruism<br />

Humor<br />

Sublimation<br />

Suppression<br />

Acting Out<br />

Denial<br />

Displacement<br />

Dissociation<br />

Identification<br />

Intellectualization<br />

Projection<br />

Rationalization<br />

Reaction formation<br />

Regression<br />

Repression<br />

Splitting<br />

Criminal makes large donation to charity.<br />

Nervous surgeon jokes about an upcoming procedure.<br />

Aggressiveness used to succeed in business ventures.<br />

Choosing not to think about life stresses during work.<br />

A teenager punches a hole in the wall.<br />

An HIV-positive man adamantly states he is healthy.<br />

Mother yells at her child because she is angry at her husband.<br />

Floating above one’s body during a traumatic event.<br />

An abused child becomes a bully.<br />

A newly diagnosed cancer patient calmly describes the pathogenesis of their disease.<br />

A woman who fantasizes about another man thinks her husband is cheating on her.<br />

Saying a job was not important, after getting fired.<br />

A patient with sexual thoughts enters a monastery.<br />

An adult clutching a sentimental teddy bear.<br />

Traumatic childhood events remain buried in the unconscious.<br />

A patient says that all men are cold and insensitive but that women are warm and friendly.


Psychoanalytic Theory 33<br />

CLINICAL VIGNETTES<br />

A middle-aged woman is in a grocery store. She goes to the bulk candy section and<br />

sees gummy bears—her favorite. Part of her mind says she should grab a handful<br />

and stuff them in her mouth. Which part of the mind is telling her this?<br />

The id<br />

A man and a woman are in a car accident. Neither has insurance and both cars are<br />

totaled, and both of them lose their licenses. The woman goes home and says<br />

“Well, at least I wasn’t driving a Ferrari—that would have been much more<br />

expensive.” The man says “I hit her because I was swerving to avoid something<br />

else. I should have had my brakes balanced.” What defense mechanisms are they<br />

using and are they mature or immature defenses?<br />

The woman is using humor—a mature mechanism. The man is using rationalization—<br />

an immature mechanism.<br />

A medical student is mistreated by his professor, but later tells his fellow medical<br />

students that the professor is “great”! What defense mechanism is he using?<br />

Reaction formation<br />

A medical student’s dog dies. When he is consoled by fellow students, he states,<br />

“He was really old and I was expecting him to go soon anyway. Most large dogs<br />

only live until they are 10.” What defense mechanism is he using?<br />

Intellectualization<br />

A medical student fails the last anatomy exam after he studied 12 hours per day in<br />

the corner of the library for 1 month. He tells his friend, “I think I failed because<br />

I didn’t study enough. Besides, I am not really going to use anatomy later on.”<br />

What defense mechanism is he using?<br />

Rationalization


This page intentionally left blank


CHAPTER 5<br />

Learning Theory<br />

What is behaviorism?<br />

It is a learning theory based on the idea<br />

that behavior is a product of learning<br />

through association or reinforcement.<br />

ASSOCIATIVE LEARNING VS NONASSOCIATIVE LEARNING<br />

What is associative learning?<br />

What are associative learning processes?<br />

What is nonassociative learning?<br />

What are nonassociative learning<br />

processes?<br />

Learning that occurs when a connection<br />

or pairing is made between a particular<br />

stimulus and a particular response<br />

Classical conditioning<br />

Operant conditioning<br />

Nonassociative learning describes<br />

behavior change as a result of<br />

presentation of one stimulus repeatedly.<br />

It also describes learning which has no<br />

association with an end stimulus (such<br />

as a reward or punishment).<br />

Observational learning<br />

Habituation<br />

Sensitization<br />

Associative Learning<br />

Imprinting<br />

What is imprinting?<br />

Imprinting describes learning that<br />

happens at a specific (usually early)<br />

developmental stage. It is usually rapid<br />

and is unrelated to the consequences of<br />

the behavior being learned. During this<br />

time, the animal or person imitates the<br />

behavior of another stimulus.<br />

35


36 Deja Review: <strong>Behavioral</strong> <strong>Science</strong><br />

What is an example of imprinting?<br />

When birds follow the first thing they<br />

see moving after they hatch. The critical<br />

time period is the few moments after<br />

birth and the association is made so<br />

quickly that the first object that they<br />

see suitably moving is what they will<br />

follow.<br />

Classical Conditioning<br />

What is classical conditioning?<br />

Classical conditioning is a way of<br />

pairing a stimulus and response. As<br />

demonstrated by Pavlov, a novel<br />

stimulus (a ringing bell) can be paired<br />

with an unconditioned stimulus (food)<br />

to elicit an unconditioned or natural<br />

response (salivation).<br />

Thus, if a bell (novel stimulus) is<br />

rung every time food (unconditioned<br />

stimulus) is presented to the dog,<br />

it will be conditioned to associate<br />

the bell with food and will learn to<br />

salivate (natural response) at the<br />

sound of the bell. This is the<br />

conditioned response.<br />

CS<br />

2<br />

US<br />

1<br />

3<br />

UR<br />

CR<br />

US: Unconditioned stimulus<br />

CS: Conditioned stimulus<br />

UR: Unconditioned response<br />

CR: Conditioned response<br />

1. Normally, unconditioned stimulus leads to<br />

unconditioned response.<br />

2. Conditioned stimulus paired with unconditioned<br />

stimulus.<br />

3. Now conditioned stimulus also leads to the response<br />

—termed conditioned response.<br />

Note: UR and CR are the<br />

accurate terminology; the<br />

response itself is still the same.<br />

Bell<br />

3<br />

Doctor’s office<br />

3<br />

2<br />

Salivation<br />

2<br />

Nausea<br />

Food Salivation<br />

1<br />

Pavlov’s example<br />

Figure 5.1<br />

Chemotherapy Nausea<br />

1<br />

Medical example


Learning Theory 37<br />

What is the term describing a similar<br />

stimulus to the conditioned stimulus<br />

eliciting the same response?<br />

What is a medical example of classical<br />

conditioning?<br />

What is extinction?<br />

Is it possible for the conditioned<br />

response to be paired with the<br />

conditioned stimulus after extinction<br />

has taken place?<br />

What does learned helplessness mean?<br />

What mood disorder may be explained<br />

by the theory of learned helplessness?<br />

Stimulus generalization. The classic<br />

example was “Little Albert” who was<br />

conditioned to fear a white rat and was<br />

subsequently afraid of other white<br />

fuzzy things.<br />

Phobias are believed to be results of<br />

classical conditioning. For example,<br />

Mary had a frightening experience on a<br />

ship. She may generalize that fear so that<br />

even the sight of a ship causes her<br />

anxiety.<br />

It is the disappearance of the conditioned<br />

response if the conditioned and<br />

unconditioned stimuli are no longer<br />

presented together.<br />

Yes, this is called spontaneous recovery.<br />

The effect of repeatedly pairing an<br />

adverse stimulus to the inability to<br />

escape, leading to thoughts that no<br />

efforts will be successful<br />

It has been thought that this theory may<br />

explain depression in humans. In this<br />

theory, a person has tried repeatedly but<br />

unsuccessfully to control external events.<br />

The person then pairs any adverse event<br />

to the inability to do anything about<br />

them. The person then becomes hopeless,<br />

depressed, and apathetic.<br />

Operant Conditioning<br />

What is operant conditioning?<br />

What are the different consequences<br />

in operant conditioning?<br />

It is the idea that a behavior is learned<br />

because of the reward or punishment<br />

associated with it.<br />

Positive reinforcement<br />

Negative reinforcement<br />

Punishment<br />

Extinction


38 Deja Review: <strong>Behavioral</strong> <strong>Science</strong><br />

What are reinforcers?<br />

What is positive reinforcement?<br />

What is negative reinforcement?<br />

What type of reinforcement increases<br />

the likelihood of a behavior occurring<br />

again?<br />

What will decrease the likelihood<br />

of a behavior occurring?<br />

What is shaping?<br />

When is shaping used?<br />

What is the definition of punishment?<br />

What is the definition of extinction<br />

in operant conditioning?<br />

What is an example of extinction?<br />

Any event or stimulus that increases the<br />

likelihood of the behavior occurring<br />

again<br />

Presentation of a rewarding stimulus<br />

after a certain behavior is performed.<br />

For example, parents may reward their<br />

children with ice cream (positive<br />

reinforcer) when their rooms are clean,<br />

thus increasing the likelihood that the<br />

children will clean their rooms.<br />

Removal of an aversive stimulus after a<br />

certain behavior is performed. For<br />

example, parents may exempt the<br />

children that clean their rooms from<br />

having to take out the trash, thus<br />

increasing the likelihood that the<br />

children will clean their rooms.<br />

Positive and negative reinforcement<br />

Punishment (note the contrast between<br />

negative reinforcement and punishment)<br />

Shaping is learning that occurs when a<br />

person is rewarded for a behavior<br />

which is similar to a desired behavior.<br />

Subsequently, only behavior which is<br />

more and more similar to the particular<br />

desired behavior is rewarded.<br />

It is a progressive modification of<br />

behavior which occurs by reinforcement<br />

of behavior which is close to the desired<br />

outcome.<br />

Presentation of an aversive stimulus to<br />

reduce the likelihood of an unwanted<br />

behavior occurring<br />

Extinction is the disappearance of a<br />

certain behavior when the reinforcement<br />

is no longer present.<br />

A rat that is initially trained to press a<br />

bar if rewarded with food will quickly<br />

cease to press the bar if food is no<br />

longer obtained by the behavior.


Learning Theory 39<br />

What are the different schedules of<br />

reinforcement?<br />

Which reinforcement shows the fastest<br />

extinction when reinforcement is<br />

taken away?<br />

What is variable ratio reinforcement?<br />

What is the difference between<br />

classical and operant conditioning?<br />

There are five types of reinforcement<br />

schedules:<br />

1. Continuous: every time behavior is<br />

performed<br />

2. Fixed ratio: set number of times<br />

3. Variable ratio: random number of<br />

times<br />

4. Fixed interval: set amount of time<br />

5. Variable interval: random amount of<br />

time<br />

Continuous<br />

Reinforcement is given at a variable<br />

time interval after the behavior is<br />

performed. This type of reinforcement<br />

shows the slowest extinction when the<br />

reinforcement is taken away. Slot<br />

machines are an example of variable<br />

ratio reinforcement.<br />

Classical conditioning refers to behaviors<br />

learned by association of stimuli and<br />

responses whereas operant conditioning<br />

refers to behaviors learned by the<br />

reward and reinforcement associated<br />

with them.<br />

NON-ASSOCIATIVE LEARNING<br />

What is observational learning?<br />

If a person observes others and then<br />

imitates their behavior, what is that<br />

behavior called?<br />

What four aspects are needed in order<br />

for a person to be able to model?<br />

In observational learning, the observer’s<br />

behaviors change based upon observing<br />

the model’s behaviors. The consequences<br />

of the model’s behaviors, whether they<br />

are positive or negative, have an effect<br />

on the observer’s behaviors.<br />

Modeling. Modeling is a type of<br />

observational learning. Compared to<br />

operant learning it is a more efficient<br />

and faster type of learning. Modeling is<br />

useful in acquiring new skills.<br />

1. Attention to the model<br />

2. Retention of details<br />

3. Motor reproduction<br />

4. Motivation and opportunity


40 Deja Review: <strong>Behavioral</strong> <strong>Science</strong><br />

Where in medicine is modeling used?<br />

Can modeling have a negative outcome?<br />

Can modeling have a positive outcome?<br />

What is habituation?<br />

What is an example of habituation?<br />

How is habituation used in medicine?<br />

How do you describe flooding?<br />

What is an example of flooding?<br />

How do you describe systematic<br />

desensitization?<br />

The common saying, “see one, do one,<br />

teach one,” is a description of modeling<br />

in learning how to do medical<br />

procedures.<br />

Yes. Modeling may occur when a child<br />

models the actions of a parent with a<br />

particular phobia and hence also acquires<br />

the same phobia or perpetuation of<br />

abuse by an abused person.<br />

Yes. Modeling may involve other<br />

types of learning, eg, seeing a role<br />

model/mentor’s behavior achieve<br />

a positive result would then act as<br />

a positive reinforcer (which would<br />

be an example of operant<br />

conditioning).<br />

Habituation occurs when stimulus<br />

presentation results in decreased<br />

responsiveness.<br />

You may notice the hum of the air<br />

conditioner when it first comes on,<br />

but due to habituation, your<br />

awareness of that continual hum<br />

will decrease and you can focus<br />

on your studies.<br />

Habituation is used to overcome<br />

phobias. Some of the specific techniques<br />

using habituation are flooding and<br />

systemic desensitization.<br />

It is excessive presentation of the<br />

stimulus to achieve quick habituation<br />

by preventing escape and forcing<br />

a reduction in the associated<br />

behavior.<br />

One could force an individual with<br />

an obsession about germs to touch<br />

a toilet.<br />

In systematic desensitization, the<br />

patient is gradually exposed to<br />

anxiety-producing situations while<br />

simultaneously teaching relaxation or<br />

anxiety-reducing techniques.


Learning Theory 41<br />

What is sensitization?<br />

What is an example of sensitization?<br />

Sensitization occurs when stimulus<br />

presentation results in increased<br />

responsiveness and/or generalization of<br />

response to other stimuli.<br />

Joe is alone in a dark house when he<br />

hears a sudden loud noise. He suddenly<br />

becomes more aware of every little<br />

sound in the house.<br />

Intelligence Tests<br />

Which term is used to describe the<br />

ability to learn, understand, or to deal<br />

with new or trying situations?<br />

Which term did Binet use to describe<br />

the average intellectual age of people<br />

with a specific chronological age?<br />

Which scale is used to determine a<br />

person’s IQ?<br />

Intelligence<br />

Mental age<br />

Stanford-Binet scale<br />

How is the IQ calculated? (Mental age/chronological age) × 100<br />

If a person has an IQ of 100, what does<br />

that indicate?<br />

If a person has an IQ


42 Deja Review: <strong>Behavioral</strong> <strong>Science</strong><br />

CLINICAL VIGNETTES<br />

An 8-year-old girl with acute myelogenous leukemia (AML) gets weekly<br />

chemotherapy treatments. The chemotherapy makes her very nauseated, but her<br />

mother makes up for it by taking her to an ice cream shop by the hospital. Her<br />

favorite flavor is maple nut. After the first month of treatment, she will no longer<br />

eat ice cream as it causes intense nasusea.<br />

What are the unconditioned stimulus (US), unconditioned response (UR),<br />

conditioned stimulus (CS), and conditioned response (CR) in this situation?<br />

US: chemotherapy; UR: nausea; CS: ice cream; CR: nausea<br />

The patient also refuses to eat pancakes with maple syrup. What is this<br />

phenomenon?<br />

Ice cream has been paired with chemotherapy here. The patient has also generalized<br />

the maple ice cream to the syrup, eliciting the same nausea.<br />

After the patient’s AML is in remission and her treatment is over, her parents<br />

slowly reintroduce her to eating progressively larger amounts of ice cream while<br />

listening to her favorite calming music. What is this process called?<br />

Her parents are later using the process of systematic desensitization to unpair the<br />

CS with the US.<br />

A rat is being conditioned to press a lever in its cage, but not the button next to it.<br />

Name the operant conditioning consequence or phenomenon represented by each<br />

statement:<br />

The rat is given a piece of cheese when it gets close to the lever. Subsequent<br />

pieces of cheese are given as it gets closer and closer, finally resulting in a big<br />

piece when it finally presses it.<br />

Shaping<br />

The rat receives a piece of cheese every time the lever is pressed.<br />

Positive reinforcement<br />

The rat cage floor is electrified, but pressing the lever deactivates it.<br />

Negative reinforcement (note the behavior will increase)<br />

The rat presses the button and a loud and terrifying noise is emitted.<br />

Punishment (note the behavior will decrease)<br />

The rat continues to press the lever, but nothing happens. He eventually stops<br />

pressing the lever.<br />

Extinction<br />

The floor of the cage is electrified by the cruel scientist, and no amount of lever<br />

or button pressing will turn it off. The rat gives up.<br />

Learned helplessness


CHAPTER 6<br />

Sleep <strong>Science</strong> and<br />

Disorders<br />

NORMAL SLEEP<br />

What are the normal stages of sleep?<br />

REM (rapid eye movement) and NREM<br />

(nonrapid eye movement). NREM is<br />

divided into four stages: 1, 2, 3, and 4.<br />

Awake<br />

REM<br />

Sleep stage<br />

Stage 1<br />

Stage 2<br />

Stage 3<br />

Stage 4<br />

Time asleep<br />

Figure 6.1<br />

What is slow wave sleep? Slow wave sleep occurs during stages 3<br />

and 4 of NREM sleep. It is also known<br />

as delta sleep and is the deepest portion<br />

of sleep. Electroencephalogram (EEG)<br />

shows delta waves, which are the<br />

lowest frequency waves.<br />

Delta waves = Deep sleep<br />

43


44 Deja Review: <strong>Behavioral</strong> <strong>Science</strong><br />

What waveforms are seen in<br />

REM sleep?<br />

Where else are beta waves seen?<br />

What waveforms are associated with<br />

Stages 1 and 2 of NREM?<br />

Beta waves. These are of highest<br />

frequency.<br />

They are found over the frontal lobes in<br />

a person who is awake with their eyes<br />

open. These are associated with a<br />

person who is alert and actively<br />

concentrating. In a person who is awake<br />

with their eyes closed, alpha waves are<br />

seen typically over the occipital and<br />

posterior parietal lobes.<br />

Stage 1 (light sleep) is associated with<br />

theta waves and Stage 2 (deeper sleep)<br />

is associated with sleep spindles and<br />

K-complexes.<br />

On average, how much time does a REM: 25%<br />

normal adult spend in each stage NREM: 75%<br />

of sleep? 1. Stage 1: 5%<br />

2. Stage 2: 45%<br />

3. Stage 3: 25%<br />

4. Stage 4: 25%<br />

What is REM latency?<br />

What is the length of REM latency<br />

in an adult?<br />

Besides rapid eye movements, what<br />

other physiologic changes occur in<br />

REM sleep?<br />

How is this different from the<br />

physiologic changes in NREM sleep?<br />

Which neurotransmitter usually<br />

initiates sleep?<br />

REM latency is the length of time after<br />

falling asleep before REM sleep occurs.<br />

REM latency in an adult is approximately<br />

90 minutes. The REM sleep cycle then<br />

repeats itself approximately every<br />

90 minutes thereafter.<br />

Pulse, respiration, blood pressure, and<br />

brain oxygen use increase. There is<br />

penile/clitoral erection, dreaming, and<br />

decreased skeletal muscle tone.<br />

In NREM, blood pressure, pulse, and<br />

respiration are slow. There may also be<br />

intermittent limb movements.<br />

Tip: This makes sense—if your blood<br />

pressure (BP) and pulse are up and<br />

your genitals are aroused, NREM sleep<br />

wouldn’t be very restful!<br />

Serotonin. It is released from the dorsal<br />

raphe nucleus and is a derivative of<br />

tryptophan. It increases total sleep time<br />

and slow wave activity.<br />

Tip: Turkey is high in tryptophan—this<br />

is why you get sleepy after a big<br />

Thanksgiving dinner.


Sleep <strong>Science</strong> and Disorders 45<br />

Which neurotransmitters are involved<br />

in REM sleep?<br />

How does REM change with age?<br />

What effect does dopamine have on the<br />

sleep cycle?<br />

Acetylcholine (ACh) from the basal<br />

forebrain and norepinephrine (NE) from<br />

the locus ceruleus. ACh increases REM<br />

sleep and NE decreases it.<br />

Time spent in REM decreases with age.<br />

Dopamine increases wakefulness. Thus,<br />

antipsychotics, which block dopamine,<br />

can result in increased sedation.<br />

Stimulants, which increase both NE and<br />

dopamine, promote wakefulness.<br />

ABNORMAL SLEEP<br />

Approximately how many adults<br />

experience sleep disorders every year?<br />

What is the most common type<br />

of sleep disorder?<br />

What is a primary sleep disorder?<br />

What is the difference between<br />

dyssomnias and parasomnias?<br />

Name the five major dyssomnias<br />

How long must you have problems<br />

with insomnia before primary<br />

insomnia can be diagnosed?<br />

About one in three<br />

Insomnia, which includes problems<br />

initiating and maintaining sleep.<br />

Sleep disturbances that arise from<br />

endogenous sources, not from substance<br />

use, medical problems, or other<br />

psychiatric problems. Primary sleep<br />

disorders are divided into two major<br />

categories:<br />

1. Parasomnias<br />

2. Dyssomnias<br />

Dyssomnias are due to dysfunctional<br />

sleep regulation characterized by<br />

problems initiating or maintaining<br />

sleep, or excessive daytime sleepiness.<br />

Parasomnias involve abnormal behaviors<br />

or physiologic events during sleep,<br />

rather than abnormal functioning of the<br />

usual mechanisms of sleep. These include<br />

sleep terror disorder, sleepwalking<br />

disorder, and nightmare disorder.<br />

1. Primary insomnia<br />

2. Primary hypersomnia<br />

3. Narcolepsy<br />

4. Breathing-related sleep disorder<br />

5. Circadian rhythm sleep disorder<br />

At least 1 month


46 Deja Review: <strong>Behavioral</strong> <strong>Science</strong><br />

What interventions other than<br />

medications can be useful<br />

in insomnia?<br />

What is the subtype of recurrent<br />

primary hypersomnia associated with<br />

obesity, impulsivity, hyperphagia,<br />

hypersexuality, and disorganized<br />

thought called?<br />

Other than daytime sleepiness and<br />

“sleep attacks,” what symptoms are<br />

classically associated with narcolepsy?<br />

Which class of drugs is normally used<br />

to treat narcolepsy?<br />

Why are people with breathing-related<br />

sleeping disorder chronically sleepy<br />

during the day?<br />

What is the most common cause<br />

of breathing-related sleep disorder<br />

and how is it treated?<br />

In a patient with excessive fatigue,<br />

what might the patients’ bed partner<br />

tell you about their sleep that might<br />

lead you to suspect obstructive sleep<br />

apnea (OSA)?<br />

What are the complications from<br />

untreated OSA?<br />

Set a regular bedtime, abstain from<br />

caffeine and alcohol, use the bed only<br />

for sleep and sex, and avoid daytime<br />

naps and strenuous exercise or large<br />

meals just before bedtime. Collectively,<br />

this is referred to as sleep hygiene.<br />

Kleine-Levin syndrome<br />

Cataplexy: sudden loss of muscle<br />

tone associated with strong emotions<br />

Hypnagogic and hypnopompic<br />

hallucinations: REM intrusions that<br />

occur during the transition period<br />

between sleep and wakefulness<br />

(Hypnagogic symptoms occur when<br />

going to sleep and hypnopompic occur<br />

while waking up.)<br />

Sleep paralysis: inability to move just<br />

before going to sleep or awakening<br />

Stimulants, eg, Ritalin<br />

During the night they frequently stop<br />

breathing and then are awoken by<br />

hypoxia. These frequent arousals<br />

prevent the patients from getting deep,<br />

restful sleep.<br />

Obstructive sleep apnea (OSA)<br />

Treatment: continuous positive airway<br />

pressure (CPAP) and possibly removal<br />

of tonsils and adenoids (ideally weight<br />

loss would be primary intervention<br />

in the obese.)<br />

Loud snoring and periods of time<br />

where the patient appears to stop<br />

breathing<br />

Hypertension, pulmonary hypertension,<br />

and increased all-cause mortality,<br />

cardiovascular disease, and<br />

cerebrovascular disease


Sleep <strong>Science</strong> and Disorders 47<br />

In a person with sleep problems related<br />

to OSA, why might you avoid<br />

benzodiazepines?<br />

What are the three most common causes<br />

of a circadian rhythm sleep disorder?<br />

During which phase of sleep would<br />

you expect nightmares to occur in?<br />

What are the similarities between sleep<br />

terrors and sleepwalking?<br />

How could you clinically differentiate<br />

sleep terror from sleepwalking?<br />

What is the treatment of sleep terrors<br />

and sleepwalking?<br />

You risk further compromising the<br />

patient’s ventilation by respiratory<br />

depression.<br />

1. Delayed sleep phase—“night owls,”<br />

more common in adolescents and<br />

tends to improve with age<br />

2. Jet lag—typically resolves over<br />

several days<br />

3. Shift work—eg, working the<br />

night shift<br />

REM<br />

Both are more common in children and<br />

may involve semicomplex to complex<br />

motor behaviors. Patients tend to be<br />

amnestic for both and in adulthood they<br />

are equally prevalent in men and<br />

women (1% prevalence of sleep terrors<br />

and 2% prevalence of sleepwalking).<br />

Both occur in slow wave sleep.<br />

Sleep terror has strong component of<br />

autonomic arousal and fear, often<br />

beginning with a terrified scream, and a<br />

lesser element of semipurposeful motor<br />

behaviors. Sleepwalking has minimal<br />

autonomic arousal/fear and motor<br />

behaviors are usually more complex.<br />

Children will usually grow out of them.<br />

Benzodiazepines can be helpful for<br />

adults, as can scheduled awakenings<br />

and environmental control.<br />

OTHER SLEEP CHANGES<br />

What polysomnogram (PSG) findings<br />

are characteristic of major depression?<br />

Reduced slow wave sleep (less delta<br />

waves), frequent nighttime awakenings,<br />

increased sleep latency (time until<br />

falling asleep), short REM latency<br />

(REM cycle starts sooner than normal<br />

90 minutes onset), and early morning<br />

waking.<br />

Tip: Low serotonin is associated with<br />

depression; therefore you would expect<br />

reduced total sleep and slow wave sleep.


48 Deja Review: <strong>Behavioral</strong> <strong>Science</strong><br />

A common feature of Alzheimer is<br />

reduced ACh. Given this, what PSG<br />

changes are seen in an individual<br />

with Alzheimer?<br />

Decreased slow wave and REM sleep<br />

CLINICAL VIGNETTES<br />

A 51-year-old obese man presents to your clinic because of fatigue. He states that<br />

he is sleepy all the time and will often fall asleep at inopportune times—at<br />

meetings, during lunch, and even behind the wheel. He says that he goes to bed at<br />

10 PM and wakes up at 6 AM every morning. He denies any substance use. What<br />

should he be screened for?<br />

Sleep apnea. Obese patients are at increased risk for this and can often present as<br />

daytime sleepiness as sleep is not restorative. He should be evaluated with a sleep<br />

study. First-line treatment is CPAP.<br />

A 32-year-old woman is being evaluated because of strange “spells”. She often<br />

seems to “pass out” when she is startled or when she laughs quite hard. What<br />

sleep disorder is she most likely suffering from?<br />

Narcolepsy. The described phenomenon is cataplexy.<br />

An 83-year-old woman presents to your office because she feels as though she is<br />

spending less time getting a restful sleep at night. She has heard from her friends<br />

that there are changes in sleep that occur with aging. What stages of sleep tend to<br />

decrease with age?<br />

Both REM and later stage deep sleep (3 and 4) decrease in the elderly, as well as a<br />

decrease in overall quantity of sleep.


CHAPTER 7<br />

Sexuality<br />

SEXUAL DEVELOPMENT<br />

Which sex is the default pattern for<br />

sexual development?<br />

Gonad differentiation is dependent<br />

upon the presence of which<br />

chromosome?<br />

Which gene present on the<br />

Y chromosome influences gonad<br />

development?<br />

Which duct system present in male<br />

embryos helps form genitalia?<br />

Which duct system in female embryos<br />

help form female genitalia?<br />

Which organ secretes hormones that<br />

direct the differentiation of male<br />

internal and external genitalia?<br />

How does exposure to different levels<br />

of hormones during prenatal life<br />

influence humans?<br />

What term describe an individual’s<br />

sense of being male or female?<br />

At which age(s) does gender awareness<br />

become evident?<br />

What term describes the expression<br />

of gender identity in society?<br />

What term describes the conflict people<br />

experience when they feel as if they<br />

were born as the wrong gender?<br />

Female<br />

Y chromosome<br />

SRY Gene, which makes testisdetermining<br />

factor<br />

Wolffian duct system<br />

Mullerian duct system<br />

Testes<br />

It causes gender differences in certain<br />

areas of the brain.<br />

Gender identity<br />

2 or 3 years of age<br />

Gender role<br />

Gender identity disorder<br />

49


50 Deja Review: <strong>Behavioral</strong> <strong>Science</strong><br />

According to the DSM-IV-TR<br />

is homosexuality a dysfunction<br />

of sexual expression?<br />

What is the difference between sex<br />

and gender?<br />

What type of behavior in childhood<br />

may be predictive of later homosexual<br />

orientation?<br />

Which hormones have been shown<br />

to influence sexual orientation?<br />

Which evidence has been reported,<br />

which indicates that genetics plays<br />

a role in homosexuality?<br />

On which chromosome have genetic<br />

markers been found in homosexuals?<br />

No, it is a normal variant.<br />

Sex refers to the genetics of the<br />

individual (XX or XY), gender is<br />

what a person identifies as<br />

(male or female).<br />

Cross-gender behavior → stronger<br />

correlation in males<br />

Prenatal hormones → low levels of<br />

androgens in males and high levels of<br />

androgens in females<br />

Higher concordance rate observed in<br />

monozygotic twins than in dizygotic<br />

twins<br />

X chromosome<br />

SEXUAL DEVELOPMENT AND PHYSIOLOGIC ABNORMALITIES<br />

Which disorder is characterized by<br />

cells that are not responsive to<br />

androgens and testicles that may<br />

appear as inguinal or labial masses?<br />

What is the genotype of a person with<br />

androgen insensitivity?<br />

What is the phenotype of a person with<br />

androgen insensitivity?<br />

Which disorder is characterized by an<br />

adrenal gland that is unable to produce<br />

the proper amount of cortisol which<br />

leads to a significantly increased<br />

androgen secretion?<br />

What is the genotype of a person with<br />

congenital adrenal hyperplasia?<br />

What is the phenotype of a person with<br />

congenital adrenal hyperplasia?<br />

Androgen insensitivity which is also<br />

called testicular feminization<br />

XY<br />

Female<br />

Congenital adrenal hyperplasia<br />

XX<br />

Female with genitalia that are masculine


Sexuality 51<br />

What is the sexual orientation of 33%<br />

of patients with congenital adrenal<br />

hyperplasia?<br />

Which disorder is characterized by a<br />

short stature, webbed neck, and<br />

streak ovaries?<br />

What is the genotype of a person with<br />

Turner syndrome?<br />

What is the phenotype of a person with<br />

Turner syndrome?<br />

Homosexual<br />

Turner syndrome<br />

XO<br />

Female<br />

HORMONES AND THEIR INFLUENCE ON BEHAVIOR<br />

Which hormone may be decreased by<br />

an increase in stress?<br />

Which three hormones commonly used<br />

in medical treatment of conditions such<br />

as prostate cancer decrease androgen<br />

production and in turn reduces sexual<br />

interest and behavior?<br />

Which hormone is believed to play the<br />

most important role in sex drive in<br />

both genders?<br />

Which hormone may decrease sexual<br />

behavior and interest in women?<br />

Testosterone<br />

1. Androgen antagonists<br />

2. Estrogens<br />

3. Progesterone<br />

Testosterone<br />

Progesterone → this hormone is in many<br />

oral contraceptives.<br />

SEXUAL RESPONSE CYCLE<br />

What are the four stages in the sexual<br />

response cycle developed by Masters<br />

and Johnson?<br />

What is the primary characteristic of<br />

the excitement stage in men?<br />

What are characteristics of the<br />

excitement stage in women?<br />

1. Excitement<br />

2. Plateau<br />

3. Orgasm<br />

4. Resolution<br />

Penile erection<br />

Clitoral erection<br />

Vaginal lubrication<br />

Labial swelling<br />

Uterus raises in pelvic cavity → tenting<br />

effect


52 Deja Review: <strong>Behavioral</strong> <strong>Science</strong><br />

What characteristics of the excitement<br />

stage are common to men and women?<br />

What are characteristics of the recovery<br />

stage in men?<br />

What happens to the refractory period<br />

of men as they age?<br />

What is the primary characteristic<br />

of the recovery stage in women?<br />

What are characteristics of the refractory<br />

period that are common to both men<br />

and women?<br />

Nipple erection<br />

Elevation of blood pressure, pulse, and<br />

respiration<br />

Refractory period in which stimulation<br />

is not possible<br />

Increases<br />

Minimal or no refractory period<br />

Physiological systems return to their<br />

prestimulated states (eg, cardiovascular,<br />

respiratory)<br />

Muscle relaxation<br />

SEXUAL DYSFUNCTION<br />

Which term(s) describe problems in<br />

stages of the sexual response cycle?<br />

Which disorder is characterized by<br />

pain associated with sexual intercourse?<br />

Which gender is most likely to<br />

experience dyspareunia?<br />

Which disorder is characterized by<br />

inability to maintain vaginal lubrication<br />

throughout the duration of a sexual act?<br />

Which disorder is characterized by<br />

decreased interest in sexual activity?<br />

Which disorder is characterized by<br />

problems with maintaining erections?<br />

Which disorder is characterized by an<br />

inability to achieve an orgasm?<br />

Which disorder is characterized by<br />

anxiety and ejaculation before<br />

a man desires?<br />

Which stage of the sexual response<br />

cycle is affected by premature<br />

ejaculation?<br />

Sexual dysfunction<br />

Dyspareunia<br />

Females<br />

Female sexual arousal disorder → occurs<br />

in approximately 20% of women<br />

Hypoactive sexual desire<br />

Male erectile disorder also called<br />

impotence<br />

Orgasmic disorder → this disorder can<br />

be lifelong or acquired.<br />

Premature ejaculation<br />

Plateau phase → absent or reduced


Sexuality 53<br />

Which disorder is characterized by<br />

avoidance or aversion to sexual<br />

activity?<br />

Which disorder is characterized by<br />

painful muscular spasms in the outer<br />

one-third of the vagina making pelvic<br />

examination or sexual intercourse<br />

difficult?<br />

Which behavioral treatment stimulates<br />

a person’s senses during sexual<br />

activity to reduce the pressure one<br />

experiences when trying to achieve<br />

an erection or orgasm?<br />

What is the primary goal of the squeeze<br />

technique?<br />

Which segment of the nervous system<br />

is used to initiate an erection?<br />

Which segment of the nervous system<br />

is used to initiate an ejaculation?<br />

What is a mnemonic to remember this?<br />

Which behavioral technique(s) are used<br />

to reduce anxiety associated with<br />

sexual performance?<br />

Which drug is used to treat erectile<br />

dysfunction by blocking the<br />

phosphodiester-5 (PDE-5) enzyme<br />

thereby inhibiting cyclic guanosine<br />

monophosphate (cGMP)?<br />

What is the role of cGMP in sexual<br />

stimulation of the penis?<br />

Which drug is used to increase the<br />

availability of dopamine in the brain<br />

in patients with erectile disorder<br />

and female arousal disorder?<br />

What injection method is used to treat<br />

erectile dysfunction?<br />

Which vasodilators are commonly<br />

injected in intracorporeal injection?<br />

Sexual aversion disorder<br />

Vaginismus<br />

Sensate-focus exercise<br />

To treat premature ejaculation<br />

Parasympathetic<br />

Sympathetic<br />

Point and Shoot<br />

Hypnosis<br />

Relaxation techniques<br />

Systematic desensitization<br />

Sildenafil citrate (Viagra)<br />

It is a vasodilator which allows an<br />

erection to persist.<br />

Apomorphine (Uprima)<br />

Intracorporeal injection of vasodilators<br />

Papaverine<br />

Phentolamine


54 Deja Review: <strong>Behavioral</strong> <strong>Science</strong><br />

PARAPHILIAS<br />

What term describes the use of<br />

unusual objects of sexual desire or<br />

unusual sexual activities?<br />

Which paraphilia is most common and<br />

is characterized by a person who<br />

achieves sexual gratification from<br />

children


Sexuality 55<br />

Which postmyocardial infarction<br />

patients can resume sexual activity?<br />

Which problem is common in<br />

diabetic men?<br />

What are the two main causes of<br />

erectile dysfunction in diabetics?<br />

What is the primary treatment for<br />

erectile dysfunction in diabetics?<br />

What effect does spinal cord<br />

dysfunction have on sexual<br />

functioning in men?<br />

What effect does pregnancy have on<br />

sexual functioning?<br />

During what time period prior to<br />

pregnancy should a woman<br />

cease sexual activity?<br />

Patients who can tolerate increases in<br />

heart rate from 110 to 130 bpm<br />

Erectile dysfunction<br />

1. Diabetic neuropathy<br />

2. Vascular changes<br />

Metabolic control monitored by<br />

hemoglobin A 1c level<br />

Decreased fertility<br />

Erectile dysfunction<br />

Orgasmic dysfunction<br />

Reduced testosterone levels<br />

Retrograde ejaculation into the bladder<br />

Decreased sex drive → most common.<br />

Increased sex drive and pelvic<br />

vasocongestion may occur.<br />

4 weeks before expected delivery<br />

EFFECTS OF DRUGS AND NEUROTRANSMITTERS ON SEXUALITY<br />

Decreased availability of which<br />

neurotransmitter(s) causes a decrease<br />

in erection?<br />

Increased availability of which<br />

neurotransmitter(s) causes a<br />

decrease in ejaculation and orgasm?<br />

Increased availability of which<br />

neurotransmitter(s) causes an increase<br />

in erection?<br />

Which drug(s) of abuse causes<br />

increased libido with acute use?<br />

Which drug(s) of abuse causes<br />

increased libido?<br />

Dopamine (eg, chlorpromazine,<br />

haloperidol)<br />

Norepinephrine β (eg, propranolol,<br />

metoprolol)<br />

Serotonin (eg, fluoxetine, sertraline,<br />

trazodone)<br />

Dopamine (eg, levodopa)<br />

Norepinephrine in α 2 in the periphery<br />

(eg, yohimbine)<br />

Alcohol<br />

Marijuana<br />

Amphetamines<br />

Cocaine


56 Deja Review: <strong>Behavioral</strong> <strong>Science</strong><br />

Which drug(s) of abuse causes erectile<br />

dysfunction due to increased estrogen<br />

availability as a result of liver damage?<br />

Which drugs(s) of abuse causes reduced<br />

testosterone levels in men and lowered<br />

pituitary gonadotropin levels in women?<br />

Which drug(s) of abuse causes reduced<br />

libido and inhibited ejaculation?<br />

Alcohol → with chronic use<br />

Marijuana → with chronic use<br />

Heroin<br />

Methadone<br />

CLINICAL VIGNETTES<br />

A news program reports on a man who is causing problems in the local subway.<br />

As the doors open he rubs up against unsuspecting victims and then runs away.<br />

What is the name for this disorder?<br />

Frotteurism<br />

A 32-year-old male who is happily married does not become sexually aroused<br />

unless his wife wears a specific black negligee. He has always had an obsession<br />

with objects to give him sexual gratification. Which sexual paraphilia does this<br />

patient have?<br />

Fetishism<br />

A 25-year-old female experiences intense painful vaginal spasms whenever she<br />

goes to get a pelvic examination and engages in sexual intercourse. She has begun<br />

psychological counseling to treat her condition. Which type of sexual dysfunction<br />

does she suffer from?<br />

Vaginismus


CHAPTER 8<br />

Abuse and Aggression<br />

CHILD AND ELDER ABUSE AND NEGLECT<br />

What are the primary types of child<br />

and elder abuse?<br />

What are the primary traits of a<br />

child abuser?<br />

What traits in a child make them more<br />

likely to be abused?<br />

What is the most common age range<br />

of children that are abused?<br />

What are key signs of child neglect?<br />

What are common sites of bruises on<br />

a victim of child abuse?<br />

What type of marks might one see on a<br />

victim of child abuse?<br />

What are characteristics of the fractures<br />

on a victim of child abuse?<br />

Physical<br />

Sexual<br />

Emotional<br />

Low socioeconomic status<br />

Young parents<br />

Substance abuse<br />

Social isolation<br />

Stress in the household<br />

History of victimization by spouse or<br />

caretaker<br />

Child that has already been abused<br />

Low birth weight, premature infant<br />

Learning disability or language disorder<br />

Adopted child or stepchildren<br />

Hyperactive<br />


58 Deja Review: <strong>Behavioral</strong> <strong>Science</strong><br />

What are the two primary burn types<br />

on victims of child abuse?<br />

What are other warning signs for child<br />

abuse/non-accidental trauma?<br />

What is the classic physical finding<br />

of shaken baby syndrome?<br />

What type of cultural healing practices<br />

may leave markings that are important<br />

to distinguish from child abuse?<br />

What are the primary traits of an<br />

elder abuser?<br />

What are the primary traits of an elder<br />

who experiences elder abuse?<br />

What are key signs of elder neglect?<br />

What are characteristics of the bruises<br />

seen on a victim of elder abuse?<br />

What is the annual incidence of<br />

child sexual abuse cases in the<br />

United States?<br />

Has the likelihood of reporting child<br />

sexual abuse increased or decreased<br />

when compared with the past?<br />

1. Cigarette burns<br />

2. Immersion burns on legs, feet, or<br />

buttocks → child is immersed in<br />

scalding water<br />

History inconsistent with level of child’s<br />

motor development, bite marks,<br />

and any facial trauma or bruising<br />

Retinal hemorrhage<br />

Cupping (use of heated cups to suction<br />

out illness)<br />

Coining or Spooning (rubbed on body<br />

leaving linear marks)<br />

Low socioeconomic status<br />

Social isolation<br />

Substance abuse<br />

Close relationship to the abused (eg,<br />

spouse, offspring) → person with whom<br />

the elder lives and receives financial<br />

support from<br />

Some decline of mental functioning<br />

(eg, dementia)<br />

Economical or physical dependence on<br />

others<br />

Not likely to report injuries as abuse →<br />

will state that they injured themselves<br />

Lack of proper nutrition<br />

Poor personal hygiene<br />

Lack of proper medication or health<br />

aids (eg, prescription drugs, dentures,<br />

cane)<br />

Bilateral arm bruises from being<br />

grabbed<br />

More than 250,000 cases<br />

Increased


Abuse and Aggression 59<br />

Which gender is more likely to report<br />

sexual abuse during their lifetime?<br />

Which gender is most likely to<br />

be the perpetrator of child<br />

sexual abuse?<br />

Will the perpetrator of a child sexual<br />

abuse more likely be a stranger or<br />

someone the child knows?<br />

What are some traits of child sexual<br />

abusers?<br />

What is the primary age range of<br />

children who are victims of<br />

sexual abuse?<br />

What emotions is the victim of a child<br />

sexual abuse likely to experience?<br />

What are the common physical signs<br />

of child sexual abuse?<br />

What are common psychological signs<br />

of child sexual abuse?<br />

What are common physical signs of<br />

elder sexual abuse?<br />

What types of emotional abuse do<br />

children experience?<br />

Girls → 25% will report vs 12% boys<br />

Males<br />

Relative or acquaintance (eg, father,<br />

uncle, friend of the family, etc)<br />

Interpersonal relationship problems<br />

(eg, marriage problems)<br />

History of substance abuse<br />

May have a history of pedophilia<br />

9 to 12 years of age<br />

Guilt<br />

Shame<br />

Fear of abuser’s response if he or she<br />

notified someone else of his or her<br />

experience<br />

Sexually transmitted infection<br />

(eg, human papillomavirus [HPV],<br />

herpes, chlamydia)<br />

Recurrent urinary tract infection<br />

(UTI)<br />

Genital or anal injury<br />

Note: Physical signs may be absent in a<br />

victim of child sexual abuse.<br />

Inappropriate knowledge about sexual<br />

events out of proportion for a given age<br />

range<br />

Excessive initiation of sexual activity<br />

with peers<br />

Genital bruising<br />

Vaginal bleeding in women<br />

Lack of caregiver attention and love<br />

Physical neglect<br />

Caregiver rejection


60 Deja Review: <strong>Behavioral</strong> <strong>Science</strong><br />

What types of emotional abuse do<br />

elders experience?<br />

How many cases of child and elder<br />

abuse are reported annually?<br />

What is the physician’s responsibility<br />

if they suspects child or elder abuse?<br />

Neglect of needed care (eg, medical,<br />

hygiene, etc)<br />

Economic exploitation<br />

1,000,000 cases each<br />

By law, the physician must report the<br />

case to appropriate social service<br />

agency.<br />

DOMESTIC PARTNER ABUSE<br />

What are primary findings seen<br />

in women who are victims of<br />

domestic violence?<br />

What factor can greatly increase the<br />

likelihood of an abused person being<br />

killed by their abuser?<br />

What is the primary gender of the<br />

perpetrator of domestic violence?<br />

What are the characteristics of the<br />

abuser in a domestic violence situation?<br />

What are the characteristics of the<br />

abused in a domestic violence situation?<br />

What characteristic is common to both<br />

the abused and the abuser in a<br />

domestic violence situation?<br />

What is the role of the physician, in<br />

terms of reporting, if notified of<br />

domestic violence abuse?<br />

Broken bones<br />

Bruises<br />

Blackened eyes<br />

If the abused person leaves the abuser.<br />

Male<br />

Substance abuse<br />

Angry<br />

Threatens to kill the abused<br />

Apologetic after abuse has occurred<br />

May be pregnant<br />

May not report abuse to police<br />

May not leave the abuser<br />

Blame themselves for the abuse<br />

Emotional and financial dependence on<br />

the abuser<br />

Low self-esteem<br />

The physician does not have mandatory<br />

reporting since the abused is generally a<br />

competent adult. The physician can<br />

provide emotional support and<br />

encourage the abused to report the<br />

violence.


Abuse and Aggression 61<br />

SEXUAL VIOLENCE<br />

What is the definition of sexual assault?<br />

What are examples of force that can be<br />

used to commit a sexual assault?<br />

What is the definition of consent?<br />

What term describes oral and anal<br />

penetration?<br />

What is the primary age group<br />

of a rapist?<br />

What is the racial background of a<br />

perpetrator of sexual assault?<br />

Which substance is most frequently<br />

used in cases of sexual assault?<br />

What percentage of rapes are<br />

acquaintance rapes (ie, the victim<br />

knows the perpetrator)?<br />

What percentage of rapes are reported<br />

to the police?<br />

What is the age group most likely to<br />

experience a sexual assault?<br />

What is the most common place<br />

for a sexual assault to occur?<br />

A person commits a sexual assault when<br />

he or she uses force or the threat of force<br />

to touch another person sexually in a<br />

way that person does not want or when<br />

that person cannot give consent because<br />

of physical or mental inability.<br />

Note: Sexual assault is the legal term for<br />

rape.<br />

Manipulation<br />

Coercion<br />

Physical force<br />

Use of weapons<br />

Use of isolation<br />

Use of substances—alcohol and other<br />

drugs<br />

Giving permission by giving a “yes”<br />

response<br />

Sodomy<br />


62 Deja Review: <strong>Behavioral</strong> <strong>Science</strong><br />

Which disorder do/may sexual assault<br />

survivors experience?<br />

Which type of treatment is the best<br />

option for survivors of sexual assault?<br />

Is it possible for spouses to be<br />

convicted of committing a sexual<br />

assault against each other?<br />

What terms describes consensual sex<br />

that is considered rape?<br />

With which group(s) of people would<br />

consensual sex be considered<br />

statutory rape?<br />

What is the role of the physician<br />

if they suspect a sexual assault?<br />

Posttraumatic stress disorder<br />

Group therapy<br />

Yes<br />

Statutory rape<br />

Laws vary by state—usually consensual<br />

age is either 16 or 18, but some states<br />

have age difference limits (eg, not over<br />

3 years difference if under 18 years old)<br />

Profoundly disabled persons<br />

Laws vary by state—depending on age<br />

of the victim, this may fall under<br />

mandatory reporting for child or elder<br />

abuse; in adult cases it is the victims’<br />

choice to report.<br />

AGGRESSION<br />

What has happened since the 1990s<br />

to the incidence of homicide in the<br />

United States?<br />

Which weapon is most commonly used<br />

in a homicide?<br />

What are risk factors for being victims<br />

of homicide?<br />

Which racial group is most likely to be<br />

affected by a homicide?<br />

How does violence on television or<br />

video games influence aggression<br />

in children?<br />

Which gender is most likely to be most<br />

aggressive?<br />

Decreased<br />

Guns<br />

Male sex<br />

Low socioeconomic status<br />

Alcohol and drug users<br />


Abuse and Aggression 63<br />

Why are many body builders and some<br />

professional athletes likely to show<br />

increased aggression?<br />

Use of which drugs is associated with<br />

increased aggression?<br />

Which neurotransmitter(s) is associated<br />

with increased aggression?<br />

Which neurotransmitter(s) is associated<br />

with decreased aggression?<br />

What type of injury is most likely to be<br />

associated with violence?<br />

Use of androgenic or anabolic steroids<br />

increases aggression.<br />

Alcohol<br />

Amphetamines<br />

Cocaine<br />

Phencyclidine<br />

Marijuana (high doses)<br />

Dopamine<br />

Norepinephrine<br />

Serotonin<br />

γ-Aminobutyric acid (GABA)<br />

Head injury<br />

CLINICAL VIGNETTES<br />

A family who has newly immigrated to the United States comes to your family<br />

practice to establish care. The son in the family has been suffering from a chronic<br />

cough and the parents are worried. Upon examination you find well-demarcated,<br />

circular, purpuric lesions on his back. They don’t look accidental. What should you<br />

do?<br />

Ask the family about their cultural practices. This could represent a cultural<br />

healing technique such as “cupping” used to heal the boy’s cough. It is important<br />

to distinguish this from child abuse.<br />

A 21-year-old female college student decides to go on a date with her boyfriend of<br />

2 years. She consumes a large volume of alcohol while out. When they return to<br />

her dorm room, the boyfriend tries to coerce her into having sex with him. She<br />

passes out without agreeing. Since they’ve had had sex before, he cites their longterm<br />

relationship as proof of consent. Would this incident be considered sexual<br />

assault?<br />

Yes—a long-term relationship (or even marriage) does not substitute or replace<br />

consent. Even if the woman agreed to intercourse, an intoxicated person is unable<br />

to give consent and the act would still be considered sexual assault.


This page intentionally left blank


CHAPTER 9<br />

Suicide<br />

SUICIDE<br />

What is important to keep in mind<br />

when approaching the topic of suicide<br />

on the United States Medical Licensing<br />

Exam (USMLE) Step 1?<br />

Where does suicide rank as a cause of<br />

death in the United States?<br />

What are risk factors for committing<br />

suicide?<br />

Who commits suicide more frequently,<br />

men or women?<br />

There is only a limited amount of<br />

information that you will be asked<br />

regarding suicide. Always think safety<br />

first when given a question regarding a<br />

psychiatric disorder. Statistics,<br />

demographics, and risk factors, and<br />

comorbid medical disorders will<br />

probably be the focus of the exam<br />

content. Many of the questions will<br />

come in the form of a case scenario<br />

written to ascertain if you know the<br />

correlation between suicide and<br />

comorbid mental health diagnoses such<br />

as depression, bipolar disorder, and/or<br />

chemical dependence.<br />

As of 2002, suicide ranks 10th as the<br />

leading cause of death.<br />

White male, >65 years old<br />

Gun in the house or easy access to<br />

firearms<br />

Comorbid depression, substance abuse,<br />

and/or other psychiatric illness<br />

Recent loss or stressor<br />

Serious medical illness<br />

Feelings of hopelessness and<br />

impulsivity<br />

Men are four times more likely to<br />

complete suicide, though women are<br />

more likely to attempt suicide.<br />

65


66 Deja Review: <strong>Behavioral</strong> <strong>Science</strong><br />

What accounts for this difference?<br />

What age group has the highest<br />

suicide rate?<br />

Men tend to use more lethal means<br />

(firearms, hanging) and women less<br />

lethal (pill overdose).<br />

• The elderly (65 and older). Despite<br />

being only 14% of the population,<br />

they comprise about 18% to 25%<br />

of the total suicide percentage.<br />

• The elderly suicide rate is 40 per<br />

100,000 persons in comparison to the<br />

national US rate of 12 per 100,000.<br />

What has happened to the suicide rate It has risen significantly over the last<br />

in child and adolescent groups over 40 years with a mortality rate of 12%,<br />

the last 40 years? though declined since 1990.<br />

What is the most significant risk factor<br />

of suicide?<br />

What are other risk factors for suicide<br />

(in decreasing order)?<br />

How is suicide most commonly<br />

committed or attempted?<br />

Is there racial disparity among those<br />

who commit suicide?<br />

What are the most common comorbid<br />

medical disorders associated<br />

with suicide?<br />

A previous attempt.<br />

You should become extremely cautious<br />

if the attempt is recent (eg, within<br />

3 months) and if the nature of the<br />

attempt is well-thought out and<br />

deliberate.<br />

1. Persons >45 years<br />

2. Alcohol dependence<br />

3. History of violence or aggression<br />

4. Male gender<br />

Firearms are the number one method<br />

for completed suicide, regardless of<br />

gender.<br />

Other common methods include<br />

jumping, hanging, overdose,<br />

and drowning.<br />

Yes, whites commit suicide more than<br />

any other group. However, the suicide<br />

rate among young black males is slowly<br />

increasing.<br />

Major Depressive Disorder<br />

Anxiety Disorders<br />

Posttraumatic Stress Disorder (PTSD)<br />

Schizophrenia<br />

Bipolar Disorder<br />

Personality Disorders<br />

Substance Abuse


Suicide 67<br />

At what stage, in treated major<br />

depression, is suicide attempted?<br />

Are there special concerns about<br />

children and adolescents treated<br />

for depression?<br />

When do you assess for suicidality?<br />

What is the difference between active<br />

and passive suicidal ideation?<br />

Can suicide attempts be predicted?<br />

How can suicidal ideation be managed?<br />

What antidepressant would be the first<br />

choice in a suicidal patient?<br />

For adults, suicide is most attempted<br />

after pharmacologic treatment has<br />

begun. The patient has more energy and<br />

is better able to function but low mood<br />

and hopelessness may persist.<br />

There is FDA black box<br />

warning for all antidepressants that<br />

use in children and adults younger than<br />

25 may increase suicidal thoughts and<br />

behaviors. When starting such a<br />

medication in this population, frequent<br />

and close monitoring for suicidality is<br />

important.<br />

At every follow-up assess for:<br />

• Ideation<br />

• Plan (possibility and practicality)<br />

• Intent to carry out the plan<br />

• Access to firearms<br />

Passive: Patient would like to be dead<br />

(eg, “I wish I were dead!”)<br />

Active: Patient actually wants to harm<br />

him/herself (eg, “I am going to shoot<br />

myself with a gun.”)<br />

There is no reliable means to predict<br />

suicide attempts with a reasonable<br />

degree of specificity and sensitivity.<br />

If suicide risk is imminent, the patient<br />

must be detained and the appropriate<br />

mental health care sought.<br />

If suicide risk is not imminent,<br />

assessment and defusing risks is<br />

important (eg, removing firearms from<br />

house, involving family members<br />

if possible).<br />

Antidepressant medication should be<br />

started and the patient should be<br />

followed closely and seen frequently.<br />

Aggressive SSRI (selective serotonin<br />

reuptake inhibitor) therapy is the best<br />

choice—avoid monoamine oxidase<br />

(MAO) inhibitors and tricyclics in a<br />

patient with active suicidal ideation as<br />

they can be lethal in an overdose.


68 Deja Review: <strong>Behavioral</strong> <strong>Science</strong><br />

What are protective factors from<br />

suicide?<br />

• Marriage: Single persons that are<br />

divorced, have never married, or<br />

widowed have higher suicide rates<br />

than married persons. Living alone<br />

and limited social and family support<br />

are also risk factors.<br />

• Employment: Persons who are<br />

unemployed have higher suicide<br />

rates. However, among employed US<br />

citizens, those working in professional<br />

roles (doctors, lawyers, and in law<br />

enforcement) have higher suicide<br />

rates than nonprofessional persons.<br />

• No family history of a completed<br />

suicide or an attempted suicide is<br />

protective because once suicide<br />

occurs, it seemingly decreases the<br />

social “taboo” of suicide in the family.<br />

CLINICAL VIGNETTES<br />

A patient in your outpatient clinic presents with suicidal ideation, but no current<br />

intent to harm himself. You would like to decrease the probability of him<br />

committing suicide and so prescribe him a high-dose SSRI. Will this eliminate his<br />

suicide risk?<br />

Though aggressive treatment is indicated and untreated depression carries a high<br />

risk of suicide, treatment with SSRI alone will not necessarily eliminate risk of<br />

suicide. Screening for associated risk factors such as access to firearms and other<br />

lethal means is also indicated.


CHAPTER 10<br />

Genetic Basis<br />

of Behavior<br />

GENETIC STUDIES<br />

What type of study uses a family tree<br />

to show the occurrence of traits and<br />

diseases throughout generations?<br />

What type of study compares the<br />

frequency of disease in a proband<br />

(affected individual) with its frequency<br />

in the general population?<br />

What type of study compares<br />

monozygotic and dizygotic twins<br />

to determine the effects of genetic<br />

factors from environmental factors<br />

of disease?<br />

This term describes if both twins have<br />

a given trait.<br />

What type of twins is more likely<br />

to have a higher likelihood of having<br />

a disease that is genetic in origin?<br />

Pedigree study<br />

Family risk study<br />

Adoption study<br />

Concordance<br />

Monozygotic twins<br />

PSYCHIATRIC DISORDERS GENETICS<br />

What is the prevalence of schizophrenia 1%<br />

in the general population?<br />

In which gender is schizophrenia more<br />

likely?<br />

Which persons have a higher likelihood<br />

of developing schizophrenia?<br />

Equal in males and females<br />

Persons with a close genetic relationship<br />

69


70 Deja Review: <strong>Behavioral</strong> <strong>Science</strong><br />

Genetic markers on which 1, 6, 8, and 13<br />

chromosome(s) are associated with<br />

schizophrenia?<br />

Does schizophrenia have a familial<br />

component?<br />

Yes—the closer a relative you are, the<br />

more likely you are to develop<br />

schizophrenia; but concordance in twins<br />

is not 100%.<br />

What is the likelihood of developing 10%<br />

schizophrenia if you are a first-degree<br />

relative of an affected individual?<br />

What is the likelihood that a child 40%<br />

will develop schizophrenia if both<br />

parents have the disorder?<br />

What is the likelihood that the 50%<br />

monozygotic twin of a patient with<br />

schizophrenia will develop the<br />

disorder?<br />

What is the likelihood of developing 25%<br />

any mood disorder if you are a firstdegree<br />

relative of a person with bipolar<br />

disorder?<br />

What is the likelihood of developing 60%<br />

any mood disorder if someone is a child<br />

with both parents who have bipolar<br />

disorder?<br />

What is the likelihood of developing 80% to 90%<br />

bipolar disorder if you are the<br />

monozygotic twin of a person with<br />

the disorder?<br />

Is the genetic component stronger for<br />

schizophrenia or bipolar disorder?<br />

In which gender is the lifetime<br />

prevalence of a major depressive<br />

disorder higher?<br />

Bipolar disorder<br />

Females<br />

What is the percentage of males who 10%<br />

will develop a major depressive<br />

disorder in their lifetime?<br />

What is the percentage of females 15% to 20%<br />

who will develop a major depressive<br />

disorder in their lifetime?


Genetic Basis of Behavior 71<br />

Do personality disorders have a higher<br />

concordance in monozygotic twins<br />

demonstrating that they have a genetic<br />

component?<br />

If a person has antisocial personality<br />

disorder, what psychiatric condition(s)<br />

will be prevalent in relatives?<br />

If a person has avoidant personality<br />

disorder, what psychiatric condition(s)<br />

will be prevalent in relatives?<br />

If a person has borderline personality<br />

disorder, what psychiatric condition(s)<br />

will be prevalent in relatives?<br />

If a person has histrionic personality<br />

disorder, what psychiatric condition(s)<br />

will be prevalent in relatives?<br />

If a person has schizotypal personality<br />

disorder, what psychiatric condition(s)<br />

will be prevalent in relatives?<br />

Yes<br />

Alcoholism<br />

Attention-deficit hyperactivity disorder<br />

(ADHD)<br />

Anxiety disorder<br />

Major depressive disorder<br />

Substance abuse<br />

Somatization disorder<br />

Schizophrenia<br />

NEUROPSYCHIATRIC DISORDER GENETICS<br />

In which disease is there a diminution<br />

of cognitive functioning and a<br />

likelihood of genetic influence?<br />

Alzheimer disease<br />

Which chromosome has three copies in Chromosome 21<br />

Down syndrome, and is also implicated Note: Down syndrome patients often<br />

in some cases of Alzheimer disease? develop early-onset Alzheimer.<br />

What other chromosome(s) have been Chromosomes 1 and 14<br />

identified to be associated with<br />

Alzheimer disease?<br />

Which gene has been implicated in The gene encoding Apolipoprotein<br />

many cases of Alzheimer disease? E4 (Apo E4)<br />

In which chromosome is the Apo E4 Chromosome 19<br />

gene located?<br />

What disease has an abnormal gene<br />

on the short end of chromosome 4?<br />

What is the most common genetic cause<br />

of mental retardation?<br />

Huntington disease<br />

Down syndrome


72 Deja Review: <strong>Behavioral</strong> <strong>Science</strong><br />

What is the second most common<br />

genetic cause of mental retardation?<br />

What disorder, characterized by verbal<br />

and motor tics, has a genetic<br />

component?<br />

Fragile X syndrome<br />

Tourette disorder<br />

ALCOHOLISM GENETICS<br />

What is the prevalence of alcoholism<br />

in children of alcoholics compared<br />

to the general population?<br />

Four times more prevalent<br />

What is the concordance rate of 60%<br />

alcoholism in monozygotic twins?<br />

What is the concordance rate of 30%<br />

alcoholism in dizygotic twins?<br />

Which gender offspring of alcoholics<br />

is more likely to become alcoholics<br />

themselves?<br />

In which age group is the genetic<br />

influence of alcoholism strongest<br />

in males?<br />

Male offspring<br />


Genetic Basis of Behavior 73<br />

A 19-year-old male is brought to the emergency room by his college roommate for<br />

delusions, hallucinations, and disorganized speech that has occurred for the last<br />

7 months. The roommate is concerned that the patient may have schizophrenia. The<br />

patient’s father was diagnosed with schizophrenia at age 22. Not taking into account<br />

these new symptoms, what is the lifetime likelihood of this patient developing<br />

schizophrenia?<br />

The patient is a first-degree relative of an affected individual—he therefore has a<br />

10% chance of developing schizophrenia.


This page intentionally left blank


CHAPTER 11<br />

Neurochemistry in<br />

<strong>Behavioral</strong> <strong>Science</strong>s<br />

NEUROANATOMY<br />

What are the two divisions of the<br />

nervous system?<br />

What are the two main components<br />

of the CNS?<br />

Which brain structures connect<br />

the cerebral hemispheres?<br />

Which hemisphere is usually the<br />

dominant hemisphere?<br />

What is the primary role of the left<br />

hemisphere?<br />

Which hemisphere is usually the<br />

nondominant hemisphere?<br />

What is the primary role of the right<br />

hemisphere?<br />

What are the components of the<br />

peripheral nervous system?<br />

How many cranial nerves are there?<br />

How many spinal nerves are there?<br />

1. Central nervous system (CNS)<br />

2. Peripheral nervous system (PNS)<br />

1. Brain<br />

2. Spinal cord<br />

Corpus callosum<br />

Commissures: anterior, posterior,<br />

hippocampal, and habenular<br />

Left hemisphere<br />

It governs our ability to express<br />

ourselves in language.<br />

Right hemisphere<br />

It governs perceptual functions and the<br />

analysis of space, geometrical shapes,<br />

and forms.<br />

Nerve fibers outside the CNS including<br />

cranial nerves, spinal nerves, and<br />

peripheral ganglia<br />

12 cranial nerves<br />

31 pairs of spinal nerves: 8 cervical,<br />

12 thoracic, 5 lumbar, 5 sacral, and<br />

1 coccygeal<br />

75


76 Deja Review: <strong>Behavioral</strong> <strong>Science</strong><br />

In which direction does the PNS carry<br />

motor and sensory information<br />

to the CNS?<br />

What are the components of the<br />

autonomic nervous system?<br />

Motor information: away from the CNS<br />

Sensory information: to the CNS<br />

Sensory neurons and motor neurons<br />

that run between the CNS (especially<br />

the hypothalamus and medulla oblongata)<br />

and various internal organs.<br />

BRAIN LESIONS<br />

What will be the neuropsychiatric<br />

consequences of a frontal lobe lesion?<br />

What will be the neuropsychiatric<br />

consequences of a parietal lobe lesion?<br />

What will be the neuropsychiatric<br />

consequences of a temporal lobe lesion?<br />

What will be the neuropsychiatric<br />

consequences of a hippocampus lesion?<br />

What will be the neuropsychiatric<br />

consequences of an amygdala lesion?<br />

What will be the neuropsychiatric<br />

consequences of a reticular system<br />

lesion?<br />

What will be the neuropsychiatric<br />

consequences of a basal ganglia lesion?<br />

What will be the neuropsychiatric<br />

consequences of a hypothalamus<br />

lesion of the ventromedial nucleus?<br />

Deficits in concentration, judgment,<br />

motivation, and orientation<br />

Disinhibition<br />

Personality and emotional changes<br />

Right parietal lobe → (contralateral<br />

neglect) result in neglecting part of the<br />

body or space<br />

Left parietal lobe → verbal deficits<br />

Hallucinations<br />

Memory deficits<br />

Personality changes<br />

Bilateral damage to hippocampus leads<br />

to massive anterograde and some<br />

retrograde amnesia.<br />

Unilateral damage of hippocampus<br />

leads to memory storage and retrieval<br />

problems.<br />

Kluver-Bucy syndrome → uninhibited<br />

behavior, hyperorality, hypersexuality<br />

Sleep-wake cycle changes<br />

Tremor or other involuntary movements<br />

as seen in Parkinson or Huntington<br />

diseases<br />

Decreased satiety → leads to obesity


Neurochemistry in <strong>Behavioral</strong> <strong>Science</strong>s 77<br />

What will be the neuropsychiatric<br />

consequences of a hypothalamus lesion<br />

of the lateral nucleus?<br />

What will be the neuropsychiatric<br />

consequences of a hypothalamus lesion<br />

of the anterior hypothalamus?<br />

What will be the neuropsychiatric<br />

consequences of a hypothalamus lesion<br />

of the posterior hypothalamus?<br />

What will be the neuropsychiatric<br />

consequences of a hypothalamus lesion<br />

of the septate nucleus?<br />

What will be the neuropsychiatric<br />

consequences of a hypothalamus lesion<br />

of the suprachiasmatic lesion?<br />

Decreased hunger → leads to weight loss<br />

Disturbances of parasympathetic<br />

activity<br />

Disturbances of body cooling<br />

Disturbances of heat conservation<br />

Change in sexual urges and emotions<br />

Disturbances of circadian rhythm<br />

NEUROTRANSMITTERS<br />

What are the four main steps involved<br />

in neurotransmitter release?<br />

What are the two different types of<br />

neurotransmitters?<br />

What are the two main excitatory and<br />

inhibitory neurotransmitters<br />

in the CNS?<br />

Which factors contribute to the<br />

magnitude of reaction<br />

neurotransmitters<br />

have on neurons?<br />

What are the three major classes of<br />

neurotransmitters?<br />

How are neurotransmitters removed<br />

from the synaptic cleft?<br />

1. Presynaptic neuron stimulation.<br />

2. Neurotransmitter release.<br />

3. Neurotransmitter moves across<br />

synaptic cleft.<br />

4. Neurotransmitter acts on<br />

postsynaptic neuron receptors.<br />

1. Excitatory: increase neuron firing<br />

2. Inhibitory: decrease neuron firing<br />

1. Excitatory: glutamate<br />

2. Inhibitory: γ-aminobutyric acid<br />

(GABA)<br />

1. Affinity of receptors<br />

2. Number of receptors<br />

1. Amino acids<br />

2. Biogenic amines<br />

3. Peptides<br />

Reuptake by the presynaptic neuron<br />

Degradation by enzymes (eg, monoamine<br />

oxidase)


78 Deja Review: <strong>Behavioral</strong> <strong>Science</strong><br />

Why is this important in pharmacology?<br />

These are common targets for<br />

psychotropic drug actions (eg, SSRIs,<br />

MAO inhibitors)<br />

Table 11.1 Neurotransmitter Alterations in Psychiatric Conditions<br />

Acetylcholine<br />

Alzheimer<br />

Disease Anxiety Depression Mania Schizophrenia<br />

↓<br />

Dopamine ↓ ↑ ↑<br />

GABA<br />

Norepinephrine ↑ ↓<br />

↓<br />

Serotonin ↓ ↓ ↑<br />

AMINES<br />

Which amines are included in the<br />

biogenic amines, which are also called<br />

the monoamines?<br />

What is the monoamine theory<br />

of depression?<br />

Why are metabolites of monoamines<br />

measured in psychiatric studies?<br />

What type of biogenic amine<br />

is dopamine?<br />

In which psychiatric condition(s) is an<br />

altered level of dopamine evident?<br />

How is dopamine synthesized?<br />

Catecholamines<br />

Ethylamines<br />

Indolamines<br />

Quaternary amines<br />

It proposes that there is an underlying<br />

neuroanatomical basis for depression<br />

due to deficiencies of central<br />

noradrenergic and/or serotonergic<br />

systems.<br />

They may be present in higher levels<br />

than the primary monoamines.<br />

Catecholamine<br />

Mood disorders<br />

Parkinson disease<br />

Schizophrenia<br />

By the conversion of tyrosine to<br />

dopamine by tyrosine hydroxylase<br />

tyrosine hydroxylase<br />

tyrosine ⎯⎯⎯⎯⎯⎯⎯⎯⎯→ dopamine


Neurochemistry in <strong>Behavioral</strong> <strong>Science</strong>s 79<br />

What is the main metabolite<br />

of dopamine?<br />

In which psychiatric condition(s) can<br />

there be an increased concentration<br />

of HVA?<br />

In which psychiatric condition(s) can<br />

there be a decreased concentration<br />

of HVA?<br />

What type of biogenic amine is<br />

norepinephrine?<br />

What behavioral factors does<br />

norepinephrine alter?<br />

How is norepinephrine synthesized?<br />

Homovanillic acid (HVA)<br />

Psychotic disorders<br />

Schizophrenia<br />

Alcoholism<br />

Depression<br />

Parkinson disease<br />

Catecholamine<br />

Anxiety<br />

Arousal<br />

Learning<br />

Memory<br />

Mood<br />

Dopamine is converted to<br />

norepinephrine by β-hydroxylase.<br />

β-hydroxylase<br />

dopamine ⎯⎯⎯⎯⎯⎯→ norepinephrine<br />

Where are most noradrenergic neurons<br />

located in the brain?<br />

What are the metabolites of<br />

norepinephrine?<br />

In which psychiatric condition(s) can<br />

there be a decreased concentration<br />

of MHPG?<br />

In which medical condition is<br />

there an increased concentration<br />

of VMA?<br />

What type of biogenic amine<br />

is serotonin?<br />

What behavioral factors does<br />

serotonin alter?<br />

Locus ceruleus<br />

3-Methoxy-4-hydroxyphenylglycol<br />

(MHPG)<br />

Vanillylmandelic acid (VMA)<br />

Severe depression<br />

Pheochromocytoma → a tumor of the<br />

adrenal medulla<br />

Indolamine<br />

Note: Another name for serotonin is<br />

5-hydroxytryptamine (5-HT).<br />

Impulse control<br />

Mood<br />

Sleep<br />

Sexuality


80 Deja Review: <strong>Behavioral</strong> <strong>Science</strong><br />

If serotonin levels are increased, which<br />

behavioral factors will be improved?<br />

If serotonin levels are increased, which<br />

behavioral factors will be impaired?<br />

If serotonin levels are decreased, which<br />

behavioral factors will be impaired?<br />

How is serotonin synthesized?<br />

Where are most serotoninergic cell<br />

bodies located in the brain?<br />

Which pharmacologic agents are used<br />

to alter the level of serotonin in<br />

the brain?<br />

What is the primary metabolite of<br />

serotonin?<br />

In which psychiatric condition(s) is<br />

there a decreased concentration<br />

of 5-HIAA?<br />

What type of biogenic amine<br />

is histamine?<br />

Which pharmacologic agents block<br />

the histamine receptor?<br />

What are side effects of the histamine<br />

receptor blockade?<br />

What type of biogenic amine<br />

is acetylcholine (ACh)?<br />

Where is acetylcholine normally<br />

found in the body?<br />

Mood<br />

Sleep<br />

Sexual functioning<br />

Impulse control<br />

Sleep<br />

Note: Patient is likely to experience<br />

depression.<br />

Tryptophan is converted to serotonin by<br />

tryptophan hydroxylase and an amino<br />

acid decarboxylase.<br />

Dorsal raphe nucleus<br />

Antidepressants—eg, selective serotonin<br />

reuptake inhibitors (SSRIs)<br />

5-Hydroxyindoleacetic acid (5-HIAA)<br />

Alcoholism<br />

Bulimia<br />

Impulsive behavior<br />

Pyromania: uncontrollable desire to set<br />

things on fire<br />

Severe depression<br />

Tourette syndrome<br />

Violent behavior<br />

Ethylamine<br />

Antipsychotic drugs<br />

Tricyclic antidepressants (TCAs)<br />

Increased appetite → contributing to<br />

weight gain and obesity<br />

Sedation<br />

Quaternary amine<br />

Neuromuscular junctions


Neurochemistry in <strong>Behavioral</strong> <strong>Science</strong>s 81<br />

Which psychiatric conditions are<br />

associated with a decrease in<br />

cholinergic neurons?<br />

How is acetylcholine synthesized?<br />

How is acetylcholine degraded?<br />

Which pharmacologic agents have been<br />

shown to reduce the degradation<br />

of acetylcholine?<br />

What are the three primary amino acid<br />

neurotransmitters?<br />

Which amino acid neurotransmitter(s)<br />

are excitatory?<br />

Which amino acid neurotransmitters(s)<br />

are inhibitory?<br />

Which pharmacologic agents alter<br />

duration and frequency of GABA?<br />

Which neurotransmitter regulates<br />

glutamate activity?<br />

Which pathologic conditions may<br />

glutamate play a role in?<br />

Alzheimer disease<br />

Down syndrome<br />

Movement disorders<br />

Acetyl coenzyme A (CoA) and choline<br />

are converted to acetylcholine by<br />

choline acetyltransferase in cholinergic<br />

neurons<br />

Acetylcholine esterase (AChE) degrades<br />

acetylcholine into acetate and choline.<br />

Donepezil<br />

Tacrine<br />

Note: These agents can slow the<br />

progression of diseases such as<br />

Alzheimer disease.<br />

1. GABA<br />

2. Glutamate<br />

3. Glycine<br />

Glutamate<br />

GABA—primary inhibitory<br />

neurotransmitter<br />

Glycine<br />

Barbiturates: alter duration of GABA<br />

channel opening<br />

Benzodiazepines: alter frequency of<br />

GABA channel opening<br />

Glycine<br />

Cell death mechanisms<br />

Epilepsy<br />

Neurodegenerative diseases<br />

Psychotic disorders (eg, schizophrenia)<br />

NEUROPEPTIDES<br />

What are the two endogenous opioids?<br />

1. Endorphins<br />

2. Enkephalins


82 Deja Review: <strong>Behavioral</strong> <strong>Science</strong><br />

What behavioral factors do endogenous<br />

opioids alter?<br />

Which factor does endogenous<br />

opioids alter in research studies?<br />

Which neuropeptide(s) has been<br />

implicated in aggression and pain?<br />

Which neuropeptides have been<br />

implicated in Alzheimer disease?<br />

Which neuropeptides have been<br />

implicated in mood disorders?<br />

Which neuropeptides have been<br />

implicated in schizophrenia?<br />

Anxiety<br />

Mood<br />

Pain<br />

Seizure activity<br />

Temperature regulation<br />

Placebo effects → endogenous opioids<br />

are thought to play a major role in the<br />

placebo effects seen in research studies.<br />

Substance P<br />

Somatostatin<br />

Vasoactive intestinal peptide (VIP)<br />

Oxytocin<br />

Somatostatin<br />

Substance P<br />

Vasopressin<br />

VIP<br />

Cholecystokinin (CCK)<br />

Neurotensin<br />

CLINICAL VIGNETTES<br />

A man in your neighborhood has had a recent accident that caused brain damage.<br />

You notice his personality is much different now and he seems to have trouble<br />

with higher thought processes. He now also uses profanity more often. What part<br />

of his brain was most likely damaged?<br />

The frontal lobe


SECTION II<br />

Psychiatric<br />

Disorders and<br />

Treatment


This page intentionally left blank


CHAPTER 12<br />

Psychotic Disorders<br />

INTRODUCTION<br />

What is the most used book of criteria<br />

for the diagnosis of psychiatric<br />

disorders?<br />

What are the five axes used in the<br />

DSM-IV-TR?<br />

What is an important criterion<br />

of most DSM-IV-TR disorders?<br />

What strategy will help when learning<br />

the different types of disorders?<br />

DSM-IV-TR<br />

1. Axis I: Major psychiatric disorder<br />

2. Axis II: Personality disorders and<br />

mental retardation<br />

3. Axis III: Medical conditions<br />

4. Axis IV: Environmental factors<br />

5. Axis V: Global assessment of<br />

functioning (0-100)<br />

The symptoms cause significant<br />

social or functional impairment.<br />

This distinguishes a disorder from<br />

a normal variant.<br />

Try to focus on differences between<br />

similar disorders—they may have to<br />

do with duration, severity, or subtle<br />

features present or absent in a disorder.<br />

DISORDERS<br />

What is psychosis?<br />

What are the clinical hallmarks<br />

of psychosis?<br />

Significant impairment in reality<br />

testing (ability to distinguish real<br />

from imaginary)<br />

Delusions: fixed false beliefs, despite<br />

evidence to the contrary<br />

Hallucinations—usually auditory<br />

Disorganized speech (thought disorder)<br />

Grossly disorganized or catatonic<br />

(stupor and bizarre posturing) behavior<br />

85


86 Deja Review: <strong>Behavioral</strong> <strong>Science</strong><br />

What is the difference between<br />

a hallucination and an illusion?<br />

Name the six major primary psychotic<br />

disorders.<br />

Other than primary psychotic disorders,<br />

what other types of psychiatric illness<br />

often manifest psychotic symptoms?<br />

An illusion is the misperception of<br />

an actual sensory stimulus (eg, seeing<br />

a pool of water on the road ahead<br />

during a hot summer day),<br />

whereas hallucinations are<br />

perceptions in the absence of<br />

an external stimulus.<br />

1. Brief psychotic disorder (6 months of<br />

symptoms)<br />

4. Schizoaffective disorder<br />

5. Delusional disorder<br />

6. Shared psychotic disorder<br />

• Mood disorders: Major depression or<br />

manic episodes may include psychotic<br />

symptoms, but only during a mood<br />

disturbance.<br />

• Substance use: Acute intoxication<br />

(especially with cocaine, lysergic acid<br />

diethylamide [LSD], phencyclidine<br />

[PCP], and amphetamines) or<br />

withdrawal (especially alcohol).<br />

Anytime a patient has tactile<br />

hallucinations, you should think<br />

about drugs.<br />

• Personality disorders: May be<br />

associated with brief (not sustained)<br />

periods of psychosis.<br />

• Cognitive disorders: Both delirium<br />

and dementia demonstrate psychosis.<br />

Often delirium will have visual<br />

hallucinations in addition to clouded<br />

sensorium.<br />

• Psychosis due to a general medical<br />

conditions such as Vitamin B 12<br />

deficiency, Multiple Sclerosis (MS),<br />

Systemic Lupus Erythematosus (SLE),<br />

uremia, etc.<br />

• Narcolepsy: May have<br />

hypnagogic and hypnopompic<br />

hallucinations.


Psychotic Disorders 87<br />

Schizophrenia<br />

Which of the primary psychotic<br />

disorders is the most common?<br />

What are the DSM-IV-TR criteria<br />

for Schizophrenia?<br />

What are negative symptoms?<br />

What are considered the positive<br />

symptoms of schizophrenia?<br />

What is a “first-rank” symptom?<br />

How does the criterion of<br />

disorganized speech, (also referred<br />

to as thought disorder) manifest<br />

in schizophrenia?<br />

Schizophrenia. The incidence<br />

in the adult population is<br />

around 1%.<br />

Two or more characteristic symptoms:<br />

• Delusions<br />

• Hallucinations<br />

• Disorganized speech<br />

• Grossly disorganized behavior<br />

• Negative symptoms<br />

And social/occupational dysfunction<br />

And disturbance lasting 6+ months<br />

(including any prodrome or<br />

residual)<br />

And not due to Schizoaffective disorder<br />

or General Medical Condition or<br />

substance use<br />

Negative symptoms (think deficits) include<br />

affective flattening, alienation (social<br />

withdrawal), alogia (poverty of<br />

speech), and avolition (lack of<br />

motivation).<br />

Usually this refers to the hallucinations,<br />

delusions, bizarre behavior, and thought<br />

disorder.<br />

A particularly bizarre delusion, or<br />

hallucinations consisting of either a<br />

voice running commentary on the<br />

patient’s activities or two voices<br />

conversing. A first-rank symptom<br />

may satisfy both symptom<br />

requirements of the DSM-IV-TR<br />

criteria above.<br />

Abnormalities in thought processes and<br />

thought formation


88 Deja Review: <strong>Behavioral</strong> <strong>Science</strong><br />

Give at least two examples of abnormal<br />

thought formation.<br />

• Word salad: Words and phrases are<br />

combined together in incoherent<br />

manner.<br />

• Neologisms: Creation of new words.<br />

• Echolalia: Repeating the same word<br />

over and over (almost like a mental<br />

stutter).<br />

• Loose associations: Illogical shifting<br />

between unrelated or obliquely<br />

related topics.<br />

• Thought blocking.<br />

• Circumstantial and tangential<br />

thought: Circumstantial thought<br />

eventually gets to the point;<br />

tangential never does.<br />

X<br />

X Y<br />

W<br />

A B A B A B<br />

Normal Circumstantial Tangential<br />

Y<br />

Z<br />

What is meant by the prodromal and<br />

residual phases of schizophrenia?<br />

How long must you have symptoms for<br />

before schizophrenia can be diagnosed?<br />

How does schizophreniform disorder<br />

differ from schizophrenia?<br />

These are periods of time before and<br />

after active psychotic periods,<br />

respectively. These periods are<br />

generally characterized by attenuated<br />

symptoms of the active phase, eg, social<br />

withdrawal, peculiar behavior, or odd<br />

affect.<br />

During these periods a person would<br />

seem strange but would not necessarily<br />

meet criteria for psychotic.<br />

At least 6 months including prodromal<br />

and residual phases, but they must have<br />

at least 1 month of active characteristic<br />

symptoms (above), as well as decline in<br />

function.<br />

The duration is


Psychotic Disorders 89<br />

Does gender affect the development<br />

of schizophrenia?<br />

What other epidemiological factors<br />

are correlated with increased risk<br />

of schizophrenia?<br />

What is downward drift?<br />

What is the dopamine hypothesis?<br />

Are there any structural brain changes<br />

associated with schizophrenia?<br />

What are the five major subtypes<br />

of schizophrenia?<br />

Which form tends to have the best<br />

social functioning?<br />

Which form is the least common?<br />

There is no difference in the prevalence<br />

of schizophrenia between men and<br />

women; however, the age of onset is<br />

affected. Men tend to develop it<br />

between 15 and 25 years of age and<br />

women between 25 and 35 years of age.<br />

Having a first degree relative with<br />

schizophrenia increases a patients’ risk<br />

of schizophrenia tenfold. Being born in<br />

the cold winter months or in an area of<br />

high population density have also been<br />

associated with increased risk (though<br />

less so).<br />

This is the tendency of schizophrenics<br />

to be of lower socioeconomic status.<br />

It is generally thought that this is due<br />

to inability to function well in society,<br />

causing a “drift” into lower<br />

socioeconomic classes.<br />

This is the classic understanding of<br />

schizophrenia, which attributes the<br />

symptoms of schizophrenia to<br />

hyperactivity of the dopaminergic<br />

system. Many other theories have<br />

been postulated, however, especially<br />

involvement of Glutamate.<br />

Increase in size in the lateral and<br />

third ventricles, generalized atrophy<br />

of the cortex, and frontal lobe<br />

abnormalities are often associated<br />

with schizophrenia.<br />

1. Paranoid<br />

2. Residual<br />

3. Catatonic<br />

4. Disorganized<br />

5. Undifferentiated<br />

Paranoid. They also tend to be slightly<br />

older at onset, and have prominent<br />

hallucinations and delusions, with a<br />

lesser component of disorganization.<br />

Catatonic. Prior to the development<br />

of antipsychotics, this form was<br />

more common.


90 Deja Review: <strong>Behavioral</strong> <strong>Science</strong><br />

What is catalepsy and waxy flexibility?<br />

What is the most predictive of overall<br />

prognosis in schizophrenia?<br />

Overall, schizophrenia is associated<br />

with repeated psychotic episodes, and<br />

a chronic downhill course. What<br />

features are associated with a somewhat<br />

better prognosis?<br />

Are patients with schizophrenia at risk<br />

of suicide?<br />

What is the primary treatment for<br />

schizophrenia?<br />

What is the difference between typical<br />

and atypical antipsychotics?<br />

What is EPS?<br />

How do you treat EPS?<br />

What is tardive dyskinesia?<br />

How do you treat tardive dyskinesia?<br />

Catalepsy is the ability of a catatonic<br />

patient to hold a seemingly uncomfortable<br />

position for extended periods of time.<br />

Waxy flexibility is the slight resistance<br />

given to moving the limbs, after which<br />

the patient will often hold the new<br />

position given.<br />

Level of premorbid function<br />

Abrupt onset, female gender, presence<br />

of mood symptoms, and old age at onset<br />

Yes! Over half of schizophrenics<br />

will attempt suicide at some point in<br />

their lives, and 10% will die from it.<br />

Antipsychotics.<br />

Typical antipsychotics are older and<br />

tend to work by antagonizing dopamine.<br />

Comparatively they have higher rates<br />

of extrapyramidal symptoms (EPS) and<br />

tardive dyskinesia (TD).<br />

Atypical antipsychotics are newer, have<br />

complex mechanisms, have less EPS<br />

and TD, are more expensive, and have<br />

more metabolic side effects.<br />

This includes tremor, rigidity, akathisia<br />

(inner restlessness), and acute dystonias<br />

(muscle spasm).<br />

Anticholinergics (eg, diphenhydramine<br />

and benztropine)<br />

Abnormal movements of the face,<br />

trunk, extremities, and mouth that<br />

may happen after prolonged exposure<br />

to antipsychotic medications.<br />

Early identification and removal<br />

of the offending antipsychotic drug.<br />

With early recognition, remission rates<br />

are reasonably high. Benzodiazepines<br />

can be used for mild persistent cases.


Psychotic Disorders 91<br />

What are some differences between<br />

EPS and TD to remember?<br />

What is a potentially fatal side effect<br />

of antipsychotic treatment?<br />

How does NMS present?<br />

What is the first thing you should do<br />

if a patient presents to the ER<br />

with NMS?<br />

EPS can develop quite quickly, whereas<br />

TD tends to come on after prolonged<br />

exposure to medicines—months to years.<br />

TD is not helped by anticholinergics<br />

and may become worse. TD is also<br />

more often permanent.<br />

Neuroleptic malignant syndrome<br />

(NMS). This is an idiosyncratic<br />

reaction that is more common in<br />

young men, usually after recently<br />

starting a new antipsychotic.<br />

Mortality is nearly 20%.<br />

Fever, muscle rigidity, altered mental<br />

status, and autonomic instability<br />

Stop the antipsychotic! Then care is<br />

primarily supportive.<br />

Other Psychotic Disorders<br />

What is the DSM-IV-TR criteria<br />

for Schizoaffective Disorder?<br />

What are the two types of<br />

schizoaffective disorder?<br />

How could you differentiate<br />

between a mood disorder with<br />

psychotic symptoms (eg, major<br />

depressive disorder [MDD] with<br />

psychosis or Bipolar Mania) from<br />

schizoaffective disorder?<br />

Must meet criteria for either a major<br />

depressive episode, a manic episode,<br />

or a mixed episode at the same time<br />

as meeting characteristic symptoms of<br />

schizophrenia. The schizophrenic<br />

symptoms must persist in the<br />

absence of mood symptoms.<br />

1. Bipolar<br />

2. Depressed<br />

A person that has a mood disorder with<br />

psychotic symptoms will not have<br />

psychosis without a mood disturbance.<br />

Schizoaffective patients have psychosis<br />

at some point even when they are not<br />

experiencing an affective disturbance.<br />

Tip: Look at the names! For example, in<br />

schizoaffective disorder, the emphasis is<br />

on the schizo. The primary problem is a<br />

psychotic disorder that sometimes has<br />

a mood component, whereas in MDD<br />

with psychosis, the emphasis is on<br />

MDD. The main problem is a mood<br />

disorder that when severe, may have<br />

a component of psychosis.


92 Deja Review: <strong>Behavioral</strong> <strong>Science</strong><br />

What is a delusional disorder?<br />

What are some types of delusions<br />

that may occur in this disorder?<br />

What is a shared psychotic disorder?<br />

Patients with delusional disorder tend<br />

to have an isolated, fixed, nonbizarre<br />

delusion (eg, the Internal Revenue<br />

Service [IRS] is after them, or their partner<br />

is cheating). Even if the delusion is<br />

unfounded, it is plausible. They are not<br />

disorganized in thoughts or affect, and<br />

patients do not meet criteria for<br />

schizophrenia.<br />

1. Erotomanic: Someone is in love with<br />

patient.<br />

2. Somatic.<br />

3. Grandiose.<br />

4. Jealous: Wife/husband is cheating.<br />

5. Persecutory: Patient is being<br />

mistreated.<br />

A rare disorder where the patient<br />

believes the delusions of another person<br />

with a primary psychotic disorder.<br />

CLINICAL VIGNETTES<br />

A 27-year-old male presents to your clinic with a 4-year history of hallucinations<br />

and delusions for which he has been intermittently managed with antipsychotics.<br />

He is currently untreated. He also complains of 4 months of depressed mood,<br />

weight loss, insomnia, fatigue, and loss of interest in activities. He denies using<br />

any illicit substances or having any other medical conditions. What is the most<br />

likely diagnosis?<br />

Schizoaffective Disorder. Note that there have been psychotic symptoms without<br />

affective symptoms present, but now also meets criteria for a depressive episode.<br />

A 76-year-old man presents with 1 year of worsening depressive symptoms. He has<br />

trouble falling asleep, feels worthless, cannot concentrate, and has thoughts of<br />

death. Over 3 years ago his wife passed away from cancer. For 6 months now he<br />

has adamantly stated that the cancer was his fault and that he was the one that<br />

killed his wife, despite all evidence to the contrary. He also often hears her voice<br />

scolding him when no one is around. What is the most likely diagnosis?<br />

Depression with Psychotic Features. Note the mood-congruent delusion in the<br />

setting of depression and psychotic symptoms that only appear during affective<br />

episode. Also remember schizophrenia rarely presents in this advanced age.


CHAPTER 13<br />

Mood Disorders<br />

What are the DSM-IV-TR criteria for a<br />

major depressive episode?<br />

What is anhedonia?<br />

What is SIG E CAPS?<br />

Five or more of following symptoms<br />

present for at least 2 weeks (one of<br />

which must be either depressed mood<br />

or anhedonia):<br />

Depressed mood<br />

Anhedonia<br />

Fatigue<br />

Increase or decrease in sleep<br />

Increase or decrease in appetite or weight<br />

Decrease in ability to concentrate<br />

Feelings of worthlessness or guilt<br />

Psychomotor retardation<br />

Recurrent thoughts of death or suicidal<br />

ideation<br />

Loss of pleasure in all or almost all<br />

activities<br />

It is a mnemonic for depression:<br />

Sleep disturbances (mainly insomnia)<br />

Loss of Interest<br />

Excessive Guilt<br />

Loss of Energy<br />

Loss of Concentration<br />

Loss of Appetite<br />

Psychomotor retardation or agitation<br />

Suicidal ideation (or recurrent thoughts<br />

of death)<br />

What is the prevalence of major 5% to 12% for men and 10% to 20%<br />

depressive disorder (MDD)?<br />

for women, with a 2:1 female to<br />

male ratio<br />

93


94 Deja Review: <strong>Behavioral</strong> <strong>Science</strong><br />

Does MDD appear at a certain age? The mean age of onset is 40.<br />

What are the medical conditions that<br />

can cause or mimic a depressive<br />

episode?<br />

What medications can cause depressed<br />

mood?<br />

What is double depression?<br />

What is “postpartum onset” depression<br />

(aka postpartum depression, PPD)?<br />

What is the diagnostic criteria for<br />

dysthymic disorder?<br />

How can you differentiate dysthymic<br />

disorder from MDD?<br />

How do you treat depression<br />

pharmacologically?<br />

Thyroid dysfunction (particularly<br />

hypothyroidism)<br />

Malignancy<br />

Stroke (Post-stroke depression)<br />

Neurodegenerative disorders<br />

Mononucleosis<br />

Acquired immunodeficiency syndrome<br />

(AIDS)<br />

Syphilis<br />

Parkinson disease<br />

Stroke<br />

Multiple sclerosis<br />

Lupus<br />

Nutritional deficiency<br />

Menopause<br />

Accutane (isotretinoin)<br />

Reserpine<br />

Beta-blockers<br />

Steroids (eg, prednisone)<br />

Methyldopa<br />

Interferon (used to treat viral hepatitis)<br />

Oral Contraceptive Pills<br />

This term is used when a patient meets<br />

criteria for both MDD and dysthymic<br />

disorder.<br />

A major depressive episode happening<br />

within 4 weeks of delivery (some believe<br />

within 12 months of delivery is still<br />

PPD)<br />

Depressed mood for a least 2 years,<br />

plus two more symptoms of major<br />

depression, but not meeting criteria<br />

for MDD<br />

Dysthymic disorder must last longer,<br />

but is less severe.<br />

The first-line treatment option is the<br />

selective serotonin reuptake inhibitors<br />

(SSRIs) (see Table 13.1).


Mood Disorders 95<br />

Table 13.1 SSRIs and Their Unique Properties<br />

Brand<br />

Generic Name Name Side Effects Other<br />

Citalopram Celexa Standard Fewest medication<br />

(Sexual side<br />

interactions<br />

effects, nausea,<br />

vomiting,<br />

headache, anxiety,<br />

insomnia)<br />

Escitalopram Lexapro Standard Isomer of<br />

citalopram, very<br />

similar<br />

Fluoxetine Prozac Standard Long half-life,<br />

drug-drug<br />

interactions<br />

Paroxetine Paxil Standard, plus Short half-life, can<br />

has anticholinergic cause withdrawal<br />

effects that can<br />

symptoms (except<br />

cause weight gain, CR form), drugconstipation,<br />

drug interactions<br />

and sedation<br />

Sertraline Zoloft Most GI side Very few drug-drug<br />

effects<br />

interactions<br />

Abbreviations: GI, gatrointestinal.<br />

Are there any other antidepressants Yes, there are other antidepressants<br />

also considered as first-line treatment with different mechanism of action<br />

options? (see Table 13.2).


96 Deja Review: <strong>Behavioral</strong> <strong>Science</strong><br />

Table 13.2 Other Antidepressants Indicated for the Treatment of Depression<br />

Generic Brand Mechanism Side<br />

Name Name of Action Effects Other<br />

Bupropion Wellbutrin Unknown, likely Seizures, Contraindicated<br />

Zyban (for via dopamine insomnia, in patients with<br />

smoking and headache, seizures,<br />

cessation) norepinephrine nausea, bulimia, and<br />

vomiting, anorexia<br />

constipation,<br />

and tremor<br />

Used to treat<br />

sexual side<br />

effects caused<br />

by SSRIs<br />

Mirtazapine Remeron Presynaptic alpha-2- Highly Agranulocytosis<br />

adrenergic sedating, in 0.1%<br />

antagonist (↑ ↑appetite, Also used to<br />

norepinephrine ↑weight ↓ nausea<br />

and serotonin)<br />

also potent<br />

antagonist of<br />

5-HT 2 and<br />

5-HT 3<br />

Trazodone Desyrel Weakly inhibits Highly Used as a<br />

serotonin sedating, sleeping aid<br />

reuptake<br />

orthostatic<br />

hypotension<br />

Postsynaptically Uncommon<br />

antagonizes but can<br />

5-HT 2<br />

cause<br />

priapism<br />

(painful<br />

erection)<br />

and<br />

arrhythmias<br />

Nefazodone Serzone Similar to Headache, FDA black-box<br />

trazodone dry mouth, warning due<br />

blurred to risk of liver<br />

vision, failure<br />

somnolence,<br />

and<br />

orthostatic<br />

hypotension


Mood Disorders 97<br />

Table 13.2 Other Antidepressants Indicated for the Treatment of Depression (Continued )<br />

Generic Brand Mechanism Side<br />

Name Name of Action Effects Other<br />

Venlafaxine Effexor Serotonin and Nausea, Can increase<br />

norepinephrine dizziness, blood pressure<br />

reuptake<br />

sexual<br />

inhibitor, also dysfunction,<br />

inhibitor of headache,<br />

dopamine and dry<br />

reuptake at mouth<br />

high doses<br />

Duloxetine Cymbalta Similar to Nausea, dry Also indicated<br />

venlafaxine mouth, for neuropathic<br />

constipation, pain<br />

diarrhea,<br />

dizziness,<br />

and<br />

insomnia<br />

What are tricyclic antidepressants<br />

(TCAs)?<br />

Why are tricyclics a second-line<br />

treatment option?<br />

What are the side effects of tricyclic<br />

antidepressants?<br />

Older antidepressants (also known as<br />

heterocyclics) are currently considered<br />

a second-line treatment option.<br />

Greater side effects and lethality in<br />

overdose<br />

Anticholinergic effects: dry mouth,<br />

blurred vision, constipation, and<br />

confusion<br />

Alpha-blocking effects: sedation also<br />

caused by antihistaminic effects,<br />

orthostatic hypotension, and cardiac<br />

arrhythmias<br />

May lower the seizure threshold<br />

Are contraindicated in glaucoma<br />

Must be used with caution in urinary<br />

retention


98 Deja Review: <strong>Behavioral</strong> <strong>Science</strong><br />

What are the most commonly used<br />

tricyclics?<br />

Are there any other medications to<br />

treat depression?<br />

Why are MAOIs a third-line option?<br />

What is serotonin syndrome?<br />

When does a hypertensive crisis occur<br />

in the context of MAOI use?<br />

When are MAO inhibitors particularly<br />

efficacious?<br />

Are there any other ways to treat<br />

depression?<br />

What type of psychotherapy is used<br />

to treat depression?<br />

Amitriptyline, desipramine, imipramine,<br />

and nortriptyline<br />

Yes, the monoamine oxidase inhibitors<br />

(MAOIs), considered third-line option.<br />

MAOIs are rarely used any more.<br />

The main reason for not using them<br />

is due to the drug-drug interactions<br />

and diet restrictions. If these are not<br />

strictly followed, they may cause<br />

serotonin syndrome and<br />

hypertensive crisis.<br />

A clinical entity that consists of<br />

hyperthermia, muscle rigidity, and<br />

altered mental status. This syndrome<br />

is seen when MAOIs are combined<br />

with SSRIs (mostly), Demerol,<br />

pseudoephedrine, and other<br />

medications that can increase<br />

serotonin and norepinephrine.<br />

Hypertensive crisis originates when<br />

patients who are on MAOIs ingest food<br />

rich in tyramine, like wine and cheese,<br />

or take medications such as<br />

sympathomimetics, bronchodilators,<br />

DOPA, etc.<br />

Atypical depression (those with<br />

increased appetite and sleep) and<br />

treatment-resistant depression<br />

Yes, by using psychotherapy or<br />

electroconvulsive therapy in severe<br />

depression<br />

Cognitive-behavioral therapy, which<br />

attempts to recognize negative thoughts<br />

or behaviors and then tries to change<br />

them<br />

Psychodynamic psychotherapy, which<br />

focuses on self-understanding and<br />

inner conflicts<br />

Interpersonal therapy, which examines<br />

the patient’s problems in relation to<br />

their symptoms and how to deal with<br />

those problems


Mood Disorders 99<br />

What is electroconvulsive therapy<br />

(ECT)?<br />

When is ECT used?<br />

What are the main side effects<br />

of ECT?<br />

What are the DSM-IV-TR diagnostic<br />

criteria for a Manic Episode?<br />

What is the difference between<br />

hypomania and mania?<br />

What is a mixed episode?<br />

How do you diagnose Bipolar I<br />

Disorder?<br />

It is the induction of a generalized<br />

seizure by applying electric currents<br />

to the brain.<br />

It is mainly indicated for patients with<br />

refractory or psychotic depression, but<br />

it can also be used for the treatment of<br />

mania.<br />

Short-term memory loss<br />

A 1-week period of elevated,<br />

expansive, or irritable mood (or less<br />

if hospitalized) with at least three<br />

of the following (four if mood<br />

irritable):<br />

Grandiosity or inflated self-esteem<br />

Decreased need for sleep<br />

More talkative<br />

Flight of ideas/racing thoughts<br />

Distractibility<br />

Increased goal-directed activity<br />

Involvement in pleasurable and risky<br />

activities<br />

Hypomanic episodes last 4 days or<br />

more and do not require hospitalization<br />

or involve psychotic features. In<br />

general, hypomania is less severe<br />

than mania.<br />

A 1-week or longer period in which<br />

criteria for both manic and major<br />

depressive episodes are met.<br />

The patient has one or more manic or<br />

mixed episodes. Usually there is also a<br />

depressive episode, but this is not<br />

required for the diagnosis.


Table 13.3 Mood Stabilizers for Chronic Treatment of Bipolar Disorder and Unique<br />

Properties (Either I or II)<br />

Generic Name Brand Name Side Effects Other<br />

Carbamazepine Tegretol, Sedation, GI, Induces<br />

Carbatrol, reversible mild metabolism of<br />

Equetro leukopenia, itself and other<br />

reversible mild medications,<br />

increase in LFTs, decreasing its<br />

tremor,<br />

hyponatremia<br />

own level during<br />

therapy<br />

Associated with<br />

aplastic anemia and<br />

agranulocytosis<br />

Must monitor<br />

levels<br />

Lamotrigine Lamictal Rash may occur Good for<br />

in 10% of<br />

depressed state,<br />

individuals, may rapid cycling,<br />

rarely<br />

and mixed states<br />

progress to Start low and go<br />

Stevens-Johnson slow to avoid rash<br />

syndrome (rare) No level<br />

has been<br />

monitoring<br />

reported<br />

required<br />

Lithium Eskalith Tremor, Narrow<br />

nephrogenic therapeutic<br />

diabetes<br />

index, can cause<br />

insipidus,<br />

toxicity with ↑<br />

hypothyroidism, sweating,<br />

GI symptoms, NSAIDs, or<br />

teratogen,<br />

thiazides<br />

weight gain diuretics<br />

Must monitor<br />

levels<br />

Valproic acid Depakote Rare fatal Inhibits<br />

hepatotoxicity, metabolism of<br />

agranulocytosis other<br />

Teratogen: folic medications<br />

acid antagonist (↑ their level)<br />

→neural tube Must check LFTs<br />

defects<br />

and monitor<br />

Benign elevation level<br />

of transaminases,<br />

GI side effects,<br />

sedation, tremor,<br />

alopecia,<br />

weight gain<br />

Abbreviation: LFTs, Liver function tests; NSAIDs, nonsteroidal anti-inflammatory drugs.<br />

100


Mood Disorders 101<br />

How do you diagnose Bipolar II<br />

Disorder?<br />

Are there any drugs that induce mania?<br />

What is cyclothymic disorder?<br />

What is the prevalence of bipolar<br />

disorder?<br />

The patient has one or more major<br />

depressive episodes, plus one or more<br />

hypomanic episodes (but never any<br />

manic episodes).<br />

Yes, steroids and appetite suppressants<br />

are the main culprits. Cocaine (crack)<br />

and amphetamines may also induce it.<br />

Previously undiagnosed bipolar<br />

patients who have only been treated<br />

for a major depression may have their<br />

manic episodes precipitated by<br />

treatment with antidepressants.<br />

Cyclothymic patients meet criteria for<br />

dysthymic disorder and experience<br />

hypomanic episodes.<br />

Bipolar I has a lifetime prevalence of<br />

0.5% to 1% and a male to female ratio<br />

of 1:1. While, bipolar II has a lifetime<br />

prevalence of 0.5% and is more common<br />

in women than in men.<br />

CLINICAL VIGNETTES<br />

You are the third-year medical student on the psychiatry consult service. You and<br />

your resident are called to the ER for a patient who was recently admitted to the<br />

inpatient psychiatry service for depression and suicidal ideation. While in the<br />

hospital, he complained of having trouble sleeping and was given a medication to<br />

help both his depression and sleep problems. The patient also received a<br />

prescription for this medication when he was discharged. He now presents to the<br />

ER with a 3-hour history of a painful erection. What drug was the patient most<br />

likely given?<br />

Trazodone<br />

A patient you have been treating for depression with SSRIs comes to you for a<br />

checkup. When you ask how he is he says “The last 3 days have been fantastic!”<br />

and launches into a very long and tangential story about what he has been doing.<br />

The patient has been very productive at work, though that might have to do more<br />

with the small amount of sleep that he has been getting, but he claims that he is<br />

far too strong to need sleep. What is your new diagnosis?<br />

Bipolar, type II. This meets criteria for a hypomanic episode, but because of the<br />

lack of psychosis or need for hospitalization it is not yet mania. He is also very<br />

functional during this time.


102 Deja Review: <strong>Behavioral</strong> <strong>Science</strong><br />

A 29-year-old woman comes to the ER worried about a new rash. She has never<br />

had a rash like this before. She has no risk factors that you can identify and hasn’t<br />

been hiking in any areas with poison oak. She says she has just started therapy<br />

with an anticonvulsant drug to treat her bipolar disorder. She has no allergies that<br />

she knows of.<br />

What drug is she likely taking?<br />

Lamotrigine, a maintenance therapy for bipolar disorder. She is experiencing a<br />

drug rash, which is common with this drug.<br />

What should you do at this point?<br />

Stop the medication<br />

What is the serious complication you would like to avoid?<br />

Stevens-Johnson Syndrome. Though rash is common, rarely some patients will<br />

progress to Stevens-Johnson Syndrome.


CHAPTER 14<br />

Anxiety Disorders<br />

What are the most prevalent psychiatric<br />

disorders?<br />

What are the major types of anxiety<br />

disorders?<br />

What is the most prominent feature<br />

of all anxiety disorders?<br />

What differentiates anxiety disorders<br />

from healthy anxiety?<br />

What are the DSM-IV-TR diagnostic<br />

criteria for GAD?<br />

Anxiety disorders<br />

Generalized anxiety disorder (GAD)<br />

Panic disorder<br />

Social phobia<br />

Specific phobia<br />

Obsessive-compulsive disorder<br />

Agoraphobia<br />

Posttraumatic stress disorder (PTSD)<br />

Acute stress disorder (ASD)<br />

The presence of a sense of impending<br />

doom or threat<br />

Anxiety out of proportion to actual<br />

threat which causes significant distress<br />

or impairment<br />

Six months of excessive anxiety and<br />

worry, more days than not, that are<br />

difficult to control. Plus three or<br />

more of the following:<br />

Restlessness<br />

Fatigue<br />

Tension<br />

Sleep disturbance<br />

Irritability<br />

Decreased concentration<br />

Note: The worry may not be about<br />

symptoms of another disorder, eg,<br />

worrying about having a panic<br />

attack is panic disorder, not GAD.<br />

103


104 Deja Review: <strong>Behavioral</strong> <strong>Science</strong><br />

What GI disease is associated<br />

with GAD?<br />

Irritable bowel syndrome; 50% of<br />

patients also have GAD.<br />

What is the male to female ratio of 1:2<br />

GAD prevalence?<br />

What are important nonpsychiatric<br />

differentials to consider for GAD?<br />

What is the initial pharmacological<br />

treatment for GAD?<br />

What medications give rapid relief,<br />

but have significant long-term abuse<br />

potential?<br />

Substance-induced anxiety<br />

Substance withdrawal anxiety<br />

General medical conditions<br />

Pheochromocytoma<br />

Hyperthyroidism<br />

Electrolyte abnormalities<br />

Selective serotonin reuptake inhibitor<br />

(SSRI)<br />

Venlafaxine<br />

Buspirone<br />

Mirtazapine<br />

Benzodiazepines<br />

PANIC DISORDER<br />

What are the DSM-IV-TR criteria for a<br />

panic attack?<br />

What characterizes panic disorder?<br />

Four or more symptoms, which come<br />

up abruptly:<br />

Anxiety<br />

Palpitations<br />

Sweating<br />

Dizziness<br />

Trembling<br />

Shortness of breath<br />

Chest pain<br />

Nausea<br />

Feeling of choking<br />

Chills<br />

Fear of dying<br />

Fear of going crazy or losing control<br />

Feeling detached from one’s self<br />

Recurrent spontaneous panic attacks<br />

and anxiety about future attacks, their<br />

implications, or changing behavior to<br />

avoid attacks


Anxiety Disorders 105<br />

What differentiates panic attacks from<br />

panic disorders?<br />

A panic disorder develops when the<br />

patient has recurrent panic attacks and<br />

has anticipatory fear about future panic<br />

attacks.<br />

What is the male to female ratio for 1:3<br />

panic disorder?<br />

What is the age of onset of panic<br />

disorder?<br />

Is family history of panic disorders<br />

relevant?<br />

What are the most common psychiatric<br />

sequelae of panic attacks?<br />

What is the most effective psychotherapy<br />

for panic disorder?<br />

What is the pharmacologic treatment<br />

of panic disorder?<br />

What are the important nonpsychiatric<br />

differential diagnoses for panic<br />

disorders?<br />

What phobia may occur with<br />

Panic Disorder?<br />

Mid-twenties<br />

Yes. Evidence does support an<br />

upregulation of adrenergic output<br />

responsible for stimulating anxiety<br />

centers in the brain which predisposes<br />

certain people to panic attacks.<br />

Agoraphobia. Patients have anticipatory<br />

fear which prevents them from venturing<br />

outside alone.<br />

Cognitive behavioral therapy including<br />

relaxation techniques<br />

SSRIs (first-line)<br />

Tricyclic antidepressant (TCA),<br />

monoamine oxidase inhibitors (MAOIs)<br />

(second-line)<br />

Arrhythmias<br />

Angina<br />

Hypoxia<br />

Hyperthyroidism<br />

Substance-induced intoxication or<br />

withdrawal<br />

Agoraphobia<br />

OBSESSIVE-COMPULSIVE DISORDER<br />

What are the diagnostic criteria for<br />

obsessive-compulsive disorder (OCD)?<br />

What is an obsession?<br />

Presence of obsessions and/or compulsive<br />

behavior that are time consuming and<br />

cause significant distress or impairment<br />

of functioning<br />

Persistent, recurrent thoughts which<br />

cause irrational anxiety


106 Deja Review: <strong>Behavioral</strong> <strong>Science</strong><br />

What is a compulsion?<br />

What differentiates OCD from<br />

obsessive-compulsive personality<br />

disorder (OCPD)?<br />

A repetitive behavior that reduces<br />

anxiety—behavior may not be directly<br />

related to the obsession<br />

OCD patients are aware that their<br />

anxiety and compulsions are<br />

unreasonable. OCPD patients are<br />

orderly, inflexible, and perfectionistic—<br />

but see nothing wrong with it.<br />

What is the male to female ratio 1:1<br />

for OCD?<br />

What is the age of onset of OCD?<br />

What common psychiatric diagnoses<br />

are associated with OCD?<br />

What is the most effective psychotherapy<br />

for OCD?<br />

What is the pharmacologic treatment<br />

of OCD?<br />

What are the important differential<br />

diagnoses for OCD?<br />

Late adolescence to early adulthood<br />

Major depression<br />

Generalized Anxiety Disorder<br />

Tourette syndrome<br />

Note: It is common for patients with<br />

Tourette to have OCD, but it is not<br />

nearly as common for patients with<br />

OCD to have Tourette.<br />

Cognitive-behavioral therapy<br />

(exposure-response prevention,<br />

flooding, thought stopping)<br />

High-dose SSRI (first-line treatment)<br />

Clomipramine (second-line treatment)<br />

Specific phobia<br />

GAD<br />

Body dimorphic disorder<br />

Trichotillomania<br />

SOCIAL PHOBIA<br />

What are the diagnostic criteria<br />

for social phobia?<br />

What characterizes social phobias?<br />

Persistent fear of social situations<br />

Anxiety or panic attack when exposed<br />

to social situation<br />

Avoidance or distress of feared situation<br />

results in functional impairment<br />

Excessive, irrational fear of public or<br />

social situations, in which the patient<br />

could be scrutinized for his or her<br />

performance.


Anxiety Disorders 107<br />

What differentiates social phobias<br />

from specific phobias?<br />

Social phobias involve fear of public<br />

situations where scrutiny could occur.<br />

On the other hand, specific phobias<br />

involve fear of specific objects or<br />

situations not associated with scrutiny.<br />

What is the male to female ratio 1:1<br />

for social phobia?<br />

What is the age of onset of<br />

social phobias?<br />

What are the most common<br />

social phobias?<br />

What is the most effective<br />

psychotherapy for social phobias?<br />

What is the pharmacologic treatment<br />

of social phobias?<br />

What medications are effective<br />

for immediate relief of symptoms,<br />

but must be used cautiously?<br />

What are the important differential<br />

diagnoses for social phobia?<br />

Adolescence following childhood<br />

shyness<br />

Fear of public speaking<br />

Fear of public performances<br />

Fear of answering questions in class<br />

Cognitive-behavioral therapy including<br />

relaxation techniques<br />

SSRIs (first-line treatment)<br />

Beta-blockers<br />

Benzodiazepines—should be avoided in<br />

those with substance abuse potential<br />

Specific phobias<br />

Panic disorder<br />

OCD<br />

POSTTRAUMATIC STRESS DISORDER<br />

What are the DSM-IV-TR criteria for<br />

posttraumatic stress disorder (PTSD)?<br />

What is increased arousal in PTSD?<br />

Exposure to a traumatic event involving<br />

threat to self or others, plus 1 month of:<br />

Reexperiencing the event (flashbacks,<br />

dreams, reliving the trauma, etc)<br />

Avoidance of situations and thoughts<br />

associated with the trauma<br />

Increased arousal<br />

Anger or irritability<br />

Insomnia<br />

Hypervigilance<br />

Increased startle response<br />

Difficulty concentrating<br />

Note: Diagnosis requires two or more<br />

of above.


108 Deja Review: <strong>Behavioral</strong> <strong>Science</strong><br />

What differentiates PTSD from acute<br />

stress disorder?<br />

What differentiates PTSD from<br />

adjustment disorder?<br />

Patients with PTSD have symptoms for<br />

>1 month that can begin >1 month after<br />

the traumatic event occurs. Patients<br />

with acute stress disorder have symptoms<br />

lasting from 2 days to 1 month which<br />

begin within 1 month of traumatic<br />

event.<br />

PTSD involves life-threatening events<br />

(rape, war, etc)<br />

Adjustment disorders involve non-lifethreatening<br />

events (divorce, death of<br />

others)<br />

What is the male to female ratio 1:2<br />

for PTSD?<br />

How is PTSD classified?<br />

What is the most effective psychotherapy<br />

for PTSD?<br />

What is the pharmacologic treatment<br />

of PTSD?<br />

What medication is particularly effective<br />

for PTSD nightmares?<br />

What are important differential<br />

diagnoses for PTSD?<br />

Acute PTSD: Symptoms last 3 months.<br />

Delayed-onset PTSD: Symptoms begin<br />

>6 months after the life-threatening<br />

event.<br />

Cognitive-behavioral therapy (CBT)<br />

and support groups<br />

SSRIs (first-line treatment)<br />

Mood stabilizers<br />

Prazosin (alpha-blocker)<br />

Acute stress disorder<br />

Adjustment disorder<br />

OCD<br />

Malingering<br />

SPECIFIC PHOBIA<br />

What are the symptoms of a specific<br />

phobia?<br />

Fear of specific situations<br />

Anxiety<br />

Palpitations<br />

Sweating<br />

Headache<br />

Restlessness


Anxiety Disorders 109<br />

What characterizes specific phobias?<br />

What differentiates specific phobias<br />

from social phobias?<br />

Excessive, irrational fear of specific<br />

objects or situations<br />

Specific phobias involve fear of specific<br />

objections or situations not associated<br />

with being scrutinized.<br />

What is the male to female ratio for 1:2<br />

specific phobia?<br />

What is the age of onset of specific<br />

phobias?<br />

What are the most common types of<br />

specific phobias?<br />

What is the most effective psychotherapy<br />

for specific phobias?<br />

What is the pharmacologic treatment<br />

of specific phobias?<br />

What are important differential<br />

diagnoses for specific phobias?<br />

Mostly childhood but can begin at<br />

anytime<br />

Animals<br />

Storms<br />

Heights<br />

Illness<br />

Injury<br />

Flooding<br />

Gradual desensitization<br />

Hypnosis<br />

CBT<br />

Benzodiazepines for certain phobias<br />

such as flying<br />

Social phobia<br />

OCD<br />

GAD<br />

Panic disorder<br />

ADJUSTMENT DISORDER WITH ANXIETY<br />

What is the main characteristic of<br />

adjustment disorder with anxiety?<br />

During what time period should the<br />

symptoms present after stressor occurs?<br />

When should symptoms of adjustment<br />

disorder with anxiety resolve?<br />

What are common causes of adjustment<br />

disorder?<br />

Emotional or behavioral symptoms<br />

associated with an identifiable stressor<br />

Within 3 months of initial stressor<br />

Within 6 months of initial symptoms if<br />

stressor removed<br />

Divorce<br />

Relocation<br />

Attending a new school


110 Deja Review: <strong>Behavioral</strong> <strong>Science</strong><br />

What should be ruled out before<br />

diagnosing adjustment disorders?<br />

How does adjustment disorder with<br />

anxiety present?<br />

What are important differential<br />

diagnoses?<br />

Bereavement<br />

Any Axis I diagnosis<br />

Occupational impairment<br />

Social impairment<br />

Scholastic impairment<br />

Bereavement<br />

Major depressive disorder<br />

Acute stress disorder<br />

PTSD<br />

CLINICAL VIGNETTES<br />

A 47-year-old woman describes herself as a “worrier for most of my life.” She<br />

doesn’t worry about anything in particular, but she lays awake at night thinking<br />

about things. It’s exhausting and she can’t seem to get anything else done—of<br />

course, it doesn’t help that she can’t concentrate anyway. What is the most likely<br />

diagnosis?<br />

Generalized Anxiety Disorder<br />

A 32-year-old man was involved in an industrial accident that killed his best<br />

friend. He was right next to him when it happened, and he remembers every bit of<br />

it. In fact, sometimes he feels like he relives it. He has recurrent nightmares about<br />

the event, can’t seem to get to sleep, and can’t face going back to work.<br />

What is the most likely diagnosis?<br />

PTSD<br />

How would you treat his nightmares?<br />

His nightmares may be responsive to medical therapy with Prazosin.<br />

A 26-year-old female presents to your clinic with complaints about episodes of shortness<br />

of breath, palpitations, tingling around her mouth, and blurry vision. She has only<br />

experienced two of these episodes, but she fears that she may have more. She asks<br />

you to start her on some medication to help with these fears and the episodes<br />

themselves. Her last menstrual period was 7 weeks ago, and she has a history of<br />

cocaine abuse.<br />

What class of medication would be most useful in providing immediate relief<br />

of symptoms for this patient?<br />

Benzodiazepine<br />

Why might this class of medications be contraindicated in this patient?<br />

She may be pregnant (most benzodiazepines are pregnancy category D) and has<br />

a history of substance abuse making treatment with benzodiazepines problematic.


CHAPTER 15<br />

Cognitive Disorders<br />

Name the three major categories<br />

of cognitive disorders.<br />

1. Delirium<br />

2. Dementia<br />

3. Amnestic disorders<br />

DELIRIUM<br />

What is delirium?<br />

What causes delirium?<br />

While most substances of abuse can<br />

cause delirium with acute intoxication,<br />

which three are most likely to cause<br />

delirium related to withdrawal?<br />

What factors predispose to delirium?<br />

Delirium is a disturbance of consciousness<br />

and attention that usually develops over<br />

a short period of time. The confusion<br />

and memory impairment is not better<br />

accounted for by a preexisting dementia.<br />

Any disorder that causes a disruption<br />

of brain physiology, most commonly a<br />

medical or surgical condition, medication.<br />

The cause is often multifactorial. Elderly<br />

patients and those with underlying<br />

dementia are at higher risk.<br />

Alcohol (delirium tremens),<br />

benzodiazepines, and barbiturates<br />

Acute medical illness<br />

Age: elderly and young children<br />

Preexisting brain damage: dementia,<br />

cerebrovascular disease, tumor<br />

A history of delirium<br />

Advanced Cancer<br />

What are some medications physicians<br />

give patients that contribute<br />

to delirium?<br />

111<br />

A long list, including:<br />

Anticholinergics (including Benadryl)<br />

Benzodiazepines<br />

Levodopa and other dopaminergics<br />

Opioids<br />

Cardiac medications, eg, Beta-Blockers<br />

Many antibiotics


112 Deja Review: <strong>Behavioral</strong> <strong>Science</strong><br />

How common is delirium?<br />

Clinically, what is the hallmark<br />

of delirium?<br />

In addition to alterations in<br />

consciousness, what other features<br />

are often found in a patient<br />

with delirium?<br />

How do you test for delirium?<br />

How do you treat delirium?<br />

While treating the cause of a delirium,<br />

how do you manage a delirious<br />

patient?<br />

Very common. Roughly 15% to 20% of<br />

general hospital patients will have an<br />

episode of delirium. In patients over the<br />

age of 65, the prevalence can be as high<br />

as 30%!<br />

Altered level of consciousness<br />

(especially attention and level of<br />

arousal). It typically develops over a<br />

period of hours to days and mental<br />

status alterations wax and wane during<br />

the day, with periods of lucidity.<br />

Altered sleep-wake cycle<br />

Perceptual disturbance<br />

Impaired memory and orientation<br />

Nocturnal worsening of symptoms<br />

Psychomotor agitation<br />

Though there is no lab test for delirium,<br />

a good cognitive tool like the minimental<br />

status examination can help<br />

identify delirium. Cognitive disturbance<br />

from delirium tends to fluctuate.<br />

Treat any medical cause, stop medications<br />

that may be contributing (if possible),<br />

orient the patient frequently, and place<br />

in a well-lit room during the day.<br />

Low doses of antipsychotics, like<br />

haloperidol or an atypical antipsychotic<br />

can be helpful for agitation and<br />

hallucinations.<br />

DEMENTIA<br />

What is the core symptom of dementia?<br />

In order to diagnose dementia, what<br />

must be present in addition to memory<br />

impairment?<br />

Memory impairment<br />

At least one of the following:<br />

Aphasia (language and naming problems)<br />

Apraxia (impaired ability to do learned<br />

motor tasks, like using objects)<br />

Agnosia (difficulty recognizing or<br />

identifying objects)<br />

Disturbance of executive function (the<br />

ability to plan, organize, and carry out<br />

tasks, judgment)


Cognitive Disorders 113<br />

How does dementia differ from the<br />

normal memory changes of aging?<br />

When diagnosing dementia, what other<br />

disorders in your differential are key<br />

to rule out?<br />

What is the prevalence of dementia?<br />

What is the most common type<br />

of dementia?<br />

What is the classical clinical course<br />

for Alzheimer disease?<br />

How does this differ from the course<br />

of vascular dementia?<br />

As we age, we are less able to learn new<br />

information, and we process information<br />

at a slower speed.<br />

However, these changes do not normally<br />

interfere with the basic functioning.<br />

It is crucial that you do not miss a<br />

delirium or a depression. In the elderly,<br />

it is not uncommon for them to report<br />

multiple memory complaints. If you<br />

misdiagnose this as dementia, you will<br />

miss a potentially reversible cause of<br />

memory impairment. Likewise, if you<br />

miss a delirium, you may miss a<br />

potentially serious medical problem.<br />

Additionally, there are several potentially<br />

reversible causes of dementia that you<br />

should look for, including: neurosyphilis,<br />

vitamin B 12 , thiamine, and folate<br />

deficiencies, and normal pressure<br />

hydrocephalus.<br />

Incidence/prevalence increases with<br />

age. The prevalence is approximately<br />

1.5 % in those over 65 years of age. The<br />

prevalence increases to 20% after age 85.<br />

Alzheimer Disease (AD) represents<br />

about 50% to 60% of dementias. The<br />

second most common form is vascular<br />

dementia (formerly multi-infarct<br />

dementia)<br />

Others include front temporal (Pick<br />

and Creutzfeldt-Jakob), Parkinson,<br />

Huntington, and human<br />

immunodeficiency virus (HIV)<br />

dementias.<br />

Slow, gradual onset of memory loss<br />

and cognitive impairment (often there<br />

are problems with judgment, mood<br />

symptoms, and behavioral disturbances<br />

as well). The disease is progressive and<br />

death usually occurs within 3 years<br />

after diagnosis.<br />

Vascular dementia classically has a<br />

stepwise decline, as opposed to the slow<br />

and steady decline in Alzheimer. Onset<br />

of deficits may be abrupt, and with good<br />

control of cardiovascular risk factors the<br />

course may remain relatively stable.


114 Deja Review: <strong>Behavioral</strong> <strong>Science</strong><br />

What are the major risk factors for<br />

Alzheimer disease?<br />

On postmortem examination, what<br />

changes are normally seen in the<br />

brain of an Alzheimer disease patient?<br />

What areas of the brain show cell loss<br />

in Alzheimer disease?<br />

What class of medications is used<br />

to slow the progression of Alzheimer<br />

disease?<br />

Age, family history, Apo E4 allele, and<br />

Down syndrome<br />

Neurofibrillary tangles and senile<br />

(amyloid) plaques<br />

While there is also often global cortical<br />

atrophy, neuronal degeneration is<br />

classically in the cholinergic neurons<br />

of the nucleus basalis of Meynert.<br />

Cholinesterase inhibitors such as<br />

donepezil, rivastigmine, and<br />

galantamine may be used. Memantine<br />

(Namenda) is an NMDA (N-methyl-Daspartate)<br />

receptor antagonist that<br />

shows promise for slowing progression<br />

in AD.<br />

Table 15.1 Delirium vs Dementia<br />

Delirium<br />

Dementia<br />

Onset Develops quickly (hours to days) Develops over weeks<br />

to years<br />

Key feature Impaired attention and level of Impaired memory<br />

consciousness<br />

with normal level<br />

of consciousness<br />

Course Fluctuates within the course Usually stable within<br />

of a day with lucid periods<br />

a day<br />

Worsens at night<br />

May worsen at night<br />

(sundowning)<br />

Occurrence Most common in elderly and Increases with age<br />

young children<br />

Psychiatric Hallucinations and delusions Hallucinations and<br />

Symptoms may be present delusions may be<br />

present<br />

Physical Abnormal EEG Normal EEG<br />

findings Acute medical illness No acute medical<br />

illness<br />

Prognosis Symptoms tend to resolve with Usually progressive<br />

treatment of underlying cause


Cognitive Disorders 115<br />

AMNESTIC SYNDROMES<br />

How do amnestic syndromes differ<br />

from dementia?<br />

Which brain structures are affected<br />

in amnestic syndromes?<br />

Damage to mediotemporal structures<br />

is associated with what vitamin<br />

deficiency?<br />

Name at least four other etiologies<br />

of amnestic syndromes.<br />

In amnestic syndromes, the disturbance<br />

of function is isolated to memory, while<br />

other cognitive functions remain<br />

relatively intact (unlike dementia).<br />

The bilateral mediotemporal structures<br />

(eg, mammillary bodies, hippocampus,<br />

fornix)<br />

Thiamine deficiency. Often this is<br />

related with chronic alcohol abuse<br />

(Korsakoff syndrome)<br />

1. Traumatic brain injury<br />

2. Herpes encephalitis<br />

3. Cerebrovascular disease<br />

4. Hypoxia<br />

CLINICAL VIGNETTES<br />

A 79-year-old African American woman has dementia. Her son has heard about the<br />

genetic basis for Alzheimer disease and is worried about his risk of developing the<br />

disease. Looking over her chart you see that she was high functioning 10 years prior<br />

and then first suddenly became rather mildly cognitively impaired about 7 years<br />

ago. She continued at that level until 5 years ago when she developed significant<br />

memory problems and needed some part-time help at home. Two years ago she<br />

again got worse and required assisted living. What do you tell her son about his<br />

risks of Alzheimer disease?<br />

His risk is the same as the normal population. The patient’s history with a<br />

“stepwise decline” is suggestive of vascular dementia, not AD.<br />

You are on the psychiatric consult service and called to see a psychotic patient in<br />

the ICU. The resident there tells you that the patient—a 57-year-old woman with<br />

no prior psychiatric history—is having new-onset schizophrenia. She seemed fine<br />

this morning, but this evening she has hallucinations and is yelling at the nursing<br />

staff. What is the likely diagnosis in this patient? What can be done to help, aside<br />

from medications?<br />

This patient most likely has delirium and almost absolutely does not have<br />

schizophrenia. You should be wary of diagnosing an older individual with a newonset<br />

psychotic disorder, especially when underlying medical illness is actively<br />

involved. Delirium is so common in acutely sick patients that it is called “ICU<br />

psychosis.” First and foremost, trying to alleviate the medical condition (or stopping<br />

the offending medication) which has precipitated the delirium is the first course<br />

of action. Environmental factors may help, such as cues to help orient patients to<br />

time, date, and place.


116 Deja Review: <strong>Behavioral</strong> <strong>Science</strong><br />

A 57-year-old man with a history of end-stage liver disease and peptic ulcers<br />

underwent surgery to repair a perforated ulcer 2 days ago. He is agitated, confused,<br />

and he believes bugs are crawling on him. He appears floridly delirious and you<br />

begin his workup. His blood work reveals no evidence of infection, his blood<br />

pressure is becoming more labile, and heart rate is increasing. Your attending<br />

physician shows up and is very concerned that he may seize. What is the most<br />

likely etiology for the delirium?<br />

Alcohol withdrawal


CHAPTER 16<br />

Somatoform Disorders<br />

What are the primary types<br />

of somatoform disorders?<br />

What is factitious disorder?<br />

What is malingering?<br />

What is the primary difference between<br />

somatoform disorders and factitious<br />

disorder or malingering?<br />

Somatization disorder<br />

Conversion disorder<br />

Hypochondriasis<br />

Body dysmorphic disorder<br />

Pain disorder<br />

A disease in which the patient<br />

intentionally feigns illness to assume<br />

the sick role, but with no other obvious<br />

motivation (aka Munchausen<br />

syndrome)<br />

Intentional feigning of illness for the<br />

purpose of gaining a conscious reward<br />

Somatoform disorders are unconscious<br />

behaviors, whereas behaviors in<br />

factitious disorders and malingering<br />

are consciously derived by the patient.<br />

Table 16.1<br />

Conscious Behavior?<br />

Motivation Apparent?<br />

Somatoform No No<br />

Conversion No Yes<br />

Factitious Yes No<br />

Malingering Yes Yes<br />

Are there any physical examination<br />

findings which are common<br />

in somatoform disorders?<br />

No. Disease etiology must be ruled out<br />

in these patients; however, there is no<br />

medical condition that can account for<br />

their symptoms.<br />

117


118 Deja Review: <strong>Behavioral</strong> <strong>Science</strong><br />

What are some disorders with<br />

nonspecific symptoms that should<br />

be thought of before somatoform<br />

disorder is diagnosed?<br />

Should patients with somatoform<br />

disorders be told that they are<br />

imagining their symptoms?<br />

What is a good way to frame<br />

the possibility of a lack of general<br />

medical problem?<br />

What is a good statement to make<br />

when discussing the possibility<br />

of psychiatric treatment with a patient?<br />

SLE, Multiple Sclerosis, Myasthenia<br />

gravis, Hyperparathyroidism, Thyroid<br />

disturbance, Porphyria, Malignancy<br />

No. Supportive treatment with<br />

suggestions that psychotherapy may<br />

alleviate their distress is associated with<br />

a better prognosis. Remember that pain<br />

is a subjective experience.<br />

Reassurance that there is no serious<br />

underlying medical cause, but without<br />

dismissing their symptoms—this may<br />

alleviate some anxiety.<br />

“Many of my patients with similar<br />

symptoms have found a lot of benefit<br />

from...” [SSRIs, psychotherapy, etc]<br />

SOMATIZATION DISORDER<br />

What are the diagnostic criteria<br />

of complaints seen in somatization<br />

disorder?<br />

Patients must manifest four pain,<br />

two gastrointestinal, one sexual, and<br />

one neurologic symptoms, all of which<br />

cannot be fully explained by medical<br />

etiology.<br />

What is the age of onset of<br />

Somatic complaints must begin prior<br />

somatization disorder? to age 30.<br />

What are common patterns of behaviors<br />

seen in these patients?<br />

Can the symptoms occur in the presence<br />

of a true medical condition?<br />

What is the prevalence of somatization<br />

disorders in the United States?<br />

What are risk factors for somatization<br />

disorder?<br />

What is the most common first<br />

manifestation of this disorder<br />

in women?<br />

These patients often have had multiple<br />

exploratory surgeries and visit multiple<br />

doctors.<br />

Yes, however, the complaints will often<br />

be in excess of what would be normally<br />

expected.<br />

0.2% to 2% in women and


Somatoform Disorders 119<br />

What is the treatment for somatization<br />

disorders?<br />

Regular brief appointments with the<br />

primary care provider are often useful<br />

in reassuring these patients. Unnecessary<br />

laboratory tests or procedures should not<br />

be performed. In addition, psychotherapy<br />

may be used.<br />

CONVERSION DISORDER<br />

From which body system are<br />

complaints derived from in<br />

conversion disorders?<br />

What are the most common<br />

manifestations seen?<br />

What is a common presentation?<br />

What are common findings on physical<br />

examination of these patients?<br />

What is a common association<br />

with the onset or exacerbation<br />

of symptoms?<br />

What medical disorder can present<br />

similarly to a conversion disorder?<br />

Neurologic (motor and/or sensory)<br />

Sudden onset of blindness<br />

Paralysis<br />

Paresthesias<br />

Seizures<br />

Dramatic onset of symptoms that are<br />

often physiologically impossible<br />

Abnormalities do not have anatomical<br />

distribution and neurologic examination<br />

is normal.<br />

Often stressful life events precede the<br />

development of symptoms.<br />

Multiple sclerosis<br />

What percentage of individuals Very few—approximately 4%<br />

diagnosed with a conversion disorder<br />

in fact have a true neurological<br />

condition?<br />

Do patients with true conversion<br />

disorders sustain injury as a result<br />

of their condition?<br />

Are patients with conversion disorder<br />

distressed over their condition?<br />

No. Patients with sudden onset of<br />

blindness do not run into objects,<br />

and those with paralysis may still<br />

inadvertently move when distracted.<br />

No. These patients are often calm<br />

regarding their pseudoneurologic<br />

deficits, termed la belle indifference.<br />

Note: This is not diagnostic of conversion<br />

disorder.


120 Deja Review: <strong>Behavioral</strong> <strong>Science</strong><br />

Do patients with conversion disorder<br />

intentionally derive any gain from<br />

their symptoms?<br />

What is the prognosis and duration<br />

of symptoms seen in conversion<br />

disorders?<br />

What is the treatment for conversion<br />

disorders?<br />

No<br />

Symptoms typically remit within 2 weeks,<br />

with favorable prognosis seen in paralysis<br />

and blindness, and a poor prognosis<br />

associated with seizures.<br />

Reassurance that the symptoms will<br />

improve usually results in resolution of<br />

the symptoms (self-limiting), however,<br />

conversion disorder often reoccurs later.<br />

Psychotherapy can be helpful.<br />

HYPOCHONDRIASIS<br />

What is the main characteristic<br />

of hypochondriasis?<br />

Are there true physical symptoms<br />

seen in this disorder?<br />

What are the most common presenting<br />

symptoms seen in hypochondriasis?<br />

Is there a gender predominance seen<br />

in hypochondriasis?<br />

What are common associations with<br />

the development of hypochondriasis?<br />

What are common behaviors seen<br />

in these patients?<br />

Preoccupation with having a serious<br />

disease, despite medical reassurance<br />

of health status<br />

Yes, however, the symptoms are<br />

misinterpreted by the patient as being<br />

of a greater significance.<br />

Nausea<br />

Abdominal pain<br />

Chest pain<br />

Palpitations<br />

No, men and women are equally<br />

affected.<br />

Often the person has experienced<br />

serious illness in childhood or knows<br />

someone who has died or suffered<br />

through a serious medical condition.<br />

Doctor shopping is common, as these<br />

patients are resistant to suggestions that<br />

there is no significant medical etiology<br />

to their symptoms.<br />

What is the prevalence of 1% to 5%<br />

hypochondriasis?<br />

What is the treatment for<br />

hypochondriasis?<br />

Group therapy and frequent reassurance<br />

with regular but brief visits to primary<br />

care physician


Somatoform Disorders 121<br />

BODY DYSMORPHIC DISORDER<br />

What is the main characteristic<br />

of body dysmorphic disorder?<br />

Is the defect always imagined<br />

by the patient?<br />

What are the two components of body<br />

dysmorphic disorder?<br />

What are risk factors for the development<br />

of body dysmorphic disorder?<br />

Where do these patients often present?<br />

What are the most common features<br />

viewed as defective in these patients?<br />

What is the most common comorbid<br />

psychiatric disorder associated with<br />

body dysmorphic disorder?<br />

What are some common behaviors<br />

seen in this disorder?<br />

What other diagnoses must be<br />

considered in the differential?<br />

Do surgical procedures and alterations<br />

tend to improve the patient’s view<br />

of his or her physical defect?<br />

What is the recommended treatment<br />

for body dysmorphic disorder?<br />

Preoccupation with a defect in physical<br />

appearance<br />

No. In some patients the defect may be<br />

imagined, but in others an exaggeration<br />

of a true physical feature may be present.<br />

1. Perceptual: Perceptual relates to the<br />

accuracy of the individual’s body.<br />

2. Attitudinal: Attitudinal relates to the<br />

feelings the person has toward his or<br />

her body.<br />

Family history of a mood disorder or<br />

obsessive-compulsive disorder<br />

Often these patients present to<br />

dermatologist and plastic surgeons.<br />

Facial features<br />

Hair<br />

Body build<br />

Depression<br />

Excessive grooming<br />

Avoidance of mirrors<br />

Excessive exercise<br />

Avoidance of public activities<br />

Anorexia nervosa<br />

Gender identity disorder<br />

Narcissistic personality disorder<br />

No. These treatments tend to worsen<br />

the disorder, leading to intensified or<br />

new preoccupations with physical<br />

appearance.<br />

Antidepressants (such as selective<br />

serotonin reuptake inhibitors [SSRIs])<br />

(only if comorbid mental illnesses such<br />

as depression or anxiety are present)<br />

and cognitive behavioral therapy


122 Deja Review: <strong>Behavioral</strong> <strong>Science</strong><br />

FACTITIOUS DISORDER<br />

What is the primary characteristic<br />

of factitious disorder?<br />

Are patients with factitious disorder<br />

or malingering aware of the false<br />

nature of their somatic complaints?<br />

How is factitious disorder different<br />

from malingering?<br />

Which gender is more prevalent<br />

in factitious disorders?<br />

What occupation has the highest<br />

prevalence of factitious disorders?<br />

What disease presentations are often<br />

seen in factitious disorder?<br />

What laboratory finding would indicate<br />

self-injection of insulin?<br />

What are common behaviors seen in<br />

patients with factitious disorder?<br />

What are common histories of patients<br />

with factitious disorder?<br />

What is the appropriate treatment<br />

for factitious disorder?<br />

What is the prevalence of factitious<br />

disorder?<br />

Which gender is most commonly seen<br />

in factitious disorder?<br />

What is a well-known variant<br />

of factitious disorder?<br />

Intentional simulation of illness<br />

Yes. This is the distinguishing<br />

characteristic between factitious<br />

and somatoform disorders.<br />

The only obvious conscious goal of<br />

factitious disorder is to assume the “sick<br />

role”—there is no clear benefit or gain.<br />

Female<br />

Health-care workers<br />

Hematuria (from adding blood to urine<br />

or from the use of anticoagulants) and<br />

hypoglycemia (from insulin injection)<br />

Low C-peptide level<br />

Requests for analgesics<br />

Extensive knowledge of medical<br />

terminology<br />

Eager desire to undergo medical<br />

procedures and operations<br />

Traveling to different locations,<br />

hospitals, emergency rooms, etc<br />

Patients often have dramatic histories<br />

with extensive details about their<br />

symptoms.<br />

Recognition and confrontation in a<br />

nonaccusatory manner<br />

0.5% to 0.8%—very difficult to determine<br />

Females<br />

Factitious disorder by proxy (aka<br />

Munchausen syndrome by proxy)


Somatoform Disorders 123<br />

What is the primary characteristic<br />

of factitious disorder by proxy?<br />

Who is the most common<br />

perpetrator seen in factitious<br />

disorder by proxy?<br />

What is the treatment for factitious<br />

disorder by proxy?<br />

Intentional simulation of illness in<br />

another person<br />

A parent (most often a mother) often<br />

stimulating illness in her child<br />

Same as in factitious disorder, however,<br />

if a child is involved, the case should be<br />

managed as child abuse and reported to<br />

the appropriate agencies.<br />

MALINGERING<br />

What is the primary characteristic<br />

of malingering?<br />

How is malingering different from<br />

factitious disorder?<br />

What are common scenarios in which<br />

malingering is often seen?<br />

Which gender is most prevalent<br />

in malingering?<br />

What is Ganser syndrome?<br />

In what populations is Ganser<br />

syndrome most commonly seen?<br />

Intentional production of symptoms for<br />

secondary gain<br />

The goal of malingering is to obtain a<br />

concrete or material gain.<br />

Individuals wanting to avoid jail time<br />

or military recruitment, seeking financial<br />

compensation, etc<br />

Males<br />

A variant of malingering in which<br />

patients give ridiculous answers to<br />

questions in order to avoid<br />

responsibility for their actions<br />

Prison inmates<br />

CLINICAL VIGNETTES<br />

A 71-year-old man comes to your office for evaluation of a right foot paralysis. He<br />

complains that he is unable to move his right foot ever since he was involved in a<br />

car accident, during which he was driving and the teenaged passenger of the other<br />

vehicle was seriously injured. His neurologic examination is normal. What is the<br />

most likely diagnosis?<br />

Conversion disorder. Note the stressful event, normal neurologic examination,<br />

and the fact that it is his driving foot that is affected.


124 Deja Review: <strong>Behavioral</strong> <strong>Science</strong><br />

A 26-year-old female nurse is admitted after being found by a friend unconscious<br />

in the hallway. On finger stick, her glucose is 17. She has no history of diabetes.<br />

What psychiatric disorder might you suspect if medical workup is negative?<br />

Factitious disorder. Note conscious behavior (likely insulin overdose), but no<br />

obvious motivation/gain.<br />

What laboratory testing can you do, and how would it support your diagnosis?<br />

A C-peptide level would be useful, and if it was low you would suspect exogenous<br />

insulin overdose, therefore supporting your diagnosis.<br />

A 37-year-old man presents to the ED complaining of “10/10 right flank pain<br />

radiating to the testicle.” He specifically requests 2 mg of IV hydromorphone and<br />

states he is allergic to both NSAIDs and Tylenol. A nurse reports that she saw him<br />

putting drops of blood into his urine collection cup. What is the most likely<br />

diagnosis?<br />

Malingering<br />

A 12-year-old girl and her parents present to your office with a chief complaint of<br />

dysphagia for 3 days. According to the girl’s parents, who are very concerned and<br />

anxious, the girl choked while eating lunch 3 days ago and the Heimlich maneuver<br />

was performed to expel the food. Since then she has been unable to swallow any<br />

solids or liquids. Additionally, she is unable to swallow any of her normal oral<br />

secretions, and has been spitting constantly in a container. On examination, the<br />

girl is very quiet, calm, and somewhat unconcerned about her condition.<br />

Assuming physical examination is normal and an upper gastrointestinal (GI)<br />

series fails to reveal any pathology, what is the likely diagnosis?<br />

Conversion disorder


CHAPTER 17<br />

Personality Disorders<br />

What is personality?<br />

What constitutes a personality<br />

disorder?<br />

What diagnostic criteria are common<br />

to all personality disorders?<br />

Which axis of the DSM-IV-TR<br />

do personality disorder<br />

diagnoses fall under?<br />

How are the personality disorders<br />

divided?<br />

Which personality disorders fall<br />

under cluster A?<br />

What is the common theme among<br />

cluster A disorders?<br />

Which personality disorders fall under<br />

cluster B?<br />

What is the common theme among<br />

cluster B disorders?<br />

Personality is “the set of characteristics<br />

that defines the behavior, thoughts, and<br />

emotions of individuals,” and is<br />

persistent over time.<br />

Personality disorders occur when<br />

a particular feature or trait of an<br />

individual’s personality becomes<br />

inflexible or maladaptive and impairs<br />

social, occupational, or personal<br />

functioning.<br />

The patterns are pervasive and<br />

begin by early adulthood, lead to<br />

significant distress or impairment,<br />

and are not better explained by an<br />

Axis I disorder.<br />

They are Axis II diagnoses.<br />

Into clusters—A, B, and C<br />

Paranoid, schizoid, and schizotypal<br />

They are considered odd or eccentric<br />

traits.<br />

Histrionic, narcissistic, antisocial, and<br />

borderline<br />

They are described as dramatic,<br />

emotional, and erratic.<br />

125


126 Deja Review: <strong>Behavioral</strong> <strong>Science</strong><br />

Which personality disorders fall under<br />

cluster C?<br />

What is the common theme among<br />

cluster C disorders?<br />

What is a good way to remember these<br />

clusters?<br />

Avoidant, dependent, and<br />

obsessive-compulsive<br />

They tend to be anxious and fearful.<br />

They are Mad, Bad, and Sad.<br />

A = Mad, B = Bad, C = Sad.<br />

CLUSTER A: THE MAD<br />

How is schizoid personality disorder<br />

described?<br />

What are the symptoms/criteria for<br />

schizoid personality disorder ?<br />

How is paranoid personality disorder<br />

defined?<br />

What are the criteria/symptoms<br />

of paranoid personality disorder?<br />

Patients have “a pervasive pattern of<br />

detachment from social relationships<br />

and a restricted range of expression of<br />

emotions in interpersonal settings.”<br />

Four or more of the following:<br />

No desire or enjoyment of close<br />

relationships<br />

Choice of solitary activities<br />

Little interest in having sexual experiences<br />

Enjoyment of few activities<br />

Lack of close friends<br />

Apparent indifference<br />

Emotional coldness/detachment/<br />

flattened affect<br />

Paranoid personality disorder involves<br />

a “pervasive and unwarranted suspicion<br />

and mistrust of people, hypersensitivity<br />

to others, and an inability to deal with<br />

feelings.”<br />

Four or more of the following:<br />

Suspicion of exploitation or<br />

deceitfulness on the part of others<br />

Preoccupation with unjustified doubts<br />

Reluctance to confide in others<br />

Reading hidden demeanings or<br />

threatening meanings into benign<br />

remarks or events<br />

Persistently bearing grudges<br />

Perception of attacks on his or her<br />

character or reputation to which he<br />

or she reacts quickly/angrily<br />

Recurrent suspicions


Personality Disorders 127<br />

What is the best treatment for paranoid<br />

personality disorder?<br />

How is schizotypal personality disorder<br />

defined?<br />

What are the symptoms of schizotypal<br />

personality disorder?<br />

What percentage of patients with<br />

Schizotypal Personality Disorder<br />

go on to develop schizophrenia?<br />

Psychotherapy and possibly<br />

antipsychotic medications to manage<br />

agitation and paranoia (overt delusions<br />

are not usually seen)<br />

Patients with schizotypal personality<br />

disorder are usually described as strange<br />

or odd in behavior, appearance, and/or<br />

thinking. However, they are neither<br />

frankly delusional nor psychotic.<br />

Ideas of reference<br />

Odd beliefs/magical thinking/believe<br />

that they have “special powers”<br />

Unusual perceptual experiences<br />

Odd thinking and speech<br />

Suspiciousness/paranoia<br />

Inappropriate/constricted affect<br />

Odd behavior/appearance<br />

Lack of close friends<br />

Excessive social anxiety<br />

10% to 20%—ie, most do not<br />

CLUSTER B: THE BAD<br />

Which personality disorders fall under<br />

cluster B?<br />

What is the common theme among<br />

cluster B disorders?<br />

What other psychiatric diagnoses<br />

do patients with cluster B disorders<br />

often carry?<br />

How are patients with antisocial<br />

personality disorder usually described?<br />

What prior childhood diagnosis must<br />

have been present in order to be<br />

diagnosed with antisocial personality<br />

disorder as an adult?<br />

Histrionic, narcissistic, antisocial,<br />

and borderline<br />

They are described as dramatic,<br />

emotional, and erratic.<br />

Mood disorders are quite prevalent,<br />

as well as somatization disorders,<br />

and substance abuse/dependence<br />

Charming, but manipulative. They often<br />

have a history of criminal activities and<br />

many have a history of substance abuse.<br />

They show no remorse for their actions,<br />

even if others are harmed.<br />

Conduct disorder


128 Deja Review: <strong>Behavioral</strong> <strong>Science</strong><br />

What are the diagnostic criteria for<br />

antisocial personality disorder?<br />

What is the best treatment of antisocial<br />

personality disorder?<br />

How is borderline personality<br />

disorder defined?<br />

What defense mechanism is prevalent<br />

in patients with borderline personality<br />

disorder?<br />

What are the symptoms/diagnostic<br />

criteria of borderline personality<br />

disorder?<br />

Three or more of the following:<br />

Failure to conform to social norms and<br />

laws<br />

Deceitfulness<br />

Impulsivity<br />

Irritability and aggressiveness<br />

Disregard for the safety of self<br />

or others<br />

Irresponsibility (often unemployment)<br />

Lack of remorse for harm to others<br />

Group therapy with setting of<br />

boundaries for behavior, selective<br />

serotonin reuptake inhibitors<br />

(SSRIs), and treatment of underlying<br />

substance abuse/misuse<br />

if present.<br />

These individuals are very refractory<br />

to treatment.<br />

Borderline personality disorder involves<br />

the primary feature of instability. This is<br />

seen in terms of the patient’s self-image,<br />

interpersonal relationships, and mood.<br />

(Think Angelina Jolie’s character in<br />

Girl Interrupted.)<br />

Splitting—eg, people are either all good<br />

or all bad.<br />

Five or more of the following:<br />

Frantic efforts to avoid real or imagined<br />

abandonment<br />

Unstable or intense interpersonal<br />

relationships<br />

Identity disturbance, impulsivity<br />

Recurrent suicidal or self-mutilating<br />

behavior<br />

Marked reactivity of mood<br />

Chronic feelings of emptiness<br />

Inappropriate/intense anger<br />

Lack of control of anger<br />

Transient stress-related paranoid<br />

ideation


Personality Disorders 129<br />

What are the most prevalent symptoms<br />

of borderline personality disorder?<br />

What developmental characteristics<br />

are often present in patients<br />

with borderline personality<br />

disorder?<br />

What is the best treatment for borderline<br />

personality disorder?<br />

What drug class should be avoided in<br />

patients with borderline personality<br />

disorder?<br />

How is narcissistic personality disorder<br />

described?<br />

What are the symptoms/diagnostic<br />

criteria of narcissistic personality<br />

disorder?<br />

What other psychiatric conditions<br />

are associated with narcissistic<br />

personality disorder?<br />

Unstable relationships and mood/<br />

affect lability<br />

Many were severely abused as children.<br />

Psychotherapy and mood stabilization<br />

with either antidepressants,<br />

carbamazepine, or valproate.<br />

Patients may also require shortterm<br />

antipsychotics for treatment<br />

of psychosis.<br />

Benzodiazepines, because of addictive<br />

potential and with overdose<br />

Narcissistic personality disorder is<br />

defined as a grandiose sense of selfimportance<br />

along with extreme<br />

sensitivity to criticism. These patients<br />

have “little ability to sympathize with<br />

others, and are more concerned about<br />

appearance than substance.” (Think Bill<br />

Murray’s character in Groundhog Day.)<br />

Five or more of the following:<br />

Grandiose sense of self-importance/<br />

exaggeration of achievements and<br />

talents<br />

Preoccupations with ideals of success,<br />

power, brilliance, beauty, or love<br />

Belief that he or she is special or unique,<br />

and can only be understood or<br />

appreciated by high-status individuals<br />

Sense of entitlement<br />

Lacks empathy for others<br />

Manipulative or exploitive of others<br />

Requires excessive admiration<br />

Believes others envy him or her,<br />

or envies others<br />

Arrogant<br />

Mood disorders and other cluster<br />

B traits


130 Deja Review: <strong>Behavioral</strong> <strong>Science</strong><br />

What is the best treatment of<br />

narcissistic personality disorder?<br />

How is histrionic personality disorder<br />

described?<br />

What are the symptoms/diagnostic<br />

criteria of histrionic personality<br />

disorder?<br />

What other psychiatric disorders are<br />

often associated with histrionic<br />

personality disorder?<br />

What is the best treatment of histrionic<br />

personality disorder?<br />

Psychotherapy (either group or<br />

individual)<br />

Histrionic personality disorder can be<br />

identified by the flamboyant/attentionseeking<br />

behaviors of patients. They are<br />

extremely emotional, and may present<br />

as very attractive and seductive.<br />

(Think Scarlett O’Hara in Gone with<br />

the Wind.)<br />

Five or more of the following:<br />

Persistent need to be the center of<br />

attention<br />

Inappropriately sexual/seductive/<br />

provocative<br />

Rapidly shifting, shallow emotions<br />

Believes relationships are more intimate<br />

than they actually are<br />

Impressionistic speech lacking detail<br />

Melodramatic<br />

Uses physical appearance to attract<br />

attention<br />

Suggestible/easy to manipulate<br />

Mood disorders and somatization<br />

disorders<br />

Psychotherapy and antidepressants for<br />

underlying mood disorders<br />

CLUSTER C: THE SAD<br />

How is avoidant personality disorder<br />

often described?<br />

What other personality disorder may<br />

avoidant personality disorder be<br />

mistaken for?<br />

Individuals with avoidant personality<br />

disorder are often shy and timid.<br />

They are very self-critical, have low<br />

self-esteem, and are preoccupied<br />

with fears of rejection or<br />

embarrassment.<br />

Schizoid personality disorder


Personality Disorders 131<br />

How can avoidant personality disorder<br />

and schizoid personality disorder be<br />

differentiated?<br />

What are the symptoms/diagnostic<br />

criteria of avoidant personality<br />

disorder?<br />

What other psychiatric conditions<br />

are often seen in patients with avoidant<br />

personality disorder?<br />

What is the best treatment of avoidant<br />

personality disorder?<br />

How is dependent personality disorder<br />

described?<br />

What are the symptoms/diagnostic<br />

criteria of dependent personality<br />

disorder?<br />

Patients with avoidant personality<br />

disorder want to have interpersonal<br />

relationships but are afraid of rejection;<br />

whereas, schizoid personality disorder<br />

patients do not wish to have<br />

relationships with others.<br />

Four or more of the following:<br />

Avoidance of occupations that involve<br />

interaction with others.<br />

Fear of intimacy/lack of intimate<br />

relationships for fear of ridicule.<br />

Unwilling to be involved with people<br />

unless certain of being liked.<br />

Preoccupation with criticism or<br />

rejection.<br />

Believes they are socially inept and<br />

inferior to others.<br />

Feelings of inadequacy inhibit social<br />

involvement.<br />

Reluctance to become involved in new<br />

activities.<br />

Social phobia, specific phobia, and<br />

agoraphobia<br />

Psychotherapy and assertiveness<br />

training, SSRIs, Beta-Blockers<br />

Patients are passive and may let others<br />

direct their lives and make important<br />

decisions. (Think Bill Murray’s<br />

character in What about Bob?)<br />

Five or more of the following:<br />

Inability to make decisions without<br />

advice<br />

Refusal to assume responsibility<br />

Has difficulty expressing disagreement<br />

Difficulty initiating projects<br />

Need for excessive nurturing and<br />

support<br />

Feelings of discomfort and helplessness<br />

when alone<br />

Persistent need to be in a relationship<br />

Unrealistic fears of being left alone


132 Deja Review: <strong>Behavioral</strong> <strong>Science</strong><br />

What other psychiatric disorders are<br />

common in individuals with dependent<br />

personality disorder?<br />

What is the best treatment for dependent<br />

personality disorder?<br />

How is obsessive-compulsive<br />

personality disorder described?<br />

How does obsessive-compulsive<br />

personality disorder (OCPD) differ<br />

from obsessive-compulsive disorder<br />

(OCD)?<br />

What are the symptoms/diagnostic<br />

criteria of obsessive-compulsive<br />

personality disorder?<br />

What is the best treatment for<br />

obsessive-compulsive personality<br />

disorder?<br />

Depression and anxiety disorders<br />

Psychotherapy and assertiveness<br />

training, treatment of comorbid<br />

conditions<br />

Individuals have extreme perfectionist<br />

tendencies and inflexibility, a<br />

preoccupation with orderliness<br />

and control.<br />

Individuals with obsessive-compulsive<br />

personality disorder do not have<br />

intrusive thoughts (obsessions) or<br />

actions that they carry out to relieve<br />

the anxiety provoked by those thoughts<br />

(compulsions).<br />

They also do not believe they have a<br />

problem; patients with OCD know their<br />

actions are irrational, whereas OCPD<br />

patients are more likely to think<br />

everyone else has the problem.<br />

Four of more of the following:<br />

Preoccupation with rules/details/<br />

organizations—often such that the point<br />

of the activity is lost<br />

Perfectionism<br />

Excessive devotion to work and<br />

productivity<br />

Inflexible about moral or ethical<br />

values<br />

Will not discard unneeded objects<br />

(Pack rats)<br />

Reluctant to delegate to others<br />

Cheap/frugal in order to hoard<br />

money<br />

Rigidity and stubbornness<br />

Psychotherapy


Personality Disorders 133<br />

CLINICAL VIGNETTES<br />

Your patient is an excellent medical student. He always studies for 6 hours a night,<br />

every night, and at the same time. He checks his orders three times before he turns<br />

them in. He does everything himself because he is convinced no one can do as<br />

good of a job as him. He keeps every note he’s ever written about a patient. No one<br />

else in the class likes him, but he is convinced that’s just because they are jealous<br />

of him—he is just fine. He does not do well and is up for dismissal from medical<br />

school. What is his most likely diagnosis?<br />

Obsessive-compulsive personality disorder<br />

A young man comes into your office complaining of lack of social relationships.<br />

He works alone as a night security guard. He got invited to a work party once, but<br />

was too afraid to go. He would like to make friends, but feels he isn’t worthy of<br />

socializing. He is sure they would only make fun of him anyway. What is the most<br />

likely diagnosis?<br />

Avoidant personality disorder


This page intentionally left blank


CHAPTER 18<br />

Dissociative Disorders<br />

What are the primary characteristics<br />

of dissociative disorders?<br />

What are the four major dissociative<br />

disorders?<br />

Which conditions are included in the<br />

differential diagnosis of dissociative<br />

disorders?<br />

Which disorder is associated with<br />

an inability to remember important<br />

personal information?<br />

Which group of people is most<br />

likely to suffer from dissociative<br />

amnesia?<br />

What is the primary trigger for<br />

dissociative amnesia?<br />

What treatment modality is used<br />

for dissociative amnesia?<br />

What disorder is associated with an<br />

inability to remember important<br />

personal information and wandering<br />

away from home to adopt a new<br />

identity?<br />

Sudden memory loss of time periods,<br />

events, and people<br />

Detachment from one’s self<br />

Derealization<br />

Blurred sense of identity<br />

1. Dissociative amnesia<br />

2. Dissociative fugue<br />

3. Dissociative identity disorder<br />

4. Depersonalization disorder<br />

Substance abuse<br />

Seizure disorders<br />

Head injury<br />

Posttraumatic stress disorder<br />

Malingering<br />

Dissociative amnesia<br />

Young adult females<br />

Often follows a psychologically<br />

traumatic event<br />

Psychotherapy<br />

Dissociative fugue<br />

135


136 Deja Review: <strong>Behavioral</strong> <strong>Science</strong><br />

How long does it normally take for<br />

the amnesia to resolve in a person<br />

who is experiencing dissociative<br />

amnesia or dissociative fugue?<br />

What treatment modality is used<br />

for dissociative fugue?<br />

Which controversial disorder is<br />

associated with a person having<br />

“multiple personalities”?<br />

What are the DSM-IV-TR criteria<br />

for Dissociative Identity Disorder?<br />

Which gender is most likely<br />

to develop dissociative identity<br />

disorder?<br />

What conditions may dissociative<br />

identity disorder resemble?<br />

What treatment modality is used<br />

for dissociative identity disorder?<br />

Which disorder is associated with<br />

repeated episodes of detachment<br />

and unreality about one’s own body,<br />

social situation, or the environment<br />

(derealization)?<br />

What treatment modality is used<br />

for depersonalization disorder?<br />

Minutes or days; may last for years<br />

Supportive psychotherapy<br />

Hypnosis<br />

Dissociative identity disorder<br />

Presence of two or more distinct<br />

identities or personality states<br />

At least two of these identities take<br />

control of the persons behavior<br />

Inability to recall personal information<br />

Not due to substance use or a general<br />

medical condition<br />

Female<br />

Borderline personality disorder<br />

Schizophrenia<br />

Psychotherapy and hypnotherapy<br />

Depersonalization disorder<br />

Psychotherapy; pharmacologic<br />

interventions utilized for associated<br />

anxiety or depression<br />

CLINICAL VIGNETTES<br />

A war veteran claims he cannot remember the beach assault during which his best<br />

friend was killed. He denies any substance use. What dissociative disorder may<br />

this be?<br />

Dissociative amnesia (Note: This could also be “Post-Traumatic Stress Disorder.”)


Dissociative Disorders 137<br />

A 42-year-old stock broker fails to return home from work one day. Two years later<br />

he is found living in a nearby small town and working as a mechanic at a local<br />

shop. He does not recall any of the past events of his life and denies having ever<br />

been a stock broker. What state is this called?<br />

Dissociative fugue (Think Jason Bourne in the movie/book The Bourne Identity.)<br />

A woman often feels as though she is floating outside of her body, watching her<br />

actions from above. It has started to interfere with her ability to stay employed.<br />

What is the most likely diagnosis?<br />

Depersonalization disorder.<br />

What about if this feeling had only happened one time during a time of extreme<br />

danger?<br />

If it had only happened once, this diagnosis cannot be made—dissociative symptoms<br />

are common in life-threatening situations.<br />

A 52-year-old man convicted for murder claims that he does not remember his<br />

crimes. He has been witnessed to apparently be “possessed” by other<br />

personalities, during which he does not remember important information about<br />

himself. What is a possible psychiatric diagnosis and what is on the differential?<br />

Possibly (though very unlikely) dissociative identity disorder. More likely this could<br />

be due to substance use or malingering.


This page intentionally left blank


CHAPTER 19<br />

Substance Abuse<br />

Disorders<br />

What are the DSM-IV-TR diagnostic<br />

criteria for substance dependence?<br />

How is tolerance defined?<br />

How is withdrawal defined?<br />

What are the DSM-IV-TR diagnostic<br />

criteria for substance abuse?<br />

139<br />

Three or more of the following:<br />

Tolerance.<br />

Withdrawal.<br />

More substance taken than was<br />

intended.<br />

Failure to control use, or a desire to<br />

stop using.<br />

Much time is spent either obtaining<br />

the substance, using the substance,<br />

or recovering from its effects.<br />

Social or occupational activities are<br />

reduced because of use.<br />

Use continues despite physical or<br />

psychological impacts of use.<br />

Either a need for increased amounts of<br />

substance to achieve the same effect or<br />

diminished effect with use of the same<br />

amount<br />

A withdrawal syndrome characteristic<br />

of the drug or the same or similar<br />

substance is taken to avoid that<br />

withdrawal syndrome.<br />

Does not meet substance dependence<br />

criteria and one of following:<br />

Substance use results in failure at<br />

work, school, or home.<br />

Recurrent use in hazardous situations<br />

(eg, driving).<br />

Recurrent substance-related legal<br />

problems.<br />

Use despite social or interpersonal<br />

problems made worse by use.


140 Deja Review: <strong>Behavioral</strong> <strong>Science</strong><br />

What pathway is involved in the<br />

chemical rewards of drug use?<br />

The “dopamine reward pathway” is<br />

thought to play a large role—it projects<br />

from the ventral tegmental area (VTA) to<br />

the nucleus accumbens.<br />

What factors affect how quickly and • Route of administration: The faster<br />

to what magnitude the chemical<br />

through the blood-brain barrier,<br />

rewards are felt after ingestion<br />

the greater the euphoria and higher<br />

of a drug? likelihood of addiction (eg, IV ><br />

smoking > oral [pills])<br />

• Chemical composition of the drug<br />

(increasing purity → increased and<br />

faster effects)<br />

• Genetic differences between people<br />

(relates to receptor stimulation)<br />

• Associated stimuli (ie, drug<br />

paraphernalia, other conditional<br />

stimuli)<br />

What are the three important<br />

observations about withdrawal<br />

that Himmelsbach made?<br />

What are the four central tenets<br />

that must be addressed in order<br />

to successfully treat drug dependence?<br />

How should withdrawal symptoms<br />

be prevented/treated during<br />

detoxification?<br />

1. There is a common association<br />

between tolerance and a specific<br />

withdrawal syndrome.<br />

2. The nature of the withdrawal<br />

syndrome is opposite to the<br />

acute effects of the drug.<br />

3. The withdrawal syndrome is most<br />

intense when the drug leaves the<br />

brain rapidly.<br />

1. The positive reinforcements/reward<br />

effects of the drug must be reduced.<br />

2. The negative reinforcements<br />

(withdrawal symptoms) must be<br />

treated, either by giving a substitute<br />

drug or by symptomatically treating<br />

the effects of drug removal.<br />

3. Detoxification—complete removal<br />

of the drug of dependence from the<br />

patient’s system.<br />

4. Relapse prevention by reducing the<br />

desire for the drug or by reducing<br />

cravings for the drug.<br />

Withdrawal symptoms may be<br />

prevented/treated by administering<br />

a substitute drug with a similar effect<br />

(eg, methadone for heroin,<br />

benzodiazepines for EtOH), or treating<br />

the withdrawal symptomatically<br />

(eg, treating diarrhea and<br />

GI symptoms).


Substance Abuse Disorders 141<br />

How can the positive reinforcement<br />

effects of drugs be reduced?<br />

How does disulfiram work?<br />

What side effect of opiate antagonists<br />

may result in compliance problems?<br />

What are the three major ways<br />

of reducing withdrawal?<br />

What criteria should be used when<br />

selecting a substitute drug for treating<br />

withdrawal?<br />

Giving specific receptor antagonists<br />

to prevent the binding of receptors<br />

by the drug of dependence; therefore,<br />

precipitating withdrawal and preventing<br />

the effects of the drug (eg, naloxone or<br />

naltrexone treatment for opiates)<br />

Converting reward to punishment (eg,<br />

the use of disulfiram [Antabuse] for<br />

alcohol dependence)<br />

Giving dopamine or opiate antagonists<br />

to cause general inhibition of the<br />

reward pathways<br />

Negative discriminative stimuli (eg, telling<br />

the patients that their drug of choice<br />

will be ineffective; therefore, they avoid it)<br />

Disulfiram inhibits aldehyde<br />

dehydrogenase in the liver. Alcohol is<br />

then unable to be fully metabolized,<br />

leading to flushing, headache, and<br />

nausea from accumulation of the<br />

aldehyde intermediate.<br />

Anhedonia<br />

1. Substitute drugs that act as agonists<br />

for the same receptor, and therefore,<br />

prevent severe withdrawal (ie, using<br />

methadone to treat heroin or opiate<br />

dependence)<br />

2. Substitution by partial agonist for the<br />

same receptor, thereby, preventing<br />

severe withdrawal and counteracting<br />

the effects of the drug if it is taken<br />

3. Substitution of a different route of<br />

administration to prevent some of<br />

the adverse effects of the drug itself<br />

while still preventing withdrawal (ie,<br />

using the nicotine patch for smoking<br />

cessation)<br />

Substitute drugs should be:<br />

Less rewarding<br />

Less damaging<br />

More manageable<br />

Able to allow the patient to be more<br />

functional (eg, methadone)


142 Deja Review: <strong>Behavioral</strong> <strong>Science</strong><br />

What are the major problems with<br />

using positive and negative<br />

reinforcement treatments?<br />

What is the basic principle behind<br />

detoxification?<br />

What percentage of patients who<br />

become drug-free relapse?<br />

What modalities have been associated<br />

with decreased incidence of relapse?<br />

What is the most likely reason<br />

for relapse?<br />

How can cravings be treated to prevent<br />

relapse?<br />

What are some of the medical effects<br />

of drug abuse?<br />

To use positive reinforcement<br />

treatments, the patient must undergo<br />

detoxification first.<br />

Negative reinforcement treatments<br />

may lead to polydrug abuse,<br />

especially if substitution methods<br />

are used.<br />

Detoxification uses the principle of<br />

substitution with a drug of crossdependence<br />

or different route of<br />

administration to allow for safer,<br />

slower withdrawal from the drug<br />

of dependence. It should not be<br />

used to precipitate withdrawal.<br />

75%—most within the first year<br />

Joining self-help groups → providing<br />

a different type of peer pressure and<br />

reinforcement<br />

Cognitive therapy → helps develop<br />

new and different coping skills<br />

Cravings is the most common reason<br />

for relapse. Cravings are caused by<br />

memories of the positive rewards of<br />

drug use or by conditioning cues that<br />

are endogenous or exogenous.<br />

Cravings may be treated by reducing<br />

the desire for the drug, providing a<br />

substitute for the drug reward, reducing<br />

endogenous cues for cravings (ie, by<br />

using anxiolytics or antidepressants),<br />

reducing the conditional anticipation<br />

of the reward (ie, giving naltrexone<br />

to an alcoholic), or by reducing<br />

pseudowithdrawal symptoms.<br />

Increased risk of lung disease<br />

and cancer<br />

Increased risk of human<br />

immunodeficiency virus (HIV),<br />

hepatitis, and other infections with<br />

intravenous (IV) drug use<br />

Acute and chronic toxicities


Substance Abuse Disorders 143<br />

What are some of the nonmedical<br />

consequences of drug abuse?<br />

What are the acute symptoms of ethanol<br />

toxicity?<br />

What are the chronic symptoms<br />

of ethanol toxicity?<br />

What are the clinical manifestations<br />

of fetal alcohol syndrome?<br />

What is the most common form<br />

of drug dependence?<br />

What are the deleterious effects<br />

of caffeine?<br />

Sociological problems such as violence,<br />

crime, and poverty<br />

Acute impairment leading to reduced<br />

cognition or restraint<br />

Other risky behaviors (eg, increased<br />

likelihood of sexual violence)<br />

Vomiting (with risk of possible<br />

aspiration due to decreased mental<br />

status)<br />

Respiratory depression<br />

Coma<br />

Death<br />

Psychiatric symptoms (depression,<br />

hallucinations)<br />

Neurological signs (dementia, vascular<br />

problems, and neuropathies)<br />

GI tract malfunction (cirrhosis of the<br />

liver, pancreatitis, and GI cancer)<br />

Cardiovascular disease (cardiomyopathy,<br />

hypertension)<br />

Pre- and postnatal retardation of growth<br />

and cognition<br />

Facial abnormalities (short palpebral<br />

fissures, thin vermillion border of the<br />

upper lip, smooth philtrum, and<br />

flattened midface)<br />

Central nervous system (CNS)<br />

damage<br />

Attention deficits<br />

Tendency for risk-taking behaviors, such<br />

as substance abuse<br />

Caffeine dependence, which is seen in<br />

60% to 70% of the population<br />

In acute intoxication there can be<br />

cardiovascular side effects<br />

(palpitation, arrhythmia, increase<br />

in BP). Caffeine withdrawal can<br />

cause headache.<br />

However, there is little to no evidence<br />

of adverse effects with chronic, casual<br />

caffeine use.


144 Deja Review: <strong>Behavioral</strong> <strong>Science</strong><br />

What is the mechanism of action<br />

of cocaine?<br />

What are the physiologic effects<br />

of cocaine use?<br />

What is the most likely mechanism<br />

of psychosis in chronic cocaine users?<br />

What is the mechanism of action<br />

of amphetamines?<br />

What are the acute and chronic effects<br />

of amphetamine use?<br />

What is the mechanism of action<br />

of dissociative anesthetic drugs like<br />

ketamine and phencyclidine?<br />

What are the effects of dissociative<br />

anesthetic use?<br />

What is the mechanism of action<br />

of marijuana?<br />

Cocaine is a stimulant that works by<br />

preventing the reuptake of catecholamine<br />

transmitters, such as dopamine and<br />

norepinephrine, in the brain and<br />

autonomic nervous system.<br />

Vasoconstriction<br />

Tachycardia<br />

Hyperthermia<br />

Hypertension<br />

Cardiac dysrhythmias<br />

Stroke<br />

Psychosis<br />

Dopamine potentiation<br />

Amphetamines block both the reuptake<br />

of dopamine and norephinephrine at<br />

the synapse, as well as cause the release<br />

of stored catecholamines.<br />

Vasoconstriction<br />

Tachycardia<br />

Hyperthermia<br />

Hypertension<br />

Cardiac dysrhythmias<br />

Stroke<br />

Psychosis<br />

Neurotoxicity<br />

Dissociative anesthetics work by<br />

blocking N-methyl-D-aspartate (NMDA)<br />

receptors and sigma receptors in the CNS.<br />

Amnesia<br />

Confusion<br />

Delusions and hallucinations<br />

Violent behavior<br />

Hyperthermia<br />

Marijuana is a tetrahydrocannabinoid<br />

that works on cannabinoid receptors, a<br />

member of G protein–linked receptors.<br />

These affect monoamine and<br />

γ-aminobutyric acid (GABA) neurons<br />

in the basal ganglia, hippocampus, and<br />

cerebellum.


Substance Abuse Disorders 145<br />

What are the chronic toxicities<br />

associated with marijuana use?<br />

Where are the opioid receptors located<br />

in the brain and what is the function<br />

of each receptor?<br />

What are the three classic types<br />

of opioid receptors?<br />

What receptors do most clinically used<br />

opiates such as morphine work on?<br />

What endogenous peptide transmitters<br />

work at opioid receptor and which<br />

receptors do they act on?<br />

Describe the mechanism of action<br />

of opioids at their receptors:<br />

How do strong agonists cause<br />

the effects of opioids?<br />

How do antagonists work?<br />

Poor memory and motivation<br />

Testosterone suppression, gynecomastia<br />

Chronic obstructive pulmonary disease<br />

(COPD)<br />

Immunosuppression<br />

Low fetal birth weight<br />

Periaqueductal grey matter →<br />

responsible for analgesia<br />

Area postrema → responsible for nausea<br />

and vomiting<br />

Ventral medulla → responsible for<br />

respiratory depression<br />

Edinger-Westphal nucleus →<br />

responsible for the pinpoint pupil<br />

response (due to extreme miosis)<br />

Nucleus accumbens → responsible for<br />

euphoria<br />

1. Mu<br />

2. Kappa<br />

3. Delta<br />

Mu<br />

Enkephalins act on mu and delta<br />

receptors, main actions are at delta<br />

receptors.<br />

Beta-endorphins act on mu and delta<br />

receptors equally.<br />

Dynorphins act on kappa receptors.<br />

Opioid receptors are G protein–coupled<br />

receptors with seven membranespanning<br />

segments. Activation of the<br />

receptors causes changes in cyclic<br />

adenosine monophosphate (cAMP),<br />

Ca 2+ , etc, leading to inhibition of<br />

neuronal excitation.<br />

Strong agonists have high affinity<br />

for the receptor and produce a<br />

conformational change that activates<br />

the receptor.<br />

Antagonists have high affinity for the<br />

receptor and do not activate the receptor.


146 Deja Review: <strong>Behavioral</strong> <strong>Science</strong><br />

What is the difference between strong<br />

and partial agonists?<br />

What is the difference between opiates<br />

and opioids?<br />

How do kappa and mu receptors differ?<br />

Which opioids/opiates are strong<br />

mu receptor agonists?<br />

Which opioids/opiates are partial<br />

mu receptor agonists?<br />

What relative of morphine without<br />

sedative effect is used to suppress<br />

cough?<br />

Which opioids/opiates are mu receptor<br />

antagonists?<br />

Which opioids/opiates are kappa<br />

receptor antagonists?<br />

What drug can be used as a narcotic<br />

reversal agent?<br />

Which opioids/opiates are metabolized<br />

by CYP2D6?<br />

Both strong and partial agonists have<br />

high affinity for the receptor; but unlike<br />

strong agonists, partial agonists have<br />

low efficacy to activate the receptor<br />

resulting in a weak effect. Because of<br />

this high affinity/low efficacy, they can<br />

antagonize/block the effects of a<br />

stronger agonist.<br />

Opiates are derived from the<br />

opium poppy, while opioids<br />

are synthetic or semisynthetic<br />

derivatives of opiates.<br />

The difference between kappa receptors<br />

and mu receptors is that activation of<br />

kappa receptors produces less analgesia,<br />

less respiratory depression/asphyxia<br />

(floor effect), and produce dysphoria<br />

instead of euphoria.<br />

Morphine<br />

Fentanyl<br />

Etorphine<br />

Heroin<br />

Hydromorphone<br />

Oxycodone<br />

Meperidine<br />

Buprenorphine<br />

Pentazocine<br />

Dextromethorphan<br />

Nalorphine<br />

Naltrexone (nonnarcotic)<br />

Buprenorphine<br />

Naltrexone (nonnarcotic)<br />

Naltrexone/Naloxone (aka Narcan)<br />

Oxycodone<br />

Codeine


Substance Abuse Disorders 147<br />

What is the significance of being<br />

metabolized by CYP2D6?<br />

Which opioid receptor agonist does not<br />

work systemically and can be used as<br />

an antidiarrheal agent?<br />

What is the clinical indication for<br />

methadone?<br />

What are the symptoms of narcotic<br />

overdose?<br />

What is the route of absorption<br />

of ethanol in the body?<br />

What is the limiting factor in ethanol<br />

absorption?<br />

What is the effect of ethanol’s water<br />

solubility?<br />

What is the mechanism of action<br />

of ethanol?<br />

What is the physiologic effect of<br />

ethanol’s action on GABA receptors?<br />

What is the physiologic effect of<br />

ethanol’s action on glutamate receptors?<br />

In drugs that are metabolized by<br />

CYP2D6, the active metabolite is<br />

morphine.<br />

Loperamide, which works by reducing<br />

GI tract motility<br />

Methadone is used to treat withdrawal<br />

from heroin while acting to decrease<br />

cravings and risk from associated<br />

lifestyle (legal, HIV, hepatitis, etc).<br />

In narcotic overdoses, CO 2 drive is<br />

reduced resulting in respiratory<br />

depression and cyanosis and<br />

diminished mental status/coma.<br />

The very low molecular weight as<br />

well as its water and lipid solubility<br />

allow for rapid absorption of ethanol<br />

from the GI tract and entry into the<br />

brain.<br />

Absorption of ethanol is limited only<br />

by the surface area of the stomach.<br />

Its water solubility allows absorbed<br />

ethanol to be distributed throughout<br />

the body water.<br />

Ethanol’s mechanism is not completely<br />

known, but likely has to do with<br />

alteration of membrane fluidity as<br />

well as actions on GABA receptors,<br />

glutamate NMDA receptors, and<br />

the serotonin system.<br />

GABA is the major inhibitory<br />

transmitter in the brain, and ethanol<br />

potentiation of GABA causes anxiolysis<br />

and possibly the reward effects by<br />

increasing dopamine release from<br />

the nucleus accumbens.<br />

Glutamate is a major excitatory<br />

transmitter in the brain, and ethanol<br />

inhibits glutamate causing amnesia and<br />

anesthetic effects and possibly reward<br />

effects.


148 Deja Review: <strong>Behavioral</strong> <strong>Science</strong><br />

What is a useful and evidence-based<br />

way to screen for alcohol abuse?<br />

CAGE questions:<br />

Ever felt the need to Cut down your<br />

drinking?<br />

Have you felt Annoyed by criticism<br />

of your drinking?<br />

Ever felt Guilty about your drinking?<br />

Ever taken an Eye opener (a drink first<br />

thing in the morning)?<br />

How many drinks are needed for Four standard drinks = a BAC of 0.08%<br />

an 180-lb man to reach the legal<br />

limit (0.08% blood alcohol content<br />

[BAC])?<br />

How many drinks are needed for an Three standard drinks = a BAC of 0.10%<br />

140-lb woman to reach the legal limit (over the legal limit)<br />

(0.08% BAC)?<br />

How is ethanol eliminated from<br />

the body?<br />

How is ethanol metabolized in<br />

the liver?<br />

What is zero-order elimination<br />

of a drug?<br />

Ninety-five percent of the ethanol<br />

ingested is metabolized in the liver.<br />

Alcohol metabolism is a zero-order<br />

process in which alcohol is converted<br />

by alcohol dehydrogenase to aldehydes,<br />

and then the aldehydes are converted<br />

by aldehyde dehydrogenase<br />

to acetate.<br />

Zero-order elimination occurs when a<br />

constant amount of drug is eliminated<br />

from the body per unit of time,<br />

regardless of the drug dose or<br />

serum concentration.<br />

Concentration of plasma<br />

Time (h)<br />

Figure 19.1 Zero-order elimination. (Drug has a constant elimination per unit time, regardless of<br />

concentration.)


Substance Abuse Disorders 149<br />

Serum drug concentration<br />

Time (h)<br />

Figure 19.2 First-Order Elimination. (Drug elimination is concentration dependent—more is<br />

eliminated as serum concentration is raised.)<br />

What is the limiting factor in ethanol<br />

metabolism by the liver?<br />

Which metabolic process is responsible<br />

for the toxic effects of alcohol?<br />

What are the chronic effects<br />

of alcohol ingestion?<br />

What are the three types of tolerance?<br />

The limiting factor in alcohol<br />

metabolism is the availability of<br />

nicotinamide adenine dinucleotide<br />

(NAD) and nicotinamide adenine<br />

dinucleotide plus hydrogen (NADH),<br />

which is used by aldehyde<br />

dehydrogenase.<br />

The toxic effects of alcohol are related to<br />

metabolism of alcohol to aldehydes, the<br />

buildup of which causes liver and tissue<br />

damage, flushing, nausea, and<br />

headache.<br />

Development of tolerance<br />

Psychological dependence<br />

Physiologic dependence (this includes<br />

tolerance and withdrawal)<br />

Huge host of medical problems,<br />

including liver disease, cancer, etc<br />

1. Metabolic tolerance: allows for<br />

more rapid metabolism and<br />

excretion<br />

2. <strong>Behavioral</strong> tolerance: involves<br />

learning to perform a task while<br />

intoxicated as if the person is not<br />

intoxicated<br />

3. Neuroadaptation: involves the<br />

evolution of alterations in the brain<br />

that help to overcome the effects of<br />

the drug


150 Deja Review: <strong>Behavioral</strong> <strong>Science</strong><br />

What are symptoms of minor<br />

withdrawal from chronic alcohol use?<br />

What are withdrawal seizures?<br />

What are alcoholic hallucinations?<br />

What is delirium tremens (DTs)?<br />

What is the earliest that DTs occur?<br />

How is alcohol withdrawal treated?<br />

Minor signs occur early and subside<br />

within 2 days. They include anxiety,<br />

tremulousness, insomnia, GI upset,<br />

palpitations, diaphoresis, and headache.<br />

Tonic-clonic movements occurring from<br />

2-48 hours after the last drink. This may<br />

progress to delirium tremens (DT).<br />

Visual, tactile, or auditory hallucinations<br />

from 12-48 hours after the last drink.<br />

Vital signs remain normal.<br />

An alcohol withdrawal syndrome with<br />

hallucinations, altered mental status,<br />

and seizures accompanied by vital<br />

sign abnormalities (tachycardia,<br />

hypertension, fever). Vital sign<br />

abnormalities may be the first clue<br />

to diagnosis. DT may lead to death.<br />

48 hours after the last drink, though<br />

may be longer<br />

Benzodiazepines for suppression of<br />

withdrawal symptoms<br />

Vitamins (particularly thiamine) to treat<br />

underlying vitamin deficiencies seen in<br />

long-term alcoholism<br />

Antihypertensives to suppress<br />

underlying hypertension that can<br />

be made worse by the physiologic<br />

withdrawal process<br />

What is the relapse rate in the first year 75%<br />

after alcohol detoxification?<br />

What are the most common reasons<br />

for relapse in alcoholics?<br />

What types of treatment can be used<br />

to prevent relapse in alcoholics?<br />

Memories/conditioning<br />

Peer pressure<br />

Genetics<br />

Rapid reinstatement of physiologic<br />

dependence<br />

Support groups (eg, AA)<br />

Psychological treatment<br />

Pharmacotherapy (naltrexone,<br />

disulfiram, or acamprosate)


Substance Abuse Disorders 151<br />

CLINICAL VIGNETTES<br />

A 48-year-old man comes in for an elective surgery. His operation is a great success<br />

and there are no major complications. Three days after his surgery he becomes<br />

very disoriented, and yells nonsense at the nurses. His HR is 110, his BP is 154/92,<br />

and he is febrile. His family asks you if there is anything they can do. What is a<br />

key piece of history you could ask the family about?<br />

Ask about his drinking habits. He may be in alcohol withdrawal since he has<br />

likely not had a drink since surgery.<br />

A 26-year-old woman just had her third DUI arrest. She gets drunk every night<br />

with her friends and has never had any regrets about doing it. She has lost four<br />

jobs because she is unable to show up on time in the morning. She claims she<br />

doesn’t have a problem because it only takes her 5 beers to get drunk—the same<br />

amount as when she started drinking 5 years ago. She now spends most of her day<br />

doing odd jobs so that she can afford to go out drinking. Her doctor told her that<br />

her liver is suffering, but she continues to drink. What is her likely diagnosis?<br />

She meets criteria for substance dependence, despite her lack of tolerance. She spends<br />

most of her time involved in substance-seeking behavior, has had occupational<br />

impairment, and has had a physical impact from use as well. She cannot be<br />

diagnosed with substance abuse as dependence takes precedence.<br />

A 19-year-old college student is brought to the ER by his fraternity brothers at 3 AM<br />

after a Friday night party. The young man is arousable to painful stimuli only. His<br />

pupils are only 1 mm, but reactive. You notice his respiratory rate is quite low. His<br />

friends say there has been a lot of drinking at the party as well as some pills.<br />

What drugs has the patient most likely ingested?<br />

Alcohol and opiates (note pinpoint pupils)<br />

What drug should be administered immediately after this patient arrives in the ER?<br />

Naloxone


This page intentionally left blank


CHAPTER 20<br />

Eating Disorders<br />

What are the two major eating disorders<br />

according to the DMV-IV-TR?<br />

What are the two subtypes of Anorexia<br />

Nervosa?<br />

What distinguishes the subtypes?<br />

What is the biggest distinguishing<br />

characteristic between Anorexia<br />

and Bulimia?<br />

What are the DSM-IV-TR diagnostic<br />

criteria for anorexia nervosa?<br />

What are the associated physical<br />

or biological findings in anorexia<br />

nervosa?<br />

How is amenorrhea defined?<br />

1. Anorexia Nervosa<br />

2. Bulimia Nervosa<br />

1. Restricting type<br />

2. Binge-Eating/Purging type<br />

The presence of either regular binge<br />

eating or purging behavior is Bingeeating/purging<br />

type. The absence of<br />

either is restricting type.<br />

In anorexia there is a refusal to maintain<br />

a body weight above 85% of ideal.<br />

Bulimic patients may be of normal<br />

weight.<br />

All of the following:<br />

Severe weight loss (weighs


154 Deja Review: <strong>Behavioral</strong> <strong>Science</strong><br />

What is lanugo?<br />

What is melanosis coli?<br />

What is a long-term risk of anorexia?<br />

What are some of the warning signs<br />

of anorexia?<br />

What is the typical profile of a patient<br />

with anorexia?<br />

Do patients with anorexia typically<br />

have coexisting disorders?<br />

What is the best treatment for anorexia<br />

nervosa?<br />

What are the DSM-IV-TR diagnostic<br />

criteria for bulimia nervosa?<br />

Fine, downy body hair, especially seen<br />

on the trunk<br />

Blackened areas on the colon, seen with<br />

laxative abuse<br />

Osteopenia, Mitral Valve Prolapse,<br />

Amenorrhea, Electrolyte Disturbance,<br />

and Death<br />

Excessive dieting, exercise, use of<br />

laxatives/diuretics/enemas<br />

Abnormal eating habits<br />

Body image disorder/body dysmorphic<br />

disorder<br />

Fear of becoming fat<br />

Decreased libido<br />

Anorexia is most commonly seen in<br />

adolescent to young adult females<br />

who are very high achieving (either<br />

academically, athletically, or both).<br />

There is often a lot of conflict within<br />

the family, sometimes described as a<br />

controlling or overly protective mother.<br />

Anxiety disorders are commonly seen,<br />

as are mood disorders and substance<br />

abuse.<br />

Nutritional therapy and psychotherapy<br />

including cognitive-behavioral therapy<br />

(CBT) can be helpful. Pharmacotherapy is<br />

not terribly helpful.<br />

All of the following:<br />

Recurrent binge eating, with both a<br />

large amount of food and a lack of<br />

control during the episode<br />

Compensatory behavior to prevent<br />

weight gain (vomiting, laxatives,<br />

exercise, etc)<br />

Binge and purge behaviors occur at<br />

least twice a week for 3 months<br />

Self-evaluation influenced by body<br />

shape and weight<br />

Does not qualify for anorexia nervosa<br />

diagnosis


Eating Disorders 155<br />

What are common associated findings<br />

in persons with bulimia nervosa?<br />

What is the cause of erosion of the<br />

tooth enamel?<br />

What is parotiditis?<br />

What causes the development of<br />

calluses on the back of the hands?<br />

What common medical consequences<br />

are seen with repeated vomiting?<br />

What is a traumatic consequence<br />

of the repeated induced vomiting<br />

seen in bulimia nervosa?<br />

How can purging be accomplished?<br />

Do Bulimics have to vomit or use<br />

laxatives to meet criteria?<br />

What are some of the psychosocial<br />

features of patients with bulimia?<br />

What is the best treatment for bulimia<br />

nervosa?<br />

Erosion of tooth enamel<br />

Parotitis (inflammation of parotid<br />

gland)<br />

Calluses on the dorsal surface of the<br />

hands<br />

Electrolyte imbalance<br />

Often normal body weight (may be<br />

slightly overweight)<br />

Repeated exposure to gastric acid<br />

secondary to induced vomiting<br />

Swelling or infection of the parotid<br />

glands, usually secondary to<br />

vomiting<br />

Scraping fingers along teeth while<br />

inducing vomiting<br />

Hypokalemia<br />

Metabolic Alkalosis<br />

Endocrine disturbances<br />

Growth disturbance<br />

Bradycardia and dysrhythmias<br />

Esophageal varices and/or Mallory-Weiss<br />

tears from repeated retching<br />

Induced vomiting<br />

Laxative/diuretic/enema use<br />

Excessive exercise<br />

No. Even excessive exercise can<br />

be a “purging” behavior.<br />

Poor self-image<br />

Depression and other mood disorders<br />

Psychotherapy<br />

Behavior therapy<br />

Antidepressants—selective serotonin<br />

reuptake inhibitors (SSRIs) preferred<br />

Nutrition education<br />

Regular meals<br />

Healthy exercise


156 Deja Review: <strong>Behavioral</strong> <strong>Science</strong><br />

What is Eating Disorder Not Otherwise<br />

Specified (NOS)?<br />

A grab-bag of disordered eating that<br />

does not meet criteria for Anorexia or<br />

Bulimia. This includes:<br />

• Purging behavior without true binge<br />

eating<br />

• Anorexia criteria without amenorrhea<br />

• Restricting type anorexia without<br />

significant weight loss (ie, no binge<br />

eating/purging to meet bulimia<br />

criteria)<br />

• Bulimia criteria except at a lower<br />

frequency than required for<br />

diagnosis<br />

• Chewing and spitting out large<br />

amounts of food<br />

• Binge eating without purging<br />

behavior<br />

CLINICAL VIGNETTES<br />

A 17-year-old woman comes to your office complaining of fatigue. Upon careful<br />

examination you see erosion of the enamel on her molars. You check an electrolyte<br />

panel and find her to be hypokalemic.<br />

What are the possible psychiatric disorders that may be present?<br />

This patient may have anorexia nervosa, bulimia nervosa, or eating disorder NOS.<br />

How do you distinguish anorexia nervosa and bulimia nervosa?<br />

To distinguish these disorders you would need to know her body weight vs her<br />

ideal body weight.<br />

What is the likely cause of her hypokalemia? If you took an arterial blood gas,<br />

what would you expect the pH to be?<br />

Her hypokalemia is likely due to recurrent vomiting—this will also cause a<br />

metabolic alkalosis (think loss of stomach H + ), so you would expect her pH to be<br />

high (alkalemic).


Eating Disorders 157<br />

A 12-year-old male comes into your office asking for help in losing weight to make<br />

the wrestling team. He states that he has been exercising significantly to lose<br />

weight, but with no effect. He is terribly afraid of gaining weight, believes he is<br />

horribly fat, and hates himself for weighing so much. His ideal body weight is 100 lb<br />

and he currently weighs 90 lb. He states he’d like to weigh 80 lb and requests you<br />

to give him diuretics to help him do so, which you refuse to do.<br />

What is his current likely diagnosis?<br />

His current diagnosis is Eating Disorder NOS—he would meet criteria for anorexia<br />

nervosa, but he does not weigh


This page intentionally left blank


CHAPTER 21<br />

Child Psychiatry<br />

PERVASIVE DEVELOPMENT DISORDERS<br />

What are the primary characteristics<br />

of the pervasive development<br />

disorders?<br />

What are the pervasive development<br />

disorders?<br />

Which disorder is characterized by<br />

significant communication problems,<br />

difficulty in forming social<br />

relationships, repetitive behavior,<br />

and unusual abilities?<br />

What term describes the unusual<br />

abilities (eg, memory, calculation skills)<br />

that some autistic patients have?<br />

The onset of autistic disorder must be<br />

before what age?<br />

What is the risk of autism in<br />

monozygotic twins and siblings?<br />

The incidence of autistic disorder is<br />

increased in which conditions?<br />

What has been shown in multiple<br />

analysis not to be associated with autism?<br />

Failure to acquire or early loss of<br />

communication and social interaction<br />

skills<br />

Autistic disorder<br />

Asperger disorder<br />

Rett disorder<br />

Childhood disintegrative disorder<br />

Autistic disorder<br />

Savant<br />

3 years of age<br />

Increased risk due to genetic component<br />

Congenital anomalies<br />

Perinatal complications<br />

Congenital rubella<br />

Phenylketonuria<br />

Fragile X syndrome<br />

Tuberous sclerosis<br />

Vaccination<br />

159


160 Deja Review: <strong>Behavioral</strong> <strong>Science</strong><br />

Which brain abnormalities are<br />

associated with autism?<br />

Which gender is most likely to be<br />

affected by autism?<br />

Which gender is more severely<br />

affected with mental retardation<br />

in autism?<br />

How is autistic disorder treated?<br />

What is the IQ of many autistic patients?<br />

Which disorder is a milder form<br />

of autism?<br />

What are the primary deficits in a<br />

patient with Asperger?<br />

Which areas of functioning are normal<br />

in Asperger but are usually deficient<br />

in autistic patients?<br />

Which disorder is characterized by a<br />

decrease in social, verbal, and cognitive<br />

development after a period of normal<br />

functioning?<br />

What are the primary characteristics<br />

of Rett disorder?<br />

Which gender is primarily affected<br />

by Rett disorder?<br />

What is the genetic inheritance of<br />

Rett disorder?<br />

Seizures<br />

Electroencephalogram (EEG)<br />

abnormalities<br />

Anatomic and functional<br />

abnormalities<br />

Males; Four times more likely<br />

Females<br />

Therapy aimed at increasing<br />

communication, social, and self-care<br />

skills<br />

Generally low; may have normal<br />

nonverbal IQ<br />

Asperger disorder<br />

Problems forming social relationships<br />

Repetitive behavior<br />

Acute interest in obscure topics<br />

Cognitive and verbal skills<br />

Rett disorder<br />

Stereotyped hand-wringing movements<br />

Poor coordination<br />

Impaired language development<br />

Loss of hand skills<br />

Loss of social engagement<br />

Deceleration of head growth<br />

Females<br />

X-linked


Child Psychiatry 161<br />

What happens to males affected<br />

by Rett disorder?<br />

Which rare disorder is characterized<br />

by a diminution of cognitive, motor,<br />

social, and verbal development after<br />

2 to 10 years of normal functioning?<br />

Which gender has the highest<br />

incidence of childhood<br />

disintegrative disorder?<br />

They die before birth.<br />

Childhood disintegrative disorder<br />

Boys<br />

DISRUPTIVE BEHAVIOR DISORDERS<br />

What are the primary characteristics<br />

of disruptive behavior disorders?<br />

Which disorders are classified<br />

as disruptive behavior disorders?<br />

Which disorder is characterized by<br />

insistent behavior that violates<br />

social norms, deviation from societal<br />

and parental rules, property destruction,<br />

and aggressive behavior?<br />

What are examples of the behaviors<br />

that violate social norms that are<br />

common in patients with conduct<br />

disorder?<br />

If a person is 18 years or older and<br />

still exhibits the symptoms of<br />

conduct disorder, which disorder<br />

might they have?<br />

Which disorder is characterized by<br />

persistent disobedient, defiant,<br />

and negative behavior toward<br />

figures in authority?<br />

What are the primary treatment<br />

modalities for the disruptive<br />

behavior disorders?<br />

Improper behavior; problems with<br />

school performance and social<br />

relationships<br />

Conduct disorder<br />

Oppositional defiant disorder<br />

Conduct disorder<br />

Arson<br />

Theft<br />

Animal harm<br />

Assault<br />

Antisocial personality disorder<br />

Oppositional defiant disorder<br />

Psychotherapy and a Structured<br />

environment, sometimes<br />

pharmacotherapy


162 Deja Review: <strong>Behavioral</strong> <strong>Science</strong><br />

ATTENTION-DEFICIT HYPERACTIVITY DISORDER<br />

What are the primary characteristics<br />

of attention-deficit hyperactivity<br />

disorder (ADHD)?<br />

Which gender is most likely to be<br />

affected by ADHD?<br />

By definition, ADHD symptoms<br />

must be evident by which age to be<br />

given the classification of ADHD?<br />

How long must the symptoms of ADHD<br />

be present to be given the classification<br />

of ADHD?<br />

Inattention<br />

Hyperactivity<br />

Impulsivity<br />

Impairment in multiple settings<br />

(eg, both school and home)<br />

Boys<br />

Before 7 years of age<br />

At least 6 months<br />

What percentage of the general child 3% to 7%<br />

population in the United States is<br />

affected by ADHD?<br />

What is the intelligence level of persons<br />

with ADHD?<br />

Normal intelligence<br />

What percentage of ADHD patients 20%<br />

have symptoms that persist into<br />

adulthood?<br />

What are the primary treatment<br />

modalities of ADHD?<br />

What is a non-stimulant medication<br />

for individuals with ADHD greater<br />

than the age of 6?<br />

Which single drug is most widely<br />

prescribed for ADHD?<br />

Stimulants—usually amphetamines<br />

Atomoxetine<br />

Methylphenidate<br />

OTHER NEUROPSYCHIATRIC DISORDERS OF CHILDHOOD<br />

Which disorder is characterized by<br />

chronic motor and vocal tics and<br />

involuntary use of profanity?<br />

Tourette syndrome


Child Psychiatry 163<br />

In which age group is Tourette<br />

syndrome most likely to be<br />

diagnosed?<br />

Tourette syndrome has a high<br />

comorbidity with which other<br />

psychiatric disorders?<br />

What is the primary treatment<br />

for Tourette syndrome?<br />

What is a treatment for Tourette<br />

syndrome that is refractory<br />

to medications?<br />

Which disorder is characterized by<br />

excessive and inappropriate anxiety<br />

concerning separation from parents,<br />

caretakers, and their home and<br />

production of physical complaints<br />

to avoid going to school?<br />

What is the most common age of<br />

onset in a person who presents<br />

with separation anxiety disorder?<br />

Usually 7 to 8 years of age; onset<br />

usually by 21 years of age<br />

ADHD<br />

Obsessive-compulsive disorder<br />

Dopamine-blocking antipsychotics<br />

(if interfering with social interactions)<br />

Botox (for tics) or Deep brain stimulation<br />

Separation anxiety disorder<br />

7 to 8 years of age<br />

Psychotherapy, especially cognitive-<br />

behavioral therapy<br />

Selective mutism<br />

Girls<br />

10 years of age<br />

Stressful life events<br />

Family and behavioral therapy<br />

Which treatment modalities are most<br />

effective in the treatment of separation<br />

anxiety disorder?<br />

Which disorder is characterized by<br />

a refusal to verbally communicate<br />

in some or all social situations in<br />

which the child may communicate<br />

with gestures?<br />

In which gender is selective mutism<br />

more common?<br />

Selective mutism has a poor<br />

prognosis if it persists after<br />

which age?<br />

What is the primary trigger for the onset<br />

of separation anxiety disorder and<br />

selective mutism?<br />

What is the most common treatment<br />

modality for selective mutism?


164 Deja Review: <strong>Behavioral</strong> <strong>Science</strong><br />

CLINICAL VIGNETTES<br />

A distressed mother comes to your clinic with her 4-year-old daughter. She claims<br />

her daughter was completely normal up until last year when she had a regression<br />

of her social and cognitive skills—right after her last batch of immunizations. The<br />

mother is now considering suing the vaccine manufacturer for causing her child to<br />

have autism. What can you tell the mother is the most likely diagnosis of the child?<br />

Childhood disintegrative disorder<br />

A father brings his 9-year-old son in to evaluate him for ADHD. His teacher told<br />

the father that his son is impulsive, inattentive, and can’t stay in his seat. The<br />

father says that neither he nor his mother has noticed these symptoms anywhere<br />

else, but he believes the teacher. He asks if his son should be started on<br />

medication. What should you advise the father?<br />

The child shouldn’t be started on any medications at this time. The child’s<br />

behavior is limited only to the school setting and does not meet criteria for ADHD.<br />

A 14-year-old male is brought to a psychiatrist because of his disregard for rules<br />

set by authority figures at home and in school. He has set numerous fires in his<br />

neighborhood. His parents are concerned that he seems to have no regard for the<br />

feelings of others. Which disorder is this patient most likely to be diagnosed with?<br />

Conduct disorder<br />

A 3-year-old girl presents with problems forming social relationships. She engages<br />

in repetitive behavior, and she has a strong interest in learning all about the<br />

different types of bubble gum in the world. Her mother states that the girl has not<br />

had any cognitive deficits and has had no developmental language delay. Which<br />

condition is the patient most likely to be diagnosed with?<br />

Asperger disorder


CHAPTER 22<br />

Psychopharmacology<br />

What is the only property of<br />

benzodiazepines to which<br />

tolerance does not develop?<br />

Which benzodiazepines are considered<br />

anxiolytics?<br />

Which benzodiazepines are considered<br />

hypnotics (used to facilitate sleep)?<br />

What drug can be used to reverse<br />

the effects of benzodiazepines?<br />

What is the general mechanism<br />

of action of benzodiazepines?<br />

Is there a danger in taking<br />

benzodiazepines during pregnancy?<br />

Tolerance does not develop to the<br />

antianxiety/anxiolytic effects of benzos;<br />

tolerance may develop to the hypnotic,<br />

muscle relaxant, and anticonvulsant<br />

effects (eg, benzodiazepines should not<br />

be used for long-term seizure control).<br />

Alprazolam<br />

Chlordiazepoxide<br />

Clonazepam<br />

Clorazepate<br />

Diazepam<br />

Lorazepam<br />

Quazepam<br />

Midazolam<br />

Estazolam<br />

Flurazepam<br />

Temazepam<br />

Triazolam<br />

Flumazenil (Romazicon)—though it is<br />

used with caution as it can precipitate<br />

withdrawal seizures<br />

Benzodiazepines target the GABA A<br />

(γ-aminobutyric acid A) chloride channel<br />

receptor, resulting in an increase in the<br />

receptor’s affinity for and causing the<br />

ion channels to open more frequently, thus<br />

allowing more chloride ions to pass<br />

through.<br />

Benzodiazepines can cross the placenta,<br />

and therefore should not be taken<br />

during pregnancy if possible.<br />

They are categories D and X.<br />

165


166 Deja Review: <strong>Behavioral</strong> <strong>Science</strong><br />

How many phases are there in<br />

benzodiazepine metabolism?<br />

Where does benzodiazepine<br />

metabolism occur?<br />

Which benzodiazepines skip phase 1<br />

and/or phase 2 of metabolism and are<br />

therefore safer to give to patients<br />

with liver failure?<br />

What are the symptoms of<br />

benzodiazepine withdrawal?<br />

What are the possible side effects<br />

of benzodiazepines?<br />

Which benzodiazepines are considered<br />

to be high potency?<br />

Which benzodiazepines are considered<br />

to be low potency?<br />

Which benzodiazepines are most<br />

commonly used for treatment<br />

of alcohol withdrawal seizures<br />

(delirium tremens)?<br />

There are three phases.<br />

1. Phase 1: The R1 and R2 residues are<br />

oxidized.<br />

2. Phase 2: The R3 residue is<br />

hydroxylated.<br />

3. Phase 3: The hydroxyl compounds<br />

are conjugated with glucuronic acid.<br />

In the liver<br />

Desmethyldiazepam<br />

Oxazepam<br />

Temazepam<br />

Lorazepam<br />

Midazolam<br />

Triazolam<br />

Anxiety<br />

Insomnia<br />

Irritability<br />

Delirium, Psychosis<br />

Weakness<br />

Tremor<br />

Seizures<br />

Drowsiness<br />

Confusion<br />

Motor incoordination<br />

Cognitive impairment<br />

Anterograde amnesia<br />

Triazolam<br />

Alprazolam<br />

Clonazepam<br />

Diazepam<br />

Chlordiazepoxide<br />

Oxazepam<br />

Chlordiazepoxide<br />

Lorazepam<br />

Diazepam


Psychopharmacology 167<br />

Which benzodiazepines are indicated<br />

for treatment of status epilepticus,<br />

as they can be given intravenously (IV)?<br />

Why are benzodiazepines safer<br />

pharmacologic agents to use to<br />

treat anxiety than barbiturates?<br />

What is the mechanism of action<br />

of barbiturates?<br />

What is the mechanism of action and<br />

common use of Buspirone?<br />

Which short-acting antianxiety agent<br />

is used to treat insomnia?<br />

What is the most commonly prescribed<br />

class of drugs for the treatment<br />

of depression?<br />

What is the mechanism of action<br />

of SSRIs?<br />

What are the names of commonly<br />

prescribed SSRIs?<br />

What serious side effects have a greater<br />

incidence associated with the SSRI<br />

Paroxetine?<br />

What caution should you take while<br />

using SSRIs in children and adolescents?<br />

Diazepam and lorazepam<br />

Less potential for abuse<br />

Higher therapeutic index<br />

Barbiturates target the GABA A chloride<br />

channel receptor and its action on<br />

chloride entry into the cell, which<br />

results in membrane hyperpolarization.<br />

There is an increase in the duration of<br />

the chloride channel opening and a<br />

decrease in neuron excitability.<br />

It is a 5-hydroxytryptamine receptor 1A<br />

(5-HT 1A ) (serotonin) agonist that may be<br />

used to treat anxiety, particularly useful<br />

in those for whom benzodiazepine<br />

therapy is contraindicated (the elderly<br />

and those with a history of substance<br />

abuse).<br />

Zolpidem tartrate<br />

SSRIs (selective serotonin reuptake<br />

inhibitors)<br />

They work by inhibiting neuronal<br />

uptake of serotonin, thereby increasing<br />

the synaptic concentration of serotonin.<br />

Fluoxetine<br />

Fluvoxamine<br />

Paroxetine<br />

Sertraline<br />

Citaprolam<br />

Anticholinergic effects, sexual<br />

dysfunction, and withdrawal syndrome<br />

There is a black box warning about<br />

increased suicidal thoughts in patients<br />


168 Deja Review: <strong>Behavioral</strong> <strong>Science</strong><br />

Which SSRIs are now available<br />

in generic form (and may be a better<br />

choice for patients with a limited<br />

budget)?<br />

Why does fluoxetine and paroxetine<br />

have a greater association with drug<br />

interactions than other SSRIs?<br />

Why is insomnia a troubling side effect<br />

of fluoxetine?<br />

Which SSRI is most likely to cause<br />

gastrointestinal disturbances?<br />

Which SSRI is currently only<br />

indicated for obsessive-compulsive<br />

disorder?<br />

What is the mechanism of action<br />

of bupropion (Wellbutrin)?<br />

What is the major indication<br />

for bupropion?<br />

What is another common indication<br />

for bupropion?<br />

What is a unique side effect<br />

of bupropion that should be thought<br />

of before prescribing it in epileptic<br />

patients?<br />

What is a common indication for the use<br />

of trazodone?<br />

What is a problematic side effect of the<br />

antidepressant trazodone seen mostly<br />

in males?<br />

What is the mechanism of action<br />

of tricyclic antidepressants (TCA)?<br />

Fluoxetine (Prozac)<br />

Paroxetine<br />

Sertraline<br />

Citalopram<br />

They have greater P-450 inhibition than<br />

the other SSRIs.<br />

It causes the most central nervous<br />

system (CNS) activation of the<br />

SSRIs.<br />

Sertraline (Zoloft)<br />

Fluvoxamine<br />

It is thought to work mostly on dopamine<br />

and norepinephrine, though the exact<br />

mechanism is unknown.<br />

Major depressive disorder<br />

Smoking cessation<br />

It can lower the seizure threshold,<br />

especially in quick release formulas<br />

and at high doses.<br />

Insomnia<br />

Priapism (painful, persistent erection)<br />

Tricyclic antidepressants inhibit the<br />

neuronal reuptake of both serotonin<br />

and norepinephrine, thus, increasing<br />

the availability of serotonin and<br />

norepinephrine at the synaptic<br />

cleft.


Psychopharmacology 169<br />

What are the names of commonly<br />

prescribed tricyclic antidepressants?<br />

Why are tricyclics used less often<br />

than SSRIs?<br />

What other syndrome are TCAs<br />

indicated for treating?<br />

What are common side effects<br />

of tricyclic antidepressants?<br />

What are the effects of TCA overdose?<br />

What is the most important clinical<br />

test in evaluating suspected tricyclic<br />

overdose?<br />

What is the treatment of acute tricyclic<br />

overdose?<br />

Which tricyclic antidepressants<br />

are known to have more anticholinergic<br />

effects?<br />

What are examples of other tertiary<br />

tricyclics antidepressants?<br />

Which tricyclic agent is the least<br />

sedating?<br />

Which tricyclic is prescribed for<br />

enuresis (bed-wetting)?<br />

Which tricyclic is classically prescribed<br />

for obsessive-compulsive disorder?<br />

Amitriptyline<br />

Clomipramine<br />

Doxepin<br />

Desipramine<br />

Imipramine<br />

Nortriptyline<br />

Side effects and potential for lethal<br />

overdose. They are still very efficacious<br />

medicines.<br />

Chronic and neuropathic pain<br />

Tricyclic antidepressants have<br />

anticholinergic effects including dry<br />

mouth, blurry vision, constipation,<br />

urinary retention, confusion, and<br />

memory deficits.<br />

Cardiac effects are the most<br />

dangerous—notably widened QRS,<br />

prolonged PR and QT intervals.<br />

Sedation, delirium, and anticholinergic<br />

effects may also be present.<br />

ECG<br />

IV Sodium Bicarbonate<br />

Tertiary tricyclics (eg, amitriptyline)<br />

Imipramine<br />

Doxepin<br />

Desipramine<br />

Imipramine<br />

Clomipramine<br />

Note: This is the most serotonin-specific<br />

tricyclic.


170 Deja Review: <strong>Behavioral</strong> <strong>Science</strong><br />

Which tricyclic is the least likely<br />

to cause orthostatic hypertension?<br />

What is the most common indication<br />

for MAOIs (monoamine oxidase<br />

inhibitors)?<br />

What is the mechanism of action<br />

of MAOIs?<br />

What are the names of commonly<br />

prescribed MAOIs?<br />

What are the common side effects<br />

of MAOIs<br />

What other medication must be avoided<br />

in patients taking MAOIs?<br />

What other substance must be avoided<br />

by patients taking MAOIs?<br />

What MAO inhibitor is now available<br />

in a transdermal patch which does not<br />

require a change in diet?<br />

What are some pharmacologic agents<br />

that can cause psychosis?<br />

Nortriptyline<br />

Treatment of atypical depression<br />

(depression with increased sleep,<br />

appetite, and leaden paralysis)<br />

They work by inhibiting the<br />

mitochondrial enzyme monoamine<br />

oxidase which metabolizes<br />

norepinephrine, serotonin, and<br />

dopamine, resulting in a buildup<br />

of these biogenic amines and their<br />

subsequent leakage into the<br />

synapse.<br />

Selegiline<br />

Phenelzine<br />

Tranylcypromine<br />

Sedation<br />

Anticholinergic effects<br />

Orthostatic hypotension<br />

Cardiac conduction disturbances<br />

Stimulants and pseudoephedrine must<br />

be avoided because they can cause<br />

hypertensive crisis.<br />

Tyramine, which is found in aged<br />

cheeses, beer, certain meats and fish,<br />

fava beans, red wine (particularly<br />

Chianti), avocados, chocolate,<br />

and dairy products. Signs of<br />

tyramine ingestions include<br />

headache, arrhythmias, and<br />

hypertensive crisis.<br />

Selegiline, which is also useful for<br />

patients who cannot swallow oral<br />

medicines<br />

Amphetamines<br />

Cocaine<br />

L-Dopa


Psychopharmacology 171<br />

What is the mechanism of action for<br />

most typical antipsychotics?<br />

What is the most popular of the<br />

biochemical theories that explain<br />

the development of psychosis?<br />

What is the most common problematic<br />

side effect of typical antipsychotics?<br />

Which of the extrapyramidal side<br />

effects is a flag for caution for future<br />

use of typical antipsychotics?<br />

Which typical antipsychotics have the<br />

most extrapyramidal effects?<br />

Which is the most potent of the typical<br />

antipsychotics?<br />

What pharmacologic evidence does not<br />

support the dopamine hypothesis?<br />

What is the mechanism of action of the<br />

atypical antipsychotic clozapine?<br />

What is the greatest benefit of atypical<br />

antipsychotics?<br />

What is the major side effect to beware<br />

of with clozapine?<br />

They are dopamine receptor<br />

antagonists.<br />

The dopamine hypothesis which cites<br />

an increase in dopamine transmission<br />

as the cause. This is supported by the<br />

clinical potentcy of typical<br />

antipsychotics being directly related<br />

to their affinity to the D2 receptor.<br />

Extrapyramidal effects/Parkinson-like<br />

side effects:<br />

Bradykinesia<br />

Rigidity<br />

Tremor<br />

Acute dystonia<br />

Akathisia<br />

Note: Chronic tardive dyskinesias may<br />

also occur.<br />

Acute dystonic reaction<br />

Fluphenazine = haloperidol<br />

> chlorpromazine > thioridazine<br />

Note: This is correlated with<br />

potency/D2 blockade.<br />

Fluphenazine<br />

Atypical antipsychotics are very<br />

efficacious, but many work on other<br />

neurotransmitters and have little effect<br />

on dopamine.<br />

It acts on dopamine, serotonin,<br />

and acetylcholine receptors.<br />

They usually have fewer extrapyramidal<br />

side effects than typical antipsychotics.<br />

Agranulocytosis → necessitates weekly<br />

or biweekly white blood cells (WBC)<br />

monitoring in patients on clozapine<br />

therapy.


172 Deja Review: <strong>Behavioral</strong> <strong>Science</strong><br />

What is the biggest benefit of clozapine?<br />

What is the mechanism of action of the<br />

atypical antipsychotic risperidone?<br />

What are other side effects of<br />

antipsychotic therapy?<br />

What is the cause of amenorrhea<br />

in women taking antipsychotics?<br />

Which pharmacologic agents are used<br />

to treat acute mania associated with<br />

bipolar disorder?<br />

What agents are used for maintenance<br />

therapy in bipolar disorder?<br />

What are some of the side effects<br />

of lithium?<br />

It is efficacious for treatment. It is<br />

efficacious for resistant schizophrenia,<br />

though because of the side effects and<br />

monitoring it is never first line.<br />

It works by blocking both dopamine<br />

and serotonin receptors.<br />

Weight gain and metabolic syndrome<br />

Anticholinergic effects<br />

Antihistaminergic effects/sedation<br />

Amenorrhea<br />

Temperature dysregulation<br />

Neuroleptic malignant syndrome<br />

Blood dyscrasias<br />

Sexual dysfunction<br />

Dopamine receptor antagonism leading<br />

to elevated prolactin levels<br />

Lithium<br />

Atypical antipsychotics<br />

Valproic Acid (aka Depakote, valproate)<br />

Carbamazepine<br />

Benzodiazepines<br />

Lithium<br />

Valproic acid<br />

Carbamazepine<br />

Atypical antipsychotics<br />

Lamotrigine (Lamictal)<br />

Polyuria → acts as an antidiuretic<br />

hormone (ADH) antagonist to cause<br />

nephrogenic diabetes insipidus<br />

Tremor<br />

Hypothyroidism<br />

Weight gain<br />

Gastrointestinal effects (nausea,<br />

vomiting, diarrhea)<br />

Leukocytosis (high white blood<br />

cell count)<br />

Acne<br />

Note: Use of lithium requires close<br />

monitoring of serum levels.


Psychopharmacology 173<br />

Which pharmacologic treatment is safe<br />

in pregnant bipolar women?<br />

What is the main drawback of using<br />

antipsychotics such as olanzapine<br />

for bipolar maintenance?<br />

Which anticonvulsants are used to<br />

treat bipolar disorder?<br />

What type of therapy is used for major<br />

depressive order that is refractory<br />

to treatment?<br />

In what other psychiatric conditions<br />

can ECT be used as treatment therapy?<br />

When does the maximum response<br />

to ECT usually occur?<br />

What is the biggest side effect of ECT?<br />

How can the amnesia and memory<br />

loss associated with ECT be<br />

minimized?<br />

None. Risk of relapse must be weighed<br />

against the risks of birth defects.<br />

Electroconvulsive therapy is likely the<br />

safest treatment overall in pregnancy.<br />

Side effects, especially weight gain<br />

Carbamazepine<br />

Valproic acid<br />

Lamotrigine<br />

Electroconvulsive therapy (ECT)<br />

Acute mania<br />

Schizophrenia with catatonic symptoms<br />

Depression, particularly with psychotic<br />

symptoms or in pregnancy<br />

Catatonia<br />

After a period of several weeks<br />

over which 5 to 12 treatments are<br />

administered.<br />

Memory loss (Retrograde amnesia),<br />

which may or may not be permanent.<br />

Unilateral electrode placement<br />

Is modern ECT different than<br />

Absolutely. Use of appropriate<br />

20 years ago? anesthesia and modern ECT techniques<br />

has significantly reduced side effects<br />

and complications.<br />

All possibilities considered,<br />

what is the most efficacious<br />

treatment for depression?<br />

ECT


174 Deja Review: <strong>Behavioral</strong> <strong>Science</strong><br />

CLINICAL VIGNETTES<br />

An unresponsive young woman is brought to the ER by her college roommate.<br />

Her roommate states that she had confided in her that she was being treated for<br />

depression for quite some time with little success. She found an empty pill bottle<br />

by the patient as well, but didn’t bring it in. On examination the patient has<br />

widely dilated pupils, a dry mouth, and tachycardia.<br />

What was likely in that bottle of pills taken by the patient?<br />

She likely is suffering from tricyclic antidepressant overdose. Note the anticholinergic<br />

symptoms and the limited success of treatment (perhaps necessitating a few different<br />

medication trials).<br />

What diagnostic test would you order? What result do you expect on that test?<br />

You should order an ECG, on which you would likely see a prolonged PR interval,<br />

a widened QRS, and a long QT interval.<br />

What is the appropriate management of the patient?<br />

Of course you would start with ABCs, but intravenous bicarbonate is a specific<br />

treatment for TCA overdose.<br />

A 62-year-old patient with schizophrenia comes in to inquire about new treatment<br />

options. He has been on haloperidol which has controlled his symptoms well. His<br />

past medical history includes pre-diabetes, obesity (though he’s trying to lose<br />

weight), hypertension, and a family history of coronary artery disease. He says that<br />

he has heard that some of the new atypical antipsychotics are more effective than<br />

his old agent.<br />

What should you advise him?<br />

Atypical medications are no more effective than typical agents, with the notable<br />

exception of clozapine. The main advantage of atypical agents is a more favorable<br />

side-effect profile with less EPS and tardive dyskinesia than typical agents.<br />

What side-effects might you worry about in this patient?<br />

Prominent metabolic effects and weight gain are side effects of atypical agents,<br />

which would be especially detrimental to this patient.<br />

If he was treatment resistant, would this change your thinking?<br />

Patients who have failed multiple other antipsychotic medications may benefit<br />

from clozapine—though the risk of agranulocytosis and subsequent FDAmandated<br />

laboratory monitoring limit its widespread use.


SECTION III<br />

Ethics, Health Care,<br />

and Statistics


This page intentionally left blank


CHAPTER 23<br />

Clinical Practice and<br />

Difficult Situations<br />

The USMLE sometimes gives “quote” questions asking you to pick something to<br />

say in a provider-patient interaction. In general, it’s usually a good idea to get more<br />

information, respect the patient’s autonomy, keep information confidential, and<br />

never lie.<br />

What is the importance of rapport?<br />

What are the most important<br />

things to do if a medical error<br />

is committed?<br />

What should you do if a patient tries<br />

to give you a gift?<br />

When is it appropriate to refer a patient<br />

to another physician?<br />

What would you say to a patient with<br />

a terminal illness who wants to die?<br />

The relationship and trust built between<br />

the doctor and patient greatly enhance<br />

the effectiveness of care.<br />

Admit the mistake and apologize for it.<br />

Never try to cover up an error.<br />

You should thank the patient for their<br />

thoughtfulness, but not accept it. Gift<br />

giving may be misinterpreted as<br />

“buying better care”—equal care should<br />

be given to all.<br />

Only when the problem cannot be<br />

handled by yourself. This is rarely the<br />

answer on the USMLE—it is<br />

inappropriate to refer a patient to a<br />

psychiatrist just because they have a<br />

difficult situation to be dealt with!<br />

Get more information about why they<br />

feel this way. Commonly, they are<br />

afraid of dying in pain; they may<br />

have seen a loved one die painfully.<br />

Reassure them that you will stick with<br />

them and that their symptoms will<br />

be well controlled. Also assess for<br />

untreated depression.<br />

177


178 Deja Review: <strong>Behavioral</strong> <strong>Science</strong><br />

Your fellow resident gets called in to<br />

work at night and you smell alcohol<br />

on his breath. He says he only had two<br />

beers and asks you not to say anything.<br />

Do you tell someone?<br />

A patient’s sibling asks you for<br />

information about their condition—<br />

do you give it to them?<br />

Can you tell information about a<br />

patient to someone uninvolved in<br />

their care?<br />

You see a former patient at a bar,<br />

whom you treated briefly 10 years ago.<br />

Can you ask them out on a date now?<br />

When is it OK to withhold information<br />

about an illness from a patient?<br />

A patient asks you to do a procedure<br />

that is legal, but is against your belief<br />

system. Must you do it?<br />

What is a good general approach to an<br />

emotional patient—be it angry, sad,<br />

or scared?<br />

What if the patient starts to cry?<br />

What are the steps to giving bad news?<br />

Yes. It’s unethical to endanger patient<br />

care with a possibly impaired physician.<br />

No. You must have formal permission<br />

from the patient to share their health<br />

information with anyone other than<br />

them.<br />

No. This is a violation of the Health<br />

Insurance Portability and Accountability<br />

Act (HIPPA). You should be careful<br />

where and with whom you discuss<br />

any patient information.<br />

No. It is never ethical to have a<br />

romantic relationship with a patient,<br />

former or current.<br />

If the patient tells you they don’t want<br />

to know. It must be the patient that<br />

indicates this—not a family member.<br />

No, you do not need to do anything nor<br />

treat anyone that would compromise<br />

your beliefs. However, you should refer<br />

the patient to someone who will treat<br />

them, as well as provide support in the<br />

meantime.<br />

Label and validate their emotions, then<br />

offer support. Eg: “You sound like you<br />

are scared—I don’t blame you, it’s OK<br />

to be scared about this procedure.”<br />

Let them cry. Try to be comfortable with<br />

silence when appropriate. Offering a<br />

tissue is always a good gesture.<br />

1. Set the stage—find a private place<br />

and ensure you have a proper<br />

amount of time free.<br />

2. Find out what the patient knows<br />

about his/her illness.<br />

3. Find out how much the patient<br />

wants to know.<br />

4. Tell the information.<br />

5. Respond to feelings.<br />

6. Make a plan for next steps.


Clinical Practice and Difficult Situations 179<br />

Which of the steps above is most likely<br />

to be asked on the USMLE?<br />

How do you use an interpreter?<br />

Is it OK to allow a child or family<br />

member to act as an interpreter?<br />

What is patient adherence (formerly<br />

called “compliance”)?<br />

What are barriers to patient adherence?<br />

How might a physician make a<br />

treatment plan less complicated?<br />

What is an open-ended question?<br />

What is a close-ended question?<br />

Finding out how much the patient<br />

knows about their illness (no 2). It’s<br />

always good to get information first so<br />

you can frame the discussion.<br />

Speak to the patient, not the interpreter.<br />

Speak clearly, being sure to pause<br />

intermittently to allow the interpreter<br />

to convey information to the patient.<br />

No, not if it can be helped. Even if the<br />

family member is fluent, it is good to at<br />

least offer to get an interpreter for the<br />

patient. Family members may also have<br />

their own agenda that may unduly<br />

influence the patient.<br />

The degree to which the patient follows<br />

the advice of the treating physician<br />

Complicated medical regimens<br />

Poor physician-patient rapport<br />

Lack of patient involvement in the<br />

treatment plan<br />

Infrequent appointments<br />

Poor support structure/inadequate help<br />

at home<br />

The treatment plan can be simplified by<br />

limiting:<br />

• The number of medications the<br />

patient is taking<br />

• The number of times the medications<br />

is taken per day<br />

• The number of changes made at each<br />

visit<br />

A question that is intentionally left<br />

broad, such that the patient can say<br />

what is really on their mind—“What<br />

brings you into the clinic today?” or<br />

“Tell me about your stomach pain.”<br />

A very specific question with a discrete<br />

answer. Used to get more detail—<br />

“Did you have nausea?” “How<br />

frequently do you have pain?”<br />

This is also useful with seductive<br />

or disorganized patients.


180 Deja Review: <strong>Behavioral</strong> <strong>Science</strong><br />

What is the cone method of<br />

interviewing, or “coning”?<br />

Starting with an open-ended question,<br />

then progressively narrowing down to<br />

more specific close-ended questions.<br />

CLINICAL VIGNETTES<br />

A 79-year-old woman came into the hospital complaining of shortness of breath.<br />

After an extensive workup, you find that she has metastatic lung cancer. Her son<br />

finds you in the hallway and states “In our culture our elders do not like to know<br />

about serious illnesses. It is my duty as a son to take care of my mother.” You are<br />

unclear about what cultural practices are appropriate to the patient. How do you<br />

approach this situation?<br />

It may very well be that there is a cultural aspect to discussions of terminal illness<br />

that is appropriate to consider in this case. If available in your institution, a “cultural<br />

consult” may be appropriate with someone more versed than you in these issues. If<br />

not, it would be inappropriate to withhold information based solely on what the son<br />

of the patient says. The best case would be to sit down with the patient and ask her<br />

about her cultural beliefs around illness—“Some members of your culture prefer<br />

not to talk about their illnesses, but instead allow their children to make medical<br />

decisions for them. How do you feel about the discussion of illness?”<br />

A patient with known sickle cell disease comes into the ER. He states he is in 10/10<br />

pain and requests 15 mg of morphine. The nurse says he is a “frequent flier” and<br />

worries that he is an addict. He doesn’t look like he’s is in any pain. What should<br />

you do?<br />

Treat his pain. With known disease it is more important that he does not suffer<br />

than to distinguish his symptoms from malingering. Remember, sickle cell crisis<br />

can be extremely painful even without outward signs of pain.<br />

You call a family meeting for a chronically ill patient whom you just found out has<br />

leukemia. Everyone is comfortably sitting in your office. What do you say next?<br />

“What do you know about your illness?” It is important to be open-ended and see<br />

what the patient’s thoughts are. Most likely he has been thinking about this much<br />

more than you and probably is very well informed; or he may have completely<br />

unrealistic views that would be good to learn about before the conversation. Probe<br />

a bit to understand this, and then to ascertain how much he wants to know before<br />

delivering the news.


CHAPTER 24<br />

Medical Ethics and<br />

Legal Issues<br />

What are the four principles<br />

of medical ethics?<br />

What is Autonomy?<br />

What is Beneficence?<br />

What is Nonmaleficence?<br />

What is Justice?<br />

Of these principles, which one tends<br />

to take precedence in ethical questions,<br />

especially on the USMLE?<br />

What is the principle of double-effect?<br />

What is the classic example of<br />

double-effect?<br />

Is malpractice a crime?<br />

1. Patient Autonomy<br />

2. Beneficence<br />

3. Nonmaleficence<br />

4. Justice<br />

The patient has the right to make their<br />

own decisions regarding their care.<br />

The principle that physicians should be<br />

of benefit to their patients.<br />

Do no harm.<br />

The allocation of resources in a fair and<br />

just manner, including equal treatment.<br />

Patient Autonomy<br />

When an action that has a primarily<br />

good effect may also cause an<br />

unintended bad effect, but is still<br />

permissible.<br />

Giving morphine to relieve the pain of a<br />

dying patient, even if it causes respiratory<br />

depression hastening the patient’s<br />

death. As long as the primary intent<br />

of the treatment is to reduce pain and<br />

not to kill the patient, it is ethically<br />

permissible so long as there is pain<br />

to be relieved.<br />

No—malpractice is a civil court matter.<br />

181


182 Deja Review: <strong>Behavioral</strong> <strong>Science</strong><br />

What four elements in a malpractice<br />

case must exist in order to find liability<br />

with the treating physician (otherwise<br />

referred to as the 4 D’s)?<br />

How is a deviation from standard<br />

of care decided upon?<br />

What is the best way to avoid<br />

a malpractice suit?<br />

When can a physician become impaired?<br />

Is it acceptable to work with an<br />

impaired health-care provider?<br />

How does a physician’s human<br />

immunodeficiency virus (HIV) status<br />

affect his or her ability to practice<br />

medicine?<br />

Duty: There must be an established<br />

doctor-patient relationship.<br />

Deviation or dereliction: Treatment<br />

strayed from established standard of care.<br />

Damages: Physical, psychological, or<br />

social damage was done.<br />

Direct cause: Damages were caused<br />

directly by negligence or dereliction.<br />

A jury decides if there has been a<br />

deviation from the standard of care.<br />

The jury makes this determination based<br />

upon testimony of one or more expert<br />

witnesses who testify as to the standard<br />

of care as generally recognized by the<br />

medical community and how the<br />

defendant deviated from that standard<br />

of care.<br />

If an error is made, admit when you are<br />

wrong and apologize to the patient.<br />

Maintain a healthy physician-patient<br />

relationship.<br />

Keep current with standards of care.<br />

When clinical judgment is affected by<br />

the following:<br />

• Physical illness<br />

• Mental illness<br />

• Substance abuse<br />

No. It is the ethical duty of a physician<br />

to report an impaired health-care<br />

provider to the proper authorities.<br />

Under the American Medical Association<br />

(AMA) ethical guidelines, an HIV<br />

physician should not engage in any<br />

activity that would put a patient at risk<br />

of contracting HIV (eg, an HIV-positive<br />

surgeon may be precluded from<br />

practicing surgery. Some states require<br />

that an HIV-positive physician disclose,<br />

as part of the informed consent process,<br />

his or her HIV status before engaging in<br />

an invasive procedure that would put<br />

the patient at risk).


Medical Ethics and Legal Issues 183<br />

Is it acceptable for a physician to<br />

establish a romantic relationship<br />

with a patient or former<br />

patient?<br />

What is competence?<br />

Who is considered legally competent?<br />

Who is an emancipated minor?<br />

Can a person be competent in some<br />

areas and not others?<br />

Can a physician deem a patient<br />

incompetent?<br />

If a patient is ruled incompetent, how<br />

does the patient become competent<br />

again?<br />

No—it’s not acceptable under any<br />

circumstances.<br />

Competence is the ability to evaluate<br />

situations and make sound judgments<br />

that are:<br />

• Consistent throughout time<br />

• Consistent with the patient’s belief<br />

system (unless the belief system is<br />

delusional)<br />

All adults >18 years of age and<br />

emancipated minors—including<br />

adults with mental illness or mental<br />

retardation—unless declared<br />

incompetent by a court of law<br />

Someone


184 Deja Review: <strong>Behavioral</strong> <strong>Science</strong><br />

Who may give informed consent<br />

for a patient?<br />

What happens if a patient lacks<br />

the capacity to make, understand, or<br />

communicate his or her health-care<br />

decisions and has not named a healthcare<br />

agent under a durable power<br />

of attorney for health care or been<br />

appointed a legal guardian by<br />

the court?<br />

What is an advance directive?<br />

What is a durable power of attorney<br />

for health care?<br />

The patient only unless:<br />

• It is an emergency, delay puts the<br />

patient’s life at risk, and there is no<br />

one available to give consent on the<br />

patient’s behalf.<br />

• The patient is legally incompetent<br />

(depending upon state law, the<br />

legally appointed guardian, or the<br />

agent named in a durable power of<br />

attorney for health care gives<br />

consent).<br />

• The patient lacks the capacity to<br />

make, understand, or communicate<br />

his or her health-care decisions (a<br />

surrogate decision-maker gives<br />

consent as provided by state law,<br />

usually in the following order of<br />

priority: health-care agent, spouse, an<br />

adult child, parent, an adult sibling,<br />

or a grandparent).<br />

• The patient is a nonemancipated<br />

minor (parent or legal guardian gives<br />

consent, and in some states, someone<br />

standing in the place of a parent may<br />

also give consent).<br />

A surrogate decision-maker may give<br />

consent on the patient’s behalf and<br />

should make a good-faith effort to make<br />

decisions based upon what the patient<br />

would have chosen. State law designates<br />

a list of surrogate decision-makers,<br />

usually in the following order of priority<br />

(in the absence of a health-care agent or<br />

legal guardian): spouse, adult child,<br />

parent, adult sibling, or a grandparent.<br />

State laws may differ.<br />

A decision made by a patient about<br />

what type of medical care he or she<br />

wishes to receive in case that he or<br />

she is not able to make decisions in the<br />

future.<br />

A durable power of attorney for health<br />

care (or health-care proxy) is a legal<br />

document that allows the patient to<br />

designate a health-care agent to make<br />

health-care decisions on behalf of the<br />

patient when and if the patient is unable<br />

to do so. It is an advance directive.


Medical Ethics and Legal Issues 185<br />

What is a living will?<br />

What is a DNR (do not resuscitate) order?<br />

Do DNR orders mean no interventions<br />

are done on a patient?<br />

When does a judge consent for medical<br />

treatment of a minor?<br />

Does this apply to a fetus?<br />

Under what circumstances can a<br />

nonemancipated minor receive<br />

treatment without the consent<br />

of his or her parent or legal guardian?<br />

A living will is a legal document that<br />

allows a patient to decide, in advance,<br />

whether he or she wants to be kept<br />

alive by artificial means if two doctors<br />

diagnose that the patient is (1) terminally<br />

and incurably ill, (2) in a persistent<br />

vegetative state, or (3) in an irreversible<br />

coma. State laws may differ as to<br />

conditions under which a living will<br />

may be honored.<br />

A DNR order is an order written<br />

by a physician, after determining<br />

whether the patient is a candidate<br />

for nonresuscitation and obtaining the<br />

appropriate consent that directs medical<br />

personnel not to resuscitate a patient in<br />

the event of cardiopulmonary arrest.<br />

No. Patients may choose to still have<br />

antibiotics, fluids and nutrition, or other<br />

“non-heroic” means depending on their<br />

wishes.<br />

When a parent or legal guardian refuses<br />

to consent to medical treatment on the<br />

minor’s behalf, and the physician<br />

believes that the treatment is medically<br />

necessary and justifies court intervention.<br />

A court is more likely to intervene when<br />

the proposed treatment carries a low<br />

risk and high benefit or when the<br />

minor’s life is threatened.<br />

No. The competent pregnant mother, in<br />

most circumstances, has a right to refuse<br />

any intervention on the part of the fetus<br />

even if it compromises her own or the<br />

fetus’ life. In some states, courts have<br />

intervened on behalf of a viable fetus.<br />

Although the answer is state dependent,<br />

most states allow minors to consent for<br />

treatment involving sexually transmitted<br />

diseases (STDs), contraception and<br />

pregnancy, and alcohol or illegal<br />

substance use. Some states also allow<br />

minors to consent for an abortion.<br />

Other states allow a minor to consent<br />

to an abortion, but require parental<br />

notification or a court order waiving<br />

parental notification.


186 Deja Review: <strong>Behavioral</strong> <strong>Science</strong><br />

When can a patient refuse treatment<br />

even though refusal is life threatening?<br />

Does this include artificial<br />

life support?<br />

Is removing artificial life support the<br />

same as physician-assisted suicide?<br />

When can a physician decide to remove<br />

life support without the consent of the<br />

patient or patient’s decision-maker?<br />

Is palliative care of the terminal patient<br />

the same as euthanasia?<br />

What is an informed consent?<br />

Does consent have to be written?<br />

Anytime, as long as they have the<br />

capacity to refuse the treatment, and<br />

the refusal is an informed refusal.<br />

Yes<br />

No. Removing artificial life support is a<br />

decision that the competent, informed<br />

patient is allowed to make. It does not<br />

accelerate the natural course of the<br />

patient’s disease process. Physicianassisted<br />

suicide is illegal in most states<br />

because it purposely accelerates<br />

death.<br />

A physician may remove life support<br />

without consent if the patient is legally<br />

dead. The patient must be “brain dead”<br />

in order to be declared legally dead in<br />

the United States, which includes global<br />

dysfunction of the brain (coma) and<br />

absent brainstem reflexes.<br />

No. Palliative care is not done with the<br />

intention of accelerating death. It is<br />

done with the intention of making the<br />

patient comfortable through the natural<br />

end point of a terminal illness. However,<br />

palliative care may unintentionally<br />

accelerate death as a side effect.<br />

Informed consent is required if the<br />

proposed treatment or procedure<br />

involves a material risk to the patient.<br />

An informed consent includes the<br />

voluntary agreement by a patient to<br />

proceed with treatment after the<br />

physician has discussed the procedure,<br />

risks, benefits, and outcome of the<br />

procedure or treatment, alternative<br />

treatments (including no treatment),<br />

and the risk, benefits, and outcomes<br />

of those alternatives.<br />

No. Basic consent (eg, consent to<br />

touching contact during a physical<br />

examination) need not be in writing and<br />

is often implied. An informed consent<br />

should be in writing and documented<br />

in the chart.


Medical Ethics and Legal Issues 187<br />

Can a physician refuse to treat patients<br />

based on race, financial status, and<br />

presence of mental illness or<br />

HIV status?<br />

If a patient is homicidal, is a physician<br />

allowed to break patient confidentiality?<br />

In what other circumstances is a<br />

physician required to break patient<br />

confidentiality by law?<br />

Generally, a physician may refuse to<br />

treat a patient as long as the reasons for<br />

refusal are not illegal. Illegal reasons to<br />

refuse treatment include race, national<br />

origin, gender, religion, and disability<br />

(which include mental illness and<br />

HIV status). Except in emergency<br />

situations, a physician may refuse to<br />

treat a patient based upon inability<br />

to pay.<br />

Yes. In almost all states, the Tarasoff<br />

decision applies, which requires a<br />

physician to warn the person in severe<br />

danger and notify law enforcement. In<br />

states such as Georgia, Tarasoff has not<br />

been adopted. Instead, the physician<br />

has a duty to prevent harm by the<br />

patient if there is a right to control. In<br />

other words, if a physician has the<br />

legal right to initiate involuntarily<br />

commitment proceedings and fails to<br />

exercise this right, the physician may be<br />

held liable for harm done by the patient<br />

to third parties if the harm was<br />

foreseeable. Georgia has not specifically<br />

recognized a duty to warn, and<br />

thus, a physician may be breaching<br />

confidentiality in warning an<br />

intended victim.<br />

• A patient is considered suicidal<br />

• Reporting child abuse, elder abuse,<br />

or domestic violence (call Child<br />

Protective Services, Adult Protective<br />

Services, or the police)<br />

• Court order (except to the extent the<br />

information is privileged)<br />

• Patient driving without cognitive<br />

abilities to do so (some states<br />

have specific procedures and<br />

forms)<br />

• Reporting certain infectious<br />

diseases, including HIV/acquired<br />

immunodeficiency syndrome<br />

(AIDS)


188 Deja Review: <strong>Behavioral</strong> <strong>Science</strong><br />

In what other circumstances<br />

can a physician breach patient<br />

confidentiality?<br />

When can a patient be “committed”<br />

or involuntarily hospitalized?<br />

If a patient is involuntarily hospitalized,<br />

can a physician administer any<br />

treatment they want?<br />

How do mentally ill patients receive<br />

treatment even if they refuse treatment?<br />

If the patient signs a written authorization,<br />

the physician may disclose private<br />

health information consistent with the<br />

authorization. State law may also<br />

authorize a physician to share private<br />

health information with specified people<br />

who are at risk for contracting HIV from<br />

a patient. Disclosure of this information<br />

is controlled by state law, and physicians<br />

must proceed carefully because if state<br />

law does not authorize disclosing<br />

HIV/AIDS information, the physician<br />

may be guilty of violating Health<br />

Insurance Portability and Accountability<br />

Act of 1996 (HIPAA) which may result<br />

in fines and a prison sentence.<br />

The process varies by state, but a patient<br />

who presents a substantial risk of<br />

imminent harm to himself or herself or<br />

others, or if a patient is so unable to care<br />

for his or her own physical safety as to<br />

create an imminently life-endangering<br />

crisis, then the patient needs involuntary<br />

inpatient treatment.<br />

No. Even an involuntarily committed<br />

patient has the right to refuse medical<br />

treatment.<br />

A judge can order the administration<br />

of treatment if the patient is found<br />

incompetent to refuse treatment. Also,<br />

if the patient is violent and posing an<br />

immediate, severe danger to himself or<br />

herself or others, this is considered a<br />

medical emergency and the physician<br />

may administer treatment without the<br />

patient’s consent.<br />

CLINICAL VIGNETTES<br />

Your patient has metastatic lung cancer and he is nearing the last weeks of his life.<br />

He is in considerable pain and is having trouble breathing without being<br />

intubated. His wishes were to not be intubated or having any life-prolonging<br />

measures. His son asks if you can do anything for his pain. You ask the nurse<br />

about giving him more morphine and she says that the respiratory depression will<br />

kill him. Is it ethical to do this?<br />

Yes—it is ethically permissible under the principle of double-effect even if it<br />

causes his death, as long as your primary goal is to treat his pain.


Medical Ethics and Legal Issues 189<br />

A 72-year-old woman comes in with a gangrenous foot. The orthopedic service<br />

would like to amputate the foot to save her life. You are worried about the risks<br />

involved in the surgery. The man in the bed next door also needs a surgical<br />

procedure, but he is unable to afford it. The woman with the gangrenous foot<br />

doesn’t want the surgery because she wants to keep herself “whole.” Of the four<br />

parties involved in this story (Patient, Orthopedic service, the man next door, and<br />

you), which ethical principles is each representing?<br />

Patient: Patient Autonomy<br />

Orthopedic service: Beneficence<br />

Man next door: Justice<br />

You: Nonmaleficence


This page intentionally left blank


CHAPTER 25<br />

Health Care in the<br />

United States<br />

HEALTH-CARE INSURANCE<br />

Yes—they vary, but there are out-<br />

of-pocket expenses for Medicare.<br />

What is the primary difference in<br />

health-care insurance coverage in the<br />

United States as compared to other<br />

industrialized countries?<br />

What options for insurance coverage<br />

are there and how do most people<br />

get their coverage?<br />

What is Medicaid?<br />

What is Medicare?<br />

What are the “Parts” of Medicare?<br />

Does Medicare have a premium,<br />

co-pay, and/or deductible?<br />

What is the group of people most likely<br />

to be uninsured or have no affordable<br />

available coverage?<br />

The United States is the only industrialized<br />

country without government-funded<br />

health care for all citizens.<br />

Employer-Sponsored Programs<br />

(most common)<br />

Individual Policy (self-purchased)<br />

Medicare<br />

Medicaid<br />

Government-funded program for<br />

low-income people below a certain<br />

income<br />

Government-funded program for those<br />

>65 years or those who are disabled or<br />

those on dialysis<br />

Part A: Hospital coverage<br />

Part B: Outpatient coverage<br />

Part C: Private supplement to A and B<br />

Part D: Prescription drug coverage<br />

Those who are employed, make too much<br />

money to qualify for Medicaid, and do<br />

not receive benefits through their<br />

employer and/or cannot afford<br />

premiums.<br />

191


192 Deja Review: <strong>Behavioral</strong> <strong>Science</strong><br />

How many people do not have health<br />

insurance in the United States?<br />

What are some of the disadvantages<br />

of self-purchased individual<br />

health-care plans?<br />

What is “pooled risk”?<br />

What is fee-for-service?<br />

Who pays more for the same<br />

service—an uninsured patient<br />

paying out of pocket or an insurance<br />

company?<br />

What is a Diagnosis-related Group<br />

(DRG)?<br />

What two health-care costs are not<br />

covered by Medicare?<br />

Which government-funded insurance<br />

plan is funded completely by the<br />

federal government?<br />

What are the primary sources of funding<br />

for health-care insurance for US citizens?<br />

Which not-for-profit insurance carrier<br />

provides insurance for 30% to 50%<br />

of working citizens in all 50 states?<br />

The number continues to rise, but<br />

approximately 45 million plus people.<br />

It may be very difficult to qualify for<br />

these if you have any illnesses. They<br />

can also be prohibitively expensive.<br />

Insurance companies make money by<br />

having the average premium of their<br />

insured group be less than the total<br />

amount they pay out for that same<br />

group. Each person has a certain<br />

amount of “risk” for developing<br />

disease. Combining the risk to each<br />

person in a group is “pooling risk.”<br />

Each service that is done is billed<br />

separately (eg, a doctor visit, x-ray,<br />

lab test, and surgical procedure are<br />

each charged for separately).<br />

The uninsured patient. Insurance<br />

companies negotiate discounted rate<br />

with physicians and hospitals—if a<br />

patient pays out of pocket, they have<br />

to pay full price.<br />

With DRGs, a hospital is paid a set<br />

amount for an individual diagnosis, no<br />

matter how long it takes to get better.<br />

For example, Patient A gets pneumonia<br />

and stays in the hospital 2 days, Patient B<br />

gets pneumonia also and takes a 10-day<br />

stay to recover. Medicare pays the hospital<br />

the same $1500 for Patient A as for<br />

Patient B.<br />

1. Long-term nursing care<br />

2. Outpatient prescription drugs<br />

Medicare. Medicaid receives funding<br />

from the federal and state governments<br />

both.<br />

Employee benefit.<br />

People obtain health-care insurance on<br />

their own.<br />

Blue Cross/Blue Shield<br />

Note: Blue Cross covers hospital costs.<br />

Blue Shield covers diagnostic tests and<br />

physicians’ fees.


Health Care in the United States 193<br />

What are the two primary health plans<br />

offered by private health-care insurers?<br />

Which type of health-care insurance<br />

plan has high premiums and does not<br />

impose restrictions on provider’s<br />

choice?<br />

What are the primary characteristics<br />

of a health-care managed-care plan?<br />

What are the types of health-care<br />

managed-care plans?<br />

What is a health maintenance<br />

organization (HMO)?<br />

What is meant by a physician being a<br />

“gatekeeper”?<br />

What are preferred provider<br />

organizations (PPOs)?<br />

1. Fee-for-service plans<br />

2. Managed-care plan<br />

Fee-for-service plans<br />

Low premiums<br />

Restrictions on provider’s choice<br />

Health maintenance organization<br />

(HMO)<br />

Preferred provider organizations (PPOs)<br />

Point of service (POS)<br />

An HMO has arrangements with<br />

health-care providers to form a<br />

“network.” The providers give the<br />

HMO a discounted rate in exchange for<br />

receiving health plan referrals. Members<br />

may only see physicians in this network,<br />

otherwise the costs are not be covered<br />

by the plan. Members select a primary<br />

care physician (PCP), often called a<br />

“gatekeeper,” who provides, arranges,<br />

coordinates, and authorizes all aspects<br />

of the member’s health care.<br />

Often the primary care physician (PCP)<br />

in an HMO plan must approve specialist<br />

and ancillary service referrals as well as<br />

coordinate the member’s care.<br />

PPOs are similar to HMOs in that they<br />

have arrangements with a “provider<br />

network.” Unlike an HMO, members<br />

may choose a doctor outside of this<br />

network, but their coverage will be at a<br />

reduced rate (ie, out-of-pocket expenses<br />

are higher).<br />

HEALTH-CARE COSTS<br />

How does the United States compare<br />

with other countries in terms of<br />

health-care spending?<br />

The United States spends significantly<br />

more per capita and as a percentage<br />

of GDP on health care than any other<br />

nation.


194 Deja Review: <strong>Behavioral</strong> <strong>Science</strong><br />

How is the US life expectancy<br />

compared with other countries?<br />

Which factors have contributed to the<br />

increase in health-care expenditures?<br />

During what time in a person’s life<br />

span is the most money spent?<br />

What are the most expensive<br />

components of health care in the<br />

United States (rank items from<br />

most expensive to least expensive)?<br />

The 2009 CIA (Central Intelligence<br />

Agency) World Factbook ranks<br />

United States at no. 50 out of 224—<br />

despite the increased spending.<br />

Larger percentage of elderly individuals<br />

Overuse of medical technology<br />

advances<br />

Medicaid and Medicare expenditures<br />

The last year of their life<br />

1. Hospitalization<br />

2. Physician costs<br />

3. Nursing home costs<br />

4. Prescription drugs<br />

5. Medical supplies<br />

6. Mental health services<br />

7. Dental and other care<br />

HEALTH-CARE DELIVERY SYSTEMS<br />

What is the role of nursing homes?<br />

To provide long-term care, especially for<br />

individuals aged 65 years and older<br />

What percentage of the elderly population 5%<br />

uses nursing home services?<br />

What is the range of costs spent on an<br />

individual that resides in a nursing<br />

home?<br />

What is hospice care?<br />

What are the goals of hospice and<br />

palliative care?<br />

What is “comfort care” and do hospice<br />

patients have to be on it?<br />

$35,000 to $75,000—depending upon<br />

nursing home level of care<br />

A unique type of service provided to<br />

people with terminal illness and a life<br />

expectancy thought to be


Health Care in the United States 195<br />

Is hospice expensive to the health-care<br />

system?<br />

No—patients generally receive more<br />

services with less expenditure than if<br />

they were to be repeatedly hospitalized.<br />

HEALTH STATUS AND DETERMINANTS<br />

What is the percentage of physical Approximately 70%<br />

illness that is due to individual<br />

patterns of living (eg, lack of exercise,<br />

poor dietary choices, and smoking)?<br />

What are the primary determinants<br />

of socioeconomic status?<br />

Which socioeconomic group tends<br />

to delay seeking health care and<br />

present with more progressive<br />

illnesses?<br />

Which gender is most likely to seek<br />

medical care?<br />

Which gender has the lowest life<br />

expectancy?<br />

Which group of people has the lowest<br />

life expectancy with regards to race<br />

and gender?<br />

Which group of people has the highest<br />

life expectancy with regards to race<br />

and gender?<br />

Which group of people represents<br />

the fastest growing segment of the<br />

US population?<br />

Education level<br />

Income<br />

Occupation<br />

Residence<br />

People of low socioeconomic status →<br />

due to lack of funds for health care<br />

Female<br />

Male<br />

Black males<br />

Asian women<br />

Elderly<br />

What is the percentage of health- 30%<br />

care costs the elderly population is<br />

responsible for?<br />

Which ethnic minority represents the<br />

largest percent of the US population?<br />

What are the leading causes of death<br />

in the United States, irrespective of<br />

age group?<br />

Hispanic<br />

Heart disease<br />

Cancer<br />

Stroke


196 Deja Review: <strong>Behavioral</strong> <strong>Science</strong><br />

What are the three leading causes<br />

of death in infants


Health Care in the United States 197<br />

A 50-year-old woman is shopping for insurance. She has rheumatoid arthritis and<br />

takes some very expensive monoclonal-antibody agents which keep her very<br />

functional, and would like to keep her own doctor. She is employed making<br />

$60,000 a year and has the option of insurance through her employer. Which<br />

options are best for her: employer-sponsored insurance, individually purchased<br />

plan, Medicare, or Medicaid? HMO or PPO?<br />

She is neither 65 years of age or older nor disabled, so would not qualify for<br />

Medicare. Similarly, she is not low income and would not be eligible for Medicaid.<br />

Since she has a preexisting condition of rheumatoid arthritis for which she takes<br />

expensive medication, an individual plan may be difficult for her to obtain. As<br />

such, employer-sponsored health insurance would be best. The best way to keep<br />

her own doctor would likely be through a PPO, where she could go out of the<br />

provider network if needed to stay with her doctor.


This page intentionally left blank


CHAPTER 26<br />

Epidemiology and<br />

Research Design<br />

EPIDEMIOLOGY<br />

What is prevalence?<br />

What is incidence?<br />

What equation represents<br />

the relationship between<br />

prevalence and incidence?<br />

What happens to the prevalence<br />

of a disease as the duration of it is<br />

lengthened, assuming the incidence<br />

is held constant?<br />

Which is usually greater in chronic<br />

diseases—prevalence or incidence?<br />

Which is more useful for describing<br />

influenza over the course of a year—<br />

incidence or prevalence?<br />

The total number of individuals who<br />

have a disease in a population at a<br />

specific time or period of time, divided<br />

by the total number of people in that<br />

population.<br />

The total number of individuals who<br />

are newly diagnosed with a disease<br />

divided by the number of individuals<br />

who are originally at risk (usually<br />

measured over a time period).<br />

Prevalence = incidence × disease duration<br />

P = I × D<br />

It would increase—use the equation.<br />

Prevalence is usually greater with long<br />

duration diseases, eg, HIV.<br />

Incidence. With acute illnesses, duration<br />

is short and prevalence at any given<br />

time is not of as much value as incidence.<br />

The shorter the illness, the closer<br />

prevalence becomes to being equal<br />

to incidence.<br />

199


200 Deja Review: <strong>Behavioral</strong> <strong>Science</strong><br />

What is Sensitivity?<br />

What is Specificity?<br />

What is best for a screening test<br />

or to “rule out” disease (low false<br />

negative)?<br />

What is best for a confirmatory<br />

test or to “rule in” disease<br />

(low false positive)?<br />

What is a good mnemonic to<br />

remember this?<br />

What is positive predictive value (PPV)?<br />

What is negative predictive value (NPV)?<br />

What is the relation between sensitivity,<br />

specificity, NPV, and PPV?<br />

What is the difference between<br />

sensitivity, specificity, NPV,<br />

and PPV?<br />

How do sensitivity and specificity<br />

affect NPV and PPV?<br />

How does prevalence of disease in a<br />

population affect PPV and NPV?<br />

What is the best way to solve problems<br />

requiring the calculation of sensitivity,<br />

specificity, PPV, and NPV?<br />

The ability of a test to detect disease if it<br />

is present<br />

The ability of a test to discern one<br />

disease from another<br />

A test with a high sensitivity<br />

True negatives divided by all of the<br />

people without a disease or illness<br />

SPin (SPecificity rules in) and SNout<br />

(SeNsitivity rules out)<br />

The likelihood that a positive result on a<br />

test represents actual disease<br />

The likelihood that a negative result on<br />

a test actually represents the absence of<br />

disease<br />

Sensitivity and specificity are directly<br />

related to NPV and PPV (respectively),<br />

but NPV and PPV occur when a test is<br />

applied to a specific population.<br />

Sensitivity and specificity are inherent<br />

properties of a test, regardless of the<br />

population’s disease prevalence. PPV<br />

and NPV are directly dependent on<br />

the population being tested.<br />

A high sensitivity will increase NPV of<br />

a test, a high specificity will increase the<br />

PPV of a test.<br />

High prevalence of disease will increase<br />

the PPV and decrease the NPV of a test.<br />

Low prevalence will decrease the PPV<br />

and increase the NPV. (This should<br />

make sense!)<br />

Use the box method—draw it out and<br />

plug in the numbers for each problem.


Epidemiology and Research Design 201<br />

+<br />

Actual disease state<br />

–<br />

+<br />

a<br />

b<br />

Test result<br />

–<br />

c<br />

d<br />

Sensitivity = a/(a + c)<br />

Specificity = d/(b + d )<br />

PPV = a/(a + b)<br />

NPV = d/(c + d)<br />

a = True positives<br />

b = False positives<br />

c = False negatives<br />

d = True negatives<br />

All patients with disease = (a + c)<br />

All patients without disease = (b + d )<br />

All positive tests = (a + b)<br />

All negative tests = (c + d )<br />

Figure 26.1<br />

How is sensitivity calculated?<br />

How is specificity calculated?<br />

How is positive predictive value<br />

calculated?<br />

How is negative predictive value<br />

calculated?<br />

True positives divided by all of the<br />

individuals with a disease<br />

Sensitivity = a/(a + c)<br />

True negatives divided by all of the<br />

people without a disease<br />

Specificity = d/(b + d)<br />

True positives divided by all people<br />

with a positive test<br />

PPV = a/(a + b)<br />

True negatives divided by all people<br />

with a negative test<br />

NPV = d/(c + d)<br />

RESEARCH STUDY DESIGNS<br />

What is a prospective study?<br />

A research study that starts before<br />

the studied disease or outcome has<br />

happened, and usually monitors for<br />

its development.


202 Deja Review: <strong>Behavioral</strong> <strong>Science</strong><br />

What is a retrospective study?<br />

What is meant by a study being<br />

“observational”?<br />

What is meant by a study being<br />

“experimental”?<br />

What is a case-control study?<br />

What is a cohort study?<br />

What are the two types of cohort studies?<br />

What is a prospective cohort study?<br />

What is a retrospective cohort study?<br />

A research study that starts after the<br />

studied disease or outcome has<br />

happened, and usually looks back<br />

for factors that led to it.<br />

The researcher separates subjects into<br />

groups and merely waits for the<br />

outcome in question to happen.<br />

The researcher separates subjects into<br />

groups and applies an intervention to<br />

one or more of those groups, hoping<br />

to modify the outcome.<br />

An observational study that compares<br />

subjects who have a disease or outcome<br />

(cases) with subjects who do not have<br />

an illness or outcome (controls).<br />

Note: Groups are divided based on<br />

disease presence (outcome).<br />

An observational study that compares<br />

subjects with and without a certain risk<br />

factors or exposure. This study then<br />

follows subjects for the development<br />

of disease.<br />

Note: Groups are divided based on risk<br />

factor (exposure).<br />

1. Prospective<br />

2. Retrospective<br />

A prospective cohort study evaluates a<br />

cohort of individuals after they have<br />

experienced the risk factor/exposure,<br />

but before development of disease/<br />

outcome. (Eg, a study is constructed to<br />

evaluate whether children exposed to<br />

secondhand smoke at birth will be more<br />

susceptible to lung cancer than those<br />

children not exposed, started when<br />

children are 5 years old).<br />

A retrospective cohort study evaluates a<br />

cohort of individuals after both the risk<br />

factor/exposure and the disease/outcome<br />

have happened. (Eg, a study of if children<br />

exposed to secondhand smoke at birth<br />

will be more susceptible to lung cancer<br />

than those children not exposed, started<br />

at age 80).


Epidemiology and Research Design 203<br />

What is a cross-sectional study?<br />

What is a clinical trial?<br />

What are two primary characteristics<br />

of high-quality clinical trials?<br />

A descriptive study (neither observational<br />

nor experimental) that compares subjects<br />

in respect to both risk factor/exposure<br />

and disease/outcome at one specific point<br />

in time.<br />

An experimental study in which groups<br />

of subjects are given different treatments<br />

or interventions to determine if there is<br />

an effect.<br />

Note: Often there is an experimental<br />

group that receives the studied<br />

intervention and a control group that<br />

receives either placebo or a different<br />

“standard” treatment.<br />

They are double-blinded and patients<br />

are randomized to be in either the<br />

experimental or control group.<br />

TESTING<br />

What are the attributes of useful testing<br />

instruments?<br />

Name the types of bias that may be<br />

prevalent in a research study.<br />

Which bias can occur when hospital A<br />

admits sicker patients than hospital B?<br />

Which bias can occur when a disease<br />

or illness is detected earlier, leading<br />

to seemingly increased survival time?<br />

Which bias can occur when people fail<br />

to return surveys or respond to a phone<br />

or email survey?<br />

Which bias can occur when a study<br />

favors selecting subjects that have<br />

a particular characteristic or set of<br />

characteristics?<br />

Lacks bias<br />

Reliably Valid<br />

Admission rate bias<br />

Lead time bias<br />

Nonresponse bias<br />

Sampling bias<br />

Selection bias<br />

Admission rate bias<br />

Lead time bias<br />

Nonresponse bias<br />

Sampling bias


204 Deja Review: <strong>Behavioral</strong> <strong>Science</strong><br />

Which bias can occur if the subjects<br />

studied are not representative of the<br />

target population about which<br />

conclusions are drawn?<br />

What percentage of patients generally<br />

respond to placebos?<br />

What happens to the response rate to<br />

placebos in psychiatric conditions?<br />

What is the definition of a doubleblind<br />

study?<br />

If initially group A receives a treatment<br />

and group B receives a placebo and<br />

later the protocol is switched so that<br />

group A receives the placebo and<br />

group B receives a treatment,<br />

what type of research study is<br />

being utilized?<br />

What is reliability?<br />

What type of reliability is demonstrated<br />

when different examiners are able to<br />

achieve test results that are similar?<br />

What type of reliability is demonstrated<br />

when subsequent tests yield similar<br />

results to initial tests?<br />

What is the definition of validity?<br />

What is the definition of precision?<br />

What is the definition of accuracy?<br />

Selection bias<br />

Usually at least 33% of patients<br />

It increases in psychiatric illnesses.<br />

A study in which neither the research<br />

scientist nor the subject knows which<br />

participants are in the experimental<br />

group and which are in the control<br />

group.<br />

Crossover studies<br />

It is the reproducibility of a given test.<br />

Interrater reliability<br />

Test-retest reliability<br />

It determines whether a test measures<br />

what it is supposed to measure.<br />

It is the consistency and reproducibility<br />

of a test. On a dartboard, it would be<br />

hitting the same place over and over<br />

(though not necessarily the correct<br />

place).<br />

It determines how true test measurements<br />

are. On a dartboard, it would be hitting<br />

the correct place (though perhaps not<br />

consistently).


Epidemiology and Research Design 205<br />

MEASURES OF ASSOCIATION<br />

Which measures are used to quantify<br />

risks in population studies?<br />

Which measure(s) of association is used<br />

to evaluate cohort studies?<br />

Which measure(s) of association is used<br />

to evaluate case-control studies?<br />

How does our four-box method<br />

for calculating specificity and sensitivity<br />

change when calculating risk?<br />

Relative risk<br />

Attributable risk<br />

Odds ratio<br />

Relative risk<br />

Attributable risk<br />

Odds ratio<br />

The left heading of “test result” is<br />

changed to “exposure/risk factor”<br />

(the top heading stays the same).<br />

+<br />

Disease state<br />

–<br />

+<br />

Exposure/risk<br />

factor<br />

–<br />

a<br />

c<br />

b<br />

d<br />

RR = ([a/(a + b)] / [c/(c + d)])<br />

AR = ([a/(a + b)] – [c/(c + d)])<br />

OR = ad/bc<br />

a = Exposed subjects with disease<br />

b = Exposed subjects without disease<br />

c = Unexposed subjects with disease<br />

d = Unexposed subjects without disease<br />

All with disease = (a + c)<br />

All without disease = (b + d )<br />

All exposed = (a + b)<br />

All unexposed = (c + d )<br />

Figure 26.2<br />

What is relative risk (RR)?<br />

How is RR calculated?<br />

RR is a measure of the likelihood of<br />

disease in exposed subjects compared<br />

to unexposed. It is expressed as a ratio,<br />

so an RR = 1.0 is the risk of the<br />

unexposed subjects.<br />

It is the incidence of disease in the<br />

exposed group divided by the incidence<br />

of disease in the unexposed group.<br />

RR = [a/(a + b)]/[c/(c + d)]<br />

At what value is RR significant? When RR is greater than or less than 1.0


206 Deja Review: <strong>Behavioral</strong> <strong>Science</strong><br />

If the RR is >1, what can be said about<br />

the risk?<br />

What is the definition of Attributable<br />

Risk (AR)?<br />

How is AR calculated?<br />

If the incidence rate of atherosclerosis<br />

in the general population in Atlanta,<br />

GA is 10/100 and in individuals with a<br />

high cholesterol diet is 50/100, what is<br />

the attributable risk?<br />

What is an odds ratio (OR)?<br />

How can you interpret odds ratio in<br />

comparison to relative risk?<br />

There is an increased risk of the disease<br />

or illness.<br />

If the RR is 0.5).<br />

or OR crosses 1?<br />

CLINICAL VIGNETTES<br />

How have advances in antiretroviral therapies influenced the prevalence of HIV,<br />

assuming incidence of HIV has remained constant?<br />

HIV has an increased prevalence due to the lengthening of life expectancy of<br />

infected patients. Using P = I × D, and assuming the medications have little effect<br />

on incidence, as duration is increased, so is prevalence. (Note that HAART does<br />

significantly decrease the transmission of HIV.)<br />

If you are told that almost all children with chickenpox have a fever, but that fever<br />

can be a feature of many illnesses. How would you describe fever in terms of<br />

sensitivity and specificity in regards to chickenpox?<br />

Sensitivity would be high (someone without fever would be unlikely to have<br />

chickenpox), specificity would be low (fever could be from many diseases).


Epidemiology and Research Design 207<br />

Two groups of male patients are looked at over a 10-year period, ones that like to<br />

go to the doctor regularly for checkups and ones that don’t get much care at all. It<br />

is found that the group which goes to the doctor regularly has a longer survival<br />

time after diagnosis of prostate cancer than the ones who don’t get much care.<br />

What type of study design is this? What type of measure of risk would you use to<br />

compare the groups? What is a likely potential type of bias in this study?<br />

This is a cohort study—it separates groups based on the risk factor/exposure of<br />

health-care utilization and then monitors for the development of disease and<br />

death. Cohort studies use relative risk measures (as opposed to case-control<br />

studies which use odds ratios). This study likely suffers from lead time bias as the<br />

survival time after diagnosis is directly dependent on how early a patient is<br />

screened for disease—likely more often for those going to the doctor regularly.<br />

A new laboratory test for depression is developed. Nearly 2500 patients are tested,<br />

800 with depression and 1700 without depression. There are 720 true positives,<br />

1500 true negatives, 80 false negatives, and 200 false positives. What is the<br />

sensitivity of test? What is the specificity of the test? If a patient from this same<br />

population tests positive for depression, how certain are you that the patient<br />

actually has depression? How about if the same patient tested negative—how sure<br />

are you that he/she doesn’t have depression?<br />

First off, make a four-box diagram, as done below. You should start with this every<br />

time. The bold numbers are row and column totals. Remember that a is true<br />

positives, b is false positives, c is false negatives, and d is true negatives.<br />

Disease<br />

+ −<br />

Exposure + 720 200 920<br />

_ 80 1500 1580<br />

800 1700 2500<br />

For the first two questions, plug in numbers. Sensitivity = 720/(80 + 720) = 90%.<br />

Specificity = 1500/(1500 + 200) = 88%. The second two questions are really asking<br />

you to calculate the positive predictive value and negative predictive value,<br />

respectively. PPV = 720/(720 + 200) = 78%. NPV = 1500/(1500 + 1580) = 95%.


This page intentionally left blank


CHAPTER 27<br />

Biostatistics<br />

STATISTICAL DISTRIBUTION<br />

What are the three measures of central<br />

tendency?<br />

What is the definition of mean?<br />

What is the definition of median?<br />

What is the definition of mode?<br />

1. Mean<br />

2. Median<br />

3. Mode<br />

The average of a set of numbers<br />

The middle number in a set of numbers<br />

when they are put in sequential order<br />

Note: If there is an even amount of<br />

data in a set, the median is the<br />

average of the two middle values<br />

in the data set.<br />

The number that appears most<br />

frequently in a set of numbers.<br />

Using the following set of numbers, Mean = [(1 + 2 + 3 + 4 + 5 + 6 + 7 + 8 +<br />

what are the mean, median, and mode? 9 + 10 + 11 + 12 + 12)/13] = 90/13 = 6.923<br />

1, 2, 3, 4, 5, 6, 7, 8, 9, 10, 11, 12, 12 Median = 7<br />

Mode = 12<br />

What is the range of a data set?<br />

Define normal distribution.<br />

What type of curve demonstrates<br />

a normal distribution?<br />

The difference between the highest and<br />

lowest values in a data set<br />

Note: The range in the data set above is<br />

12 − 1 = 11.<br />

A set of numbers in which the mean,<br />

median, and mode are equal.<br />

Mean = median = mode<br />

Gaussian or bell-shaped curve<br />

209


210 Deja Review: <strong>Behavioral</strong> <strong>Science</strong><br />

When a data set shows a large number<br />

of high values and a small number<br />

of low values, what is the distribution<br />

of the data set?<br />

When a data set shows a large number<br />

of low values and a small number of<br />

high values, what is the distribution<br />

of the data set?<br />

When a data set shows two humps,<br />

what is the distribution of the data set?<br />

Negatively skewed<br />

Mean < median < mode<br />

Note: The tail of the curve is on the left<br />

or on the negative end of the number line.<br />

Positively skewed<br />

Mean > median > mode<br />

Note: The tail of the curve is on the<br />

right or on the positive end of the<br />

number line.<br />

Bimodal<br />

Normal distribution<br />

Bimodal distribution<br />

Positively skewed<br />

Negatively skewed<br />

Figure 27.1<br />

Statistical frequency distributions.<br />

What is a variable?<br />

What is an independent variable?<br />

It is a quantity that changes throughout<br />

time.<br />

It is an attribute that the research<br />

scientist may adjust in an experiment.


Biostatistics 211<br />

What is a dependent variable?<br />

How do I remember this?<br />

What is the correlation coefficient (r)?<br />

Why is it important to take the absolute<br />

value of the r?<br />

What is the definition of standard<br />

deviation (σ)?<br />

How is standard deviation used?<br />

It is the outcome that is measured and<br />

affected by the experiment.<br />

The dependent variable is dependent on<br />

the independent variable.<br />

It expresses the strength of a relationship<br />

between two variables. Its value must<br />

be between −1.0 and +1.0.<br />

Note: The minus sign (−) implies a<br />

negative correlation and the plus sign<br />

(+) implies a positive correlation.<br />

The absolute value of r will determine<br />

the strength of the correlation.<br />

It is the root mean square deviation from<br />

the average. The standard deviation is<br />

defined as the square root of the variance.<br />

It is a measure of how dispersed a data<br />

set is around the mean. With a large σ,<br />

data will be more spread out than with a<br />

small one—even if the mean is the same.<br />

0.15% 2.35% 13.5% 34.0% 34.0% 13.5% 2.35% 0.15%<br />

−3 −2 −1 Mean +1 +2 +3<br />

68%<br />

95%<br />

99.7%<br />

Figure 27.2 Normal (Gaussian) distribution represented by a Bell-Shaped Curve. The (+) and (−)<br />

numbers under the curve correspond to the standard deviations from the mean.


212 Deja Review: <strong>Behavioral</strong> <strong>Science</strong><br />

What percentage of data falls in 68%<br />

1 standard deviation from the mean?<br />

What percentage of data falls in 95%<br />

2 standard deviations from the mean?<br />

What percentage of data falls in 99.7%<br />

3 standard deviations from the mean?<br />

STATISTICAL HYPOTHESIS AND ERROR TYPES<br />

What is meant by “null hypothesis<br />

testing”?<br />

What is the null hypothesis (H 0 )?<br />

What is the alternative hypothesis (H 1 )?<br />

Which would be true if there was no<br />

effect in your research study?<br />

Which would be true if there were an<br />

effect in your research study?<br />

What is a type I (α) error?<br />

What is a type II (β) error?<br />

What is power?<br />

The idea that, by default, there are no<br />

relationships between variables. Research<br />

must then be done to disprove this—<br />

called “rejecting the null.”<br />

A hypothesis stating that there is no<br />

difference between an experimental<br />

and control group. (ie, variables are<br />

unrelated)<br />

A hypothesis stating a relationship<br />

between variables, or an effect on an<br />

experimental group different than the<br />

control group. If this was true, you<br />

would “reject the null.”<br />

You would accept the null hypothesis.<br />

You would reject the null hypothesis.<br />

A type I error rejects the null when the<br />

null is actually correct—this type of<br />

error says there is an effect when there<br />

actually is not. A “False-Positive.”<br />

Related to p-value.<br />

A type II error does not reject the null<br />

when the alternative hypothesis is<br />

actually correct—this type of error does<br />

not detect an effect when there actually<br />

is one. A “False-Negative.”<br />

It is the probability of rejecting the null<br />

hypothesis when it is indeed incorrect.<br />

This is the study’s ability to detect an<br />

effect, if present.


Biostatistics 213<br />

If the sample size increases, what<br />

happens to power?<br />

What is statistical significance?<br />

What is the probability (p) value?<br />

It increases.<br />

That the probability of an observed<br />

effect being due solely to chance is low<br />

enough that the effect is likely true.<br />

It is the percent chance that an effect<br />

seen between variables or in an<br />

experimental group was due to chance<br />

alone. For example, if p = 0.25, there is a<br />

25% chance that the effect seen was just<br />

by chance (a type I error).<br />

What p value is most commonly p


214 Deja Review: <strong>Behavioral</strong> <strong>Science</strong><br />

STATISTICAL TESTS<br />

What types of data are evaluated<br />

in statistical tests?<br />

What is nominal data?<br />

What is ordinal data?<br />

What is interval data?<br />

What is ratio data?<br />

What type of common statistical<br />

tests is used to analyze nominal or<br />

ordinal data?<br />

What types of statistical tests are used<br />

to analyze interval or ratio data?<br />

What is the difference between<br />

parametric and nonparametric<br />

statistical tests?<br />

What type of parametric statistical test<br />

determines the difference between<br />

the means of two groups?<br />

Which type of t-test evaluates the means<br />

of two groups at one period of time?<br />

Nominal<br />

Ordinal<br />

Interval<br />

Ratio<br />

It is categorical data where the order of<br />

the categories is arbitrary (eg, gender,<br />

religious beliefs).<br />

It is categorical data where there is a<br />

logical ordering to the categories<br />

(eg, first place winner, second place<br />

winner, third place winner, etc).<br />

It is continuous data where there is a<br />

set interval between values, but where<br />

there is no “natural” zero (eg, Celsius,<br />

Fahrenheit).<br />

It is continuous data where there is both<br />

a set interval between values and a<br />

natural zero (eg, weight, height).<br />

Chi-square<br />

t-test<br />

Analysis of variance (ANOVA)<br />

Linear correlation<br />

Parametric tests analyze data with a<br />

normal distribution. Nonparametric<br />

analyze data sets without a normal<br />

distribution.<br />

t-test<br />

Independent (nonpaired) t-test<br />

(eg, comparing the mean income of one<br />

set of orthopaedic surgeons, Group<br />

Bone, to the mean income of another set<br />

of orthopaedic surgeons, Group Tendon,<br />

at the beginning of the calendar year)


Biostatistics 215<br />

Which type of t-test evaluates<br />

the means of two groups at two<br />

different time periods?<br />

Which type of parametric<br />

statistical test determines the<br />

difference of means of more<br />

than two groups?<br />

Which type of ANOVA test<br />

determines the difference of<br />

means of more than two groups<br />

using only one variable?<br />

Which type of ANOVA test<br />

determines the difference of<br />

means of more than two groups<br />

using two variables?<br />

Which type of parametric<br />

statistical test determines the<br />

relation between two continuous<br />

variables?<br />

What are some examples of<br />

nonparametric tests?<br />

Dependent (paired) t-test (eg, comparing<br />

the mean income of one set of<br />

orthopaedic surgeons, Group Bone,<br />

to the mean income of another set of<br />

orthopaedic surgeons, Group Tendon, at<br />

the beginning and in the middle of the<br />

calendar year)<br />

ANOVA<br />

One-way ANOVA (eg, comparing the<br />

mean income of orthopaedic surgeons<br />

in Group Bone, Group Tendon, and<br />

Group Ligament at the beginning of<br />

the calendar year)<br />

Two-way ANOVA (eg, comparing the<br />

mean income and malpractice insurance<br />

rate of orthopaedic surgeons in Group<br />

Bone, Group Tendon, and Group<br />

Ligament at the beginning of the<br />

calendar year)<br />

Linear correlation<br />

Mann-Whitney U<br />

Wilcoxon’s


216 Deja Review: <strong>Behavioral</strong> <strong>Science</strong><br />

CLINICAL VIGNETTES<br />

A cohort study is done to relate amount of vegetables eaten per day with the<br />

development of diabetes. It is found that the group who is fed five servings per<br />

day of vegetables has a Relative Risk of 0.65 (95% CI = 0.40 − 0.95) for developing<br />

diabetes (control group having RR = 1.00).<br />

What is the independent variable?<br />

The independent variable is amount of vegetables eaten (the manipulated variable).<br />

What is the dependent variable?<br />

The dependent variable is development of diabetes (dependent on vegetables eaten).<br />

Are the results statistically significant?<br />

The results are statistically significant as the CI does not cross 1.00 (the control value).<br />

Is the p-value greater than or less than 0.05?<br />

The p-value would be


Index<br />

A<br />

Abnormal thought formation, 88<br />

Abuse. See also Substance abuse<br />

child, 57–60<br />

domestic partner, 60<br />

elder, 57–60<br />

emotional, 59–60<br />

sexual, 58–59<br />

Acceptance, as stage of dying, 21<br />

Accuracy, 204<br />

Accutane, 94<br />

Acetylcholine, 80–81<br />

Acting out, as defense mechanism, 30<br />

Active suicidal ideation, 67<br />

Acute stress disorder (ASD), 103, 108<br />

ADHD (attention-deficit hyperactivity<br />

disorder), 162<br />

Adjustment disorder, 108–110<br />

Adolescence, 12<br />

Adoption, children’s knowledge of, 12<br />

Adoption study, 69<br />

Adulthood, 15–17<br />

Advance directive, 184<br />

Aggression, 62–63<br />

Aging, 19–20. See also Elderly<br />

memory and, dementia v., 113<br />

physiologic changes, in males, 17<br />

Agnosia, 112<br />

Agoraphobia, 103, 105<br />

Agranulocytosis, 171–172<br />

Alcohol abuse, 148–150. See also Substance<br />

abuse<br />

chronic effects, 149<br />

ethanol<br />

liver metabolism limits, 149<br />

toxicity, 143<br />

genetic factors for, 72<br />

HVA and, 79<br />

relapse rate, 150<br />

screening for, 148<br />

sexual response and, 55–56<br />

thiamine deficiency, 115<br />

tolerance types, 149<br />

toxic effects, 149<br />

in twins, 72<br />

withdrawal, 141, 143<br />

with benzodiazepines, 150<br />

benzodiazepines for, 166<br />

with disulfiram, 141<br />

DTs, 150<br />

Alcohol abuse, withdrawal (Cont.):<br />

hallucinations, 150<br />

seizures, 150<br />

symptoms, 150<br />

α (type I) error, 212<br />

Alternative hypothesis (H 1 ), 212<br />

Altruism, as defense mechanism, 27, 30<br />

Alzheimer disease<br />

brain changes, postmortem, 114<br />

cholinergic neuron decrease, 81<br />

clinical course, 113<br />

dementia and, 113–114<br />

genetic factors, 71<br />

medications, 114<br />

neuropeptides, 82<br />

risk factors, 114<br />

sleep patterns, 48<br />

vascular dementia v., 113<br />

Amenorrhea, 153<br />

from antipsychotics, 172<br />

Amines, 78–81. See also Acetylcholine;<br />

Dopamines; Serotonin<br />

γ -Aminobutyric acid (GABA), 63, 81<br />

Amnestic syndromes, 115<br />

brain structures affected, 115<br />

dementia v., 115<br />

Amphetamines, 55, 144. See also Substance<br />

abuse<br />

Amygdala lesion, 76<br />

Anaclitic depression, 11<br />

Anal phase, 4<br />

Analysis of variance (ANOVA), 214–215<br />

Androgen insensitivity, 50<br />

Anemia, 153–154<br />

Anger, as stage of dying, 21<br />

Anhedonia, 93, 141<br />

Anorexia nervosa, 121, 153–154<br />

bulimia nervosa v., 153<br />

medical effects of, 153–154<br />

treatment, 154<br />

ANOVA (analysis of variance), 214–215<br />

Antipsychotics, 171–172<br />

amenorrhea from, 172<br />

antitypical<br />

mechanism of action, 171<br />

side effects, 171<br />

for delirium, 112<br />

histamine blocking, 80<br />

for NMS, 91<br />

for schizophrenia, 90<br />

217


218 Index<br />

Antipsychotics (Cont.):<br />

therapy, side effects of, 172<br />

typical<br />

extrapyramidal effects, 171<br />

mechanism of action, 171<br />

side effects, 171<br />

Antisocial personality disorder, 29,<br />

127–128, 161<br />

genetic factors, 71<br />

treatment, 128<br />

Anxiety disorders, 103–110<br />

adjustment disorder, 108–110<br />

agoraphobia, 103<br />

ASD, 103<br />

GAD, 103–104<br />

genetic factors, 71<br />

OCD, 103<br />

panic disorder, 103–105<br />

PTSD, 103<br />

social phobia, 103<br />

specific phobia, 103<br />

suicide, 66<br />

Anxiolytics, 165<br />

Aphasia, 112<br />

Apolipoprotein E4, 71<br />

Apomorphine, 53<br />

Apraxia, 112<br />

AR (attributable risk), 206<br />

Artificial life support, 186<br />

ASD (acute stress disorder), 103, 108<br />

Asperger disorder, 159–160<br />

Association, 205–206<br />

Associative learning, 35–39<br />

classical conditioning, 35–37, 36f<br />

imprinting, 35–36<br />

nonassociative v., 35<br />

operant conditioning, 35, 37<br />

Asymmetric tonic neck, 6<br />

Atomoxetine, 162<br />

Attachment, 11–12<br />

Attention-deficit hyperactivity disorder<br />

(ADHD), 162<br />

Attributable risk (AR), 206<br />

Autistic disorder, 159–160<br />

Avoidance, as defense mechanism, 30<br />

Avoidant personality disorder, 71, 130–131<br />

B<br />

Barbiturates, 81, 167<br />

Bargaining, as stage of dying, 21<br />

Basal ganglia lesion, 76<br />

Beneficence, 181<br />

Benzodiazepines<br />

for alcohol withdrawal, 150, 166<br />

for anxiety, barbiturates v., 167<br />

anxiolytics, 165<br />

delirium from, 111<br />

Benzodiazepines (Cont.):<br />

GABA channel opening, 81<br />

for GAD, 104<br />

high potency, 166<br />

hypnotics, 165<br />

lack of tolerance development, 165<br />

low potency, 166<br />

mechanism of action, 165<br />

metabolism phases, 166<br />

during pregnancy, 165<br />

reversal of effects, 165<br />

side effects, 166<br />

for sleep disorders, 47<br />

for social phobia, 107<br />

withdrawal symptoms, 166<br />

Bereavement, 21–22, 22t<br />

β (type II) error, 212<br />

Beta-blockers, 94<br />

Bias, 203–204<br />

Biostatistics, 209–216<br />

Bipolar disorder, 66, 91, 99, 101<br />

mood stabilizers for, 100t<br />

pharmacologic agents, 172–173<br />

during pregnancy, 173<br />

prevalence, 101<br />

types, 99, 101<br />

Body dysmorphic disorder, 106, 121<br />

Borderline personality disorder, 29, 128–129<br />

diagnostic criteria, 128<br />

genetic factors, 71<br />

splitting, 128<br />

treatment, 129<br />

Brain, 75–77<br />

amnestic syndromes’ effects on, 115<br />

lesions, 76–77<br />

consequences, by location, 76–77<br />

neuroanatomy, 75–76<br />

schizophrenia and, physical changes<br />

from, 89<br />

Brief psychotic disorder, 86, 88<br />

Bulimia nervosa, 153–155<br />

anorexia nervosa v., 153<br />

medical effects, 155<br />

treatment, 155<br />

Buprenorphine, 146<br />

Bupropion, 96t, 168<br />

Buspirone, 104, 167<br />

C<br />

Caffeine, 143<br />

Carbamazepine, 100t, 173<br />

Carbatrol. See Carbamazepine<br />

Catalepsy, 90<br />

Catatonic schizophrenia, 89<br />

Catecholamine, 78–79<br />

CBT (cognitive behavioral therapy), 106<br />

CCK (cholecystokinin), 82


Index 219<br />

Celexa. See Citalopram<br />

Central nervous system (CNS), 75–76<br />

Child abuse, 57–60<br />

burns from, typical, 58<br />

fractures from, typical, 57<br />

leading causes of death, 196<br />

sexual, 58–59<br />

age range, 59<br />

by gender, 59<br />

incidence rates, 58<br />

physical signs, 59<br />

psychological signs, 59<br />

shaken baby syndrome, 58<br />

traits of abusers, 57<br />

Child neglect, 57<br />

Childbirth. See Pregnancy and childbirth<br />

Childhood disintegrative disorder, 159, 161<br />

Children<br />

adoption, knowledge of, 12<br />

developmental milestones, 6–8, 9–11t<br />

legal consent for, 185<br />

neuropsychiatric disorders, 159–164<br />

ADHD, 162<br />

disruptive behavior disorders, 161<br />

pervasive development disorders,<br />

159–161<br />

selective mutism, 163<br />

separation anxiety disorder, 163<br />

Tourette disorder, 72, 162–163<br />

understanding of death, 12<br />

Cholecystokinin (CCK), 82<br />

CI (confidence interval), 213<br />

Circadian rhythm sleep disorder, 45, 47<br />

Citalopram, 95t, 167<br />

Classical conditioning, 35–37, 36f, 39<br />

Climacterium, 17<br />

Clinical trial, 203<br />

Clomipramine, 106<br />

Close-ended questions, 179<br />

Clozapine, 171–172<br />

CNS (central nervous system), 75–76<br />

Cocaine, 55, 144. See also Substance abuse<br />

Codeine, 146<br />

Cognitive behavioral therapy (CBT), 106<br />

Cognitive disorders, 86, 111–116. See also<br />

Delirium; Dementia<br />

amnestic syndromes, 115<br />

delirium, 111–112<br />

dementia, 20, 112–114<br />

Cohort study, 202<br />

Coining, 58<br />

Commitment, of patients, 188<br />

Competence, 183<br />

Compliance. See Patient adherence<br />

Compulsions, 106<br />

Concordance, 69<br />

Conduct disorder, 127, 161<br />

Confidence interval (CI), 213<br />

Confidentiality, patient, 178, 187–188<br />

Congenital adrenal hyperplasia, 50<br />

Coning, 180<br />

Conscious, as part of mind, 25<br />

Consent, 184, 186<br />

Continuous reinforcement, 39<br />

Conversion disorder, 117t, 119–120<br />

Correlation coefficient (r), 211<br />

Countertransference, 30<br />

Cravings, 142<br />

Cross-sectional study, 203<br />

Crystallized intelligence, 20<br />

Cupping, 58<br />

Cyclothymic disorder, 101<br />

Cymbalta. See Duloxetine<br />

CYP2D6, 146–147<br />

D<br />

Data sets, 210<br />

Death and dying<br />

bereavement, 21–22, 22t<br />

children’s understanding of, 12<br />

five stages, 21–22<br />

Defense mechanisms, 26–30, 31–32t<br />

Delirium, 111–112<br />

causes, 111<br />

definition, 111<br />

dementia v., 114t<br />

treatment, 112<br />

Delirium tremens (DTs), 150<br />

Delusional disorder, 86, 91<br />

Dementia, 20, 112–114. See also Alzheimer<br />

disease<br />

amnestic syndromes v., 115<br />

core symptoms, 112<br />

delirium v., 114t<br />

diagnostic criteria, 112<br />

memory and aging v., 113<br />

prevalence, 113<br />

types, 113–114<br />

vascular, 113<br />

Denial<br />

as defense mechanism, 29–30<br />

as stage of dying, 21<br />

Depakote. See Valproic acid<br />

Dependent personality disorder, 131–132<br />

Dependent variables, 211<br />

Depersonalization disorder, 136<br />

Depression. See also Major depressive<br />

disorder<br />

anaclitic, 11<br />

bereavement, 22, 22t<br />

body dysmorphic disorder and, 121<br />

double, 94<br />

ECT for, 173<br />

in elderly, 20


220 Index<br />

Depression (Cont.):<br />

HVA and, 79<br />

learned helplessness and, 37<br />

mnemonic for, 93<br />

monoamine theory, 78<br />

postpartum, 16, 94<br />

serotonin levels, 47<br />

sleep disorders, 47–48<br />

as stage of dying, 21<br />

suicide and, 67<br />

treatment, 94–95, 95t, 96t<br />

for postpartum, 16<br />

with SSRIs, 94, 95t, 167<br />

Desyrel. See Trazodone<br />

Detoxification, 142<br />

Development<br />

through adolescence, 12<br />

milestones, 6–8, 9–11t<br />

Tanner stages, 12, 13t<br />

theories, 3–5<br />

Developmental retardation, 11<br />

Dextromethorphan, 146<br />

Diabetes, sexual dysfunction and, in males,<br />

55<br />

Diagnosis-related group (DRG), 192<br />

Diazepam, 167<br />

Disorganized speech, 87<br />

Displacement, as defense mechanism, 27–29<br />

Disruptive behavior disorders, 161<br />

Dissociation, as defense mechanism, 27<br />

Dissociative amnesia, 135<br />

Dissociative anesthetics, 144. See also<br />

Substance abuse<br />

Dissociative fugue, 135–136<br />

Dissociative identity disorder, 136<br />

Disulfiram, 141<br />

Dizygotic twins, 69, 72<br />

Do not resuscitate (DNR) order, 185<br />

Domestic partner abuse, 60<br />

Donepezil, 81, 114<br />

Dopamine hypothesis, 89<br />

“dopamine reward pathway,” 140<br />

Dopamines, 78–79<br />

aggression and, 63<br />

HVA and, 79<br />

for psychiatric conditions, 78<br />

sexual response and, 55<br />

Double-blind study, 204<br />

Down syndrome, 71, 81<br />

Downward drift, 89<br />

DRG (diagnosis-related group), 192<br />

Drug(s). See also specific drugs<br />

first-order elimination, 149f<br />

manic episodes from use of, 101<br />

sexual function and, 55–56<br />

withdrawal from, 141–142<br />

zero-order elimination, 148, 148f<br />

DTs (delirium tremens), 150<br />

Duloxetine, 97t<br />

Dyspareunia, 52<br />

Dyssomnias, 45–46<br />

Dysthymic disorder, 94<br />

E<br />

Early adulthood, 15–16<br />

Eating disorder(s), 153–157<br />

Eating Disorder Not Otherwise Specified<br />

(NOS), 156<br />

Echolalia, 88<br />

ECT (electroconvulsive therapy),<br />

98–99, 173<br />

Effexor. See Venlafaxine<br />

Ego, 25<br />

Elder abuse, 57–60<br />

Elder neglect, 58<br />

Elderly, 19–23<br />

demographics, 19<br />

life expectancy, 19<br />

longevity factors, 21<br />

physiological changes, 19–20<br />

psychological characteristics, 20<br />

psychopathology, 20–21<br />

Electroconvulsive therapy (ECT),<br />

98–99, 173<br />

Emancipated minors, 183<br />

Emotional abuse, 59–60<br />

Endogenous opioids, 82<br />

Endorphins, 81<br />

Enkephalins, 81<br />

Epidemiology, 199–201<br />

Epilepsy, 81<br />

EPS (Extrapyramidal symptoms), of<br />

schizophrenia, 90–91<br />

Equetro. See Carbamazepine<br />

Erikson, Erik, 3–4<br />

Error types, 212–214<br />

Errors. See Medical errors<br />

Escitalopram, 95t<br />

Eskalith. See Lithium<br />

Estrogens, 51<br />

Ethanol<br />

glutamate receptors, 147<br />

mechanisms of action, 147<br />

toxicity, 143<br />

Ethics, medical, 181–189<br />

Ethylamine, 78, 80<br />

Etorphine, 146<br />

Euthanasia, 186<br />

Excitatory neurotransmitters, 77<br />

Exhibitionism, 54<br />

Experimental study, 202<br />

Extinction, 37–38<br />

Extrapyramidal symptoms (EPS),<br />

of schizophrenia, 90–91


Index 221<br />

F<br />

Factitious disorder, 117, 117t, 122–123<br />

Factitious disorder by proxy, 122–123<br />

Failure to thrive, 11<br />

Family risk study, 69<br />

Fatigue, 93<br />

Fentanyl, 146<br />

Fetal alcohol syndrome, 143<br />

Fetishism, 54<br />

First-order elimination, of drugs, 149f<br />

Five stages of death and dying, 21–22<br />

5-HIAA (5-Hydroxyindoleacetic acid), 80<br />

5-HT (5-hydroxytryptamine), 79<br />

Fixation, as defense mechanism, 27<br />

Flooding, 40, 109<br />

Flumazenil, 165<br />

Fluoxetine, 95t, 167–168<br />

Fluvoxamine, 167<br />

Fragile X syndrome, 72<br />

Free association, 27<br />

Freud, Sigmund. See also Psychoanalytic<br />

theory<br />

developmental theory, 3–4<br />

structural mind theory, 25<br />

topographic mind theory, 25<br />

Frontal lobe lesion, 76<br />

Frotteurism, 54<br />

G<br />

GABA (γ -Aminobutyric acid), 63, 81<br />

GAD (generalized anxiety disorder),<br />

103–104<br />

Ganser syndrome, 123<br />

Gender identity, 49<br />

Gender identity disorder, 49, 121<br />

Gender roles, 49<br />

Generalized anxiety disorder (GAD),<br />

103–104<br />

Genetic studies, 69<br />

Genetics, 69–73<br />

alcoholism and, 72<br />

neuropsychiatric disorders and, 71–72<br />

psychiatric disorders and, 69–71<br />

Genital phase, 4<br />

Glutamate, 81, 147<br />

Glycine, 81<br />

H<br />

H 0 (null hypothesis), 212<br />

H 1 (alternative hypothesis), 212<br />

Habituation, 40<br />

Hallucinations, 85–86<br />

alcohol withdrawal, 150<br />

illusions v., 86<br />

Haloperidol, 112<br />

Health maintenance organization (HMO),<br />

193<br />

Health status, determinants for, 195–196<br />

Health-care costs, 193–194<br />

Health-care delivery systems, 194–195<br />

Health-care insurance, 191–193<br />

coverage, 192<br />

government programs, 191–192<br />

private, 193<br />

types, 191–192<br />

Health-care managed-care plans, 193<br />

Heroin, 56, 146. See also Substance abuse<br />

Himmelsbach hypothesis, for substance<br />

abuse, 140<br />

Hippocampus lesion, 76<br />

Histamines, 80<br />

Histrionic personality disorder, 29, 71, 130<br />

HIV (human immunodeficiency virus),<br />

113, 182<br />

HMO (health maintenance<br />

organization), 193<br />

Homicide, 62<br />

Homosexuality, chromosomal factors, 50<br />

Homovanillic acid (HVA), 79<br />

Hormones, behavior influenced by, 51<br />

Hospice care, 194–195<br />

Human immunodeficiency virus (HIV),<br />

113, 182<br />

Humor, as defense mechanism, 27–28<br />

Huntington’s Disease, 71, 113<br />

HVA (Homovanillic acid), 79<br />

Hydromorphone, 146<br />

Hypertensive crisis, 98<br />

Hypnotics, 165<br />

Hypoactive sexual desire, 52<br />

Hypochondriasis, 120<br />

Hypomania, 99<br />

Hypothalamus lesion, 76–77<br />

Hypotheses, 212–214<br />

I<br />

Id, 25<br />

Identification, as defense mechanism, 27, 29<br />

Illusions, 86<br />

Immature defense mechanisms, 27<br />

Impotence, 52<br />

Imprinting, 35–36<br />

Incidence, 199<br />

Incompetence, in patients, 183<br />

Independent variables, 210<br />

Indolamines, 78<br />

Infants. See also Neonates<br />

developmental milestones, 6–8, 9–11t<br />

morbidity and mortality, 5<br />

leading causes, 196<br />

Informed consent, 186<br />

Inhibitory neurotransmitters, 77<br />

Insomnia, 45–46, 167–168<br />

Insurance. See Health-care insurance


222 Index<br />

Intellectualization, as defense mechanism,<br />

27–28<br />

Intelligence quotient (IQ), 41<br />

Intelligence tests, 41<br />

Interferon, 94<br />

Interpreters, 179<br />

Interrater reliability, 204<br />

IQ (intelligence quotient), 41<br />

Isolation, as defense mechanism,<br />

27–28<br />

J<br />

Justice, 181<br />

K<br />

Kappa receptor agonists, 146<br />

Kappa receptor antagonists, 146<br />

Ketamine, 144. See also Substance abuse<br />

Kleine-Levin syndrome, 46<br />

Kubler Ross, Elizabeth, 21–22<br />

L<br />

Lamictal. See Lamotrigine<br />

Lamotrigine, 100t, 173<br />

Lanugo, 153–154<br />

Latency phase, 4<br />

Learned helplessness, 37<br />

Learning theory, 35–42<br />

associative, 35–39<br />

classical conditioning, 35–37, 36f<br />

operant conditioning, 35, 37<br />

nonassociative, 35, 39–41<br />

habituation, 40<br />

observational learning, 39<br />

Leukopenia, 153–154<br />

Levodopa, 111<br />

Lexapro. See Escitalopram<br />

Life expectancy, 19<br />

Linear correlation, 214–215<br />

Lithium, 100t, 172<br />

Living wills, 185<br />

Longevity, 21<br />

Loperamide, 147<br />

Lorazepam, 167<br />

M<br />

Major depressive disorder, 66, 91, 93–98<br />

dysthymic disorder v., 94<br />

genetic factors, 70<br />

MAOIs, 98<br />

from medications, 94<br />

prevalence, 93<br />

psychotherapy treatments, 98–99<br />

suicidal ideation, 93<br />

symptoms, 93<br />

TCAs, 97–98<br />

treatment for, 94–95, 95t, 96t<br />

Malingering, 117, 117t, 122, 123<br />

Malpractice, 181–182<br />

Mania, 99, 101<br />

Mann-Whitney test, 215<br />

MAOIs. See Monoamine oxidase inhibitors<br />

Marijuana, 55–56, 144–145. See also<br />

Substance abuse<br />

Marriage, 15<br />

Mature defense mechanisms, 27<br />

Mean, 209<br />

Median, 209<br />

Medicaid, 191<br />

Medical errors, 177<br />

Medical ethics, 181–189<br />

Medicare, 191–192<br />

Melanosis coli, 154<br />

Memantine, 114<br />

Menopause, 17<br />

Mental age, 41<br />

Mental retardation, 41<br />

Meperidine, 146<br />

Methadone, 56, 147. See also Substance abuse<br />

3-Methoxy-4-hydroxyphenyglycol (MHPG),<br />

79<br />

Methyldopa, 94<br />

N-methyl-D-aspartate (NMDA), 114<br />

Methylphenidate, 162<br />

MHPG (3-Methoxy-4-hydroxyphenyglycol),<br />

79<br />

Middle adulthood, 17<br />

Mirtazapine, 96t, 104<br />

Mode, 209<br />

Modeling, in nonassociative learning, 39–40<br />

Monoamine oxidase inhibitors (MAOIs),<br />

67, 170<br />

drug interactions, 170<br />

mechanism of action, 170<br />

side effects, 170<br />

Monoamine theory of depression, 78<br />

Monozygotic twins, 69–72<br />

Mood disorders, 86<br />

Mood stabilizers, 100t<br />

Moro reflex, 5–6<br />

Morphine, 146<br />

Mu receptor agonists, 146<br />

Mu receptor antagonists, 146<br />

N<br />

Nalorphine, 146<br />

Naltrexone, 146<br />

Namenda. See Memantine<br />

Narcissistic personality disorder,<br />

121, 129–130<br />

Narcolepsy, 45–46, 86<br />

Narcotic overdose. See Overdose, narcotic<br />

Necrophilia, 54<br />

Nefazodone, 96t


Index 223<br />

Negative predictive value (NPV), 200, 201f<br />

Negative reinforcement, 37–38<br />

Neologisms, 88<br />

Neonates. See also Infants<br />

reflexes, 5–6<br />

Neuroanatomy, 75–76<br />

Neuroleptic malignant syndrome (NMS), 91<br />

Neuropeptides, 81–82<br />

Neurotensin, 82<br />

Neurotransmitters, 77–78<br />

alterations in psychiatric conditions, 78t<br />

release steps, 77<br />

NMDA (N-methyl-D-aspartate), 114<br />

NMS (neuroleptic malignant syndrome), 91<br />

Nominal data, 214<br />

Nonassociative learning, 35, 39–41<br />

Nonmaleficence, 181<br />

Nonrapid eye movement (NREM), 43–45<br />

Norepinephrine<br />

aggression and, 63<br />

as biogenic amine, 79<br />

MHPG, 79<br />

sexual response and, 55<br />

synthesis, 79<br />

Normal distribution, 209<br />

NOS (Eating Disorder Not Otherwise<br />

Specified), 156<br />

NPV (negative predictive value), 200, 201f<br />

NREM (nonrapid eye movement), 43–45<br />

Null hypothesis (H 0 ), 212<br />

Nursing homes, 194<br />

O<br />

Observational learning, 39<br />

Observational study, 202<br />

Obsessive-compulsive disorder (OCD),<br />

28, 103, 105–106, 132<br />

Obsessive-compulsive personality disorder<br />

(OCPD), 28, 103, 106, 132<br />

Obstructive sleep apnea (OSA), 46–47<br />

OCD. See Obsessive-compulsive disorder<br />

OCPD. See Obsessive-compulsive<br />

personality disorder<br />

Odds ratio (OR), 206<br />

Open-ended questions, 179<br />

Operant conditioning, 35, 37–39<br />

Opiates, 146<br />

Opioid receptors<br />

location, 145<br />

mechanisms of action, 145<br />

types, 145<br />

Opioids<br />

agonists, 145–146<br />

antagonists, 145<br />

endogenous, 82<br />

mechanisms of action, 145<br />

opiates v., 146<br />

Oppositional defiant disorder, 161<br />

OR (odds ratio), 206<br />

Oral phase, 4<br />

Ordinal data, 214<br />

Orgasmic disorder, 52<br />

OSA (obstructive sleep apnea), 46–47<br />

Osteoporosis, 17<br />

Overdose, narcotic, 147<br />

Oxycodone, 146<br />

Oxytocin, 82<br />

P<br />

p (probability) value, 213<br />

Palliative care, 186, 194<br />

Palmar grasp, 6<br />

Panic disorder, 103–105<br />

Papaverine, 53<br />

Parachute reflex, 6<br />

Paranoid personality disorder, 29, 126–127<br />

Paranoid schizophrenia, 89<br />

Paraphilias, 54<br />

Parasomnias, 45<br />

Parietal lobe lesion, 76<br />

Parkinson’s disease, 78–79<br />

Parotiditis, 155<br />

Paroxetine, 95t, 167–168<br />

Passive suicidal ideation, 67<br />

Patient adherence, 179<br />

Patient autonomy, 181<br />

Patient confidentiality, 178, 187–188<br />

Patient referrals. See Referrals, patient<br />

Pavlov’s classical conditioning, 36, 36f<br />

Paxil. See Paroxetine<br />

Pedigree study, 69<br />

Pedophilia, 54<br />

Pentazocine, 146<br />

Peripheral nervous system (PNS), 75–76<br />

Personality disorders, 66, 125–133<br />

classification, 125–126<br />

cluster A, 126–127<br />

cluster B, 127–128<br />

cluster C, 130–131<br />

definition, 125<br />

diagnostic criteria, 125<br />

psychosis and, 86<br />

Pervasive development disorders, 159–161<br />

Asperger disorder, 159–160<br />

autistic disorder, 159–160<br />

childhood disintegrative disorder, 159, 161<br />

Rett disorder, 159–160<br />

Phallic phase, 4<br />

Phencyclidine, 144. See also Substance abuse<br />

Phentolamine, 53<br />

Phobias, 37<br />

Physician impairment, 182<br />

Piaget, Jean, 3–5<br />

PNS (peripheral nervous system), 75–76


224 Index<br />

“Pooled-risk,” 192<br />

Positive predictive value (PPV),<br />

200, 201f<br />

Positive reinforcement, 37–38<br />

Postpartum “blues,” 15–16<br />

Postpartum major depression, 16, 94<br />

Postpartum psychosis, 16<br />

Posttraumatic stress disorder (PTSD),<br />

62, 103, 107–108<br />

adjustment disorder v., 108<br />

ASD v., 108<br />

suicide and, 66<br />

Power, 212–213<br />

Power of attorney, 184<br />

PPOs (preferred provider organizations),<br />

193<br />

PPV (positive predictive value), 200, 201f<br />

Prazosin, 108<br />

Precision, 204<br />

Preferred provider organizations (PPOs),<br />

193<br />

Pregnancy and childbirth<br />

benzodiazepines during, 165<br />

bipolar disorder and, pharmacologic<br />

treatment, 173<br />

postpartum period, 15–16<br />

“blues,” 15–16<br />

major depression, 16<br />

psychosis, 16<br />

Premature birth, 5<br />

Premature ejaculation, 52<br />

Prevalence, 199<br />

Priapism, 168<br />

Primary process thinking, 26<br />

Principle of double-effect, 181<br />

Probability (p) value, 213<br />

Progesterone, 51<br />

Projection, as defense mechanism,<br />

27, 29<br />

Prospective study, 201<br />

Prozac. See Fluoxetine<br />

Psychiatric disorders, 69–71. See also Major<br />

depressive disorder; Psychosis;<br />

Schizophrenia<br />

Psychoanalytic theory, 25–33<br />

defense mechanisms, 26–30, 31–32t<br />

psychotherapy, 26–27<br />

structural theory of the mind, 25<br />

topographic theory of the mind, 25<br />

Psychosis, 85–86<br />

clinical hallmarks, 85<br />

hallucinations, 85–86<br />

from medical conditions, 86<br />

narcolepsy and, 86<br />

personality disorders and, 86<br />

from pharmacological agents, 171<br />

postpartum, 16<br />

Psychotherapy, 26–27<br />

for major depressive disorder, 98–99<br />

psychodynamic, 26–27<br />

for specific phobia, 109<br />

Psychotic disorders, 85–92<br />

PTSD. See Posttraumatic stress disorder<br />

Punishment, 37–38<br />

Purging, 155<br />

Q<br />

Quaternary amines, 78<br />

R<br />

r (correlation coefficient), 211<br />

Rape, 61–62<br />

Rapid eye movement (REM), 20, 43–45<br />

Rapport, 177<br />

Ratio data, 214<br />

Rationalization, as defense mechanism,<br />

27–28<br />

Reaction formation, as defense mechanism,<br />

27–28<br />

Referrals, patient, 177<br />

Refusal of treatment<br />

by patients, 186<br />

by physicians, 187<br />

Regression, as defense mechanism, 27, 29<br />

Reinforcement, in operant conditioning,<br />

37–39<br />

Relapse, 142, 150<br />

Relative risk (RR), 205–206, 205f<br />

Reliability, 204<br />

REM (rapid eye movement), 20, 43–45<br />

Remeron. See Mirtazapine<br />

Repression, as defense mechanism, 27, 29<br />

Research study designs, 201–203<br />

Reserpine, 94<br />

Reticular system lesion, 76<br />

Retrospective study, 202<br />

Rett disorder, 159–160<br />

Risperidone, 172<br />

Ritalin, 46<br />

Rivastigmine, 114<br />

Romazicon. See Flumazenil<br />

Rooting reflex, 6<br />

RR (relative risk), 205–206, 205f<br />

S<br />

Schizoaffective disorder, 86, 91<br />

Schizoid personality disorder, 126, 130–131<br />

Schizophrenia, 69–70, 87–91<br />

abnormal thought formation, 88<br />

antipsychotics for, 90<br />

catalepsy, 90<br />

catatonic, 89<br />

characteristic symptoms, 87–88<br />

disorganized speech, 87


Index 225<br />

Schizophrenia (Cont.):<br />

dopamine hypothesis, 89<br />

dopamines for, 78<br />

downward drift, 89<br />

EPS, 90–91<br />

gender and, 69, 89<br />

genetic factors, 69–70<br />

among monozygotic twins, 70<br />

paranoid, 89<br />

prevalence, 69<br />

prodromal phase, 88<br />

prognosis, 90<br />

residual phase, 88<br />

schizotypal personality disorder and, 127<br />

subtypes, 89<br />

suicide and, 66, 90<br />

TD, 90–91<br />

waxy flexibility, 90<br />

Schizophreniform disorder, 86<br />

Schizotypal personality disorder, 29, 71, 127<br />

Selective mutism, 163<br />

Selective serotonin reuptake inhibitors (SSRIs)<br />

for children, 167<br />

commonly used, 167<br />

for depression, 94, 95t, 167<br />

for GAD, 104<br />

MAOIs and, 98<br />

for panic disorder, 105<br />

side effects, 167<br />

for suicide, 67<br />

Sensitivity, 200<br />

Sensitization, 41<br />

Separation anxiety disorder, 163<br />

Serotonin, 79–80<br />

altered behavioral factors, 79–80<br />

as biogenic amine, 79–80<br />

decreased aggression and, 63<br />

depression, 47<br />

sexual response and, 80<br />

in sleep, 44<br />

synthesis, 80<br />

Serotonin syndrome, 98<br />

Sertraline, 95t, 167<br />

Serzone. See Nefazodone<br />

Sexual abuse, 58–59<br />

Sexual assault, 61–62<br />

Sexual aversion disorder, 53<br />

Sexual consent, 61<br />

Sexual development, 49–51<br />

Sexual masochism, 54<br />

Sexual response, 51–56<br />

drugs as influence on, 55–56<br />

dysfunction, 52–53<br />

medical conditions and, 54–55<br />

normal cycle, 51–52<br />

paraphilias, 54<br />

serotonin and, 80<br />

Sexual sadism, 54<br />

Sexual violence, 61–62<br />

Sexuality, Tanner stages of development for,<br />

12, 13t<br />

Shaken baby syndrome, 58<br />

Shaping, 38<br />

Shared psychotic disorder, 91<br />

SIG E CAPS, 93<br />

Sildenafil citrate, 53<br />

Sleep, 43–48<br />

abnormal, 45–47<br />

Alzheimer’s disease, 48<br />

depression, 47–48<br />

disorders, 45–47<br />

normal, 43–45<br />

patterns among elderly, 20<br />

Sleep terrors, 47<br />

Sleepwalking, 47<br />

Social phobia, 103, 106–107, 109<br />

Somatization disorder, 71, 118–119<br />

Somatoform disorders, 117–124<br />

body dysmorphic disorder, 106, 121<br />

conversion disorder, 117t, 119–120<br />

factitious disorder, 117, 117t, 122–123<br />

hypochondriasis, 120<br />

malingering, 117, 117t, 123<br />

primary types, 117, 117t<br />

somatization disorder, 71, 118–119<br />

Somatostatin, 82<br />

Specific phobia, 103, 108–109<br />

Specificity, 200<br />

Splitting, as defense mechanism, 27–29, 128<br />

Spontaneous recovery, 37<br />

Spooning, 58<br />

SSRIs. See Selective serotonin reuptake<br />

inhibitors<br />

Standard deviation, 211–212, 211f<br />

Stanford-Binet scale, 41<br />

“Startle reflex.” See Moro reflex<br />

Statistics<br />

distribution, 209–212, 210f<br />

hypothesis and error types, 212–214<br />

tests, 214–215<br />

Statutory rape, 62<br />

Stepping reflex, 6<br />

Steroid use, 63, 94<br />

Stimulus generalization, 37<br />

Structural mind theory, 25<br />

Sublimation, as defense mechanism, 27–28<br />

Substance abuse, 139–151<br />

aggression and, 63<br />

cravings, 142<br />

delirium and, 111<br />

diagnostic criteria, 139<br />

dissociative disorders, 135<br />

among elderly, 21<br />

Himmelsbach hypothesis, 140


226 Index<br />

Substance abuse (Cont.):<br />

medical effects, 143<br />

mood disorders and, 86<br />

nonmedical effects, 144<br />

physiological pathways, 140<br />

relapse, 142<br />

sexual assault and, 61<br />

sexual response and, 55–56<br />

suicide, 66<br />

tolerance and, 139<br />

treatment, 139–142<br />

detoxification, 142<br />

negative reinforcement, 142<br />

positive reinforcement reduction, 141<br />

principles, 140<br />

substitute drugs, 141<br />

symptom prevention, 140<br />

withdrawal, 139–142<br />

Substance P, 82<br />

Suicide, 65–68<br />

antidepressants, 67<br />

depression and, 67<br />

gender and, 65<br />

ideation, 67<br />

major depressive disorder and, 93<br />

MAOIs in treatment therapy, 67<br />

PTSD, 66<br />

race and, 66<br />

risk factors, 65–66<br />

schizophrenia, 66<br />

substance abuse, 66<br />

Superego, 26<br />

Suppression, as defense mechanism, 29<br />

Systematic desensitization, 40<br />

T<br />

Tacrine, 81<br />

Tanner stages of development, 12, 13t<br />

Tardive dyskinesia (TD), 90–91<br />

TCAs. See Tricyclic antidepressants<br />

TD (tardive dyskinesia), 90–91<br />

Tegretol. See Carbamazepine<br />

Temporal lobe lesion, 76<br />

Testing, 203–204<br />

Testosterone, 51<br />

Test-retest reliability, 204<br />

Thiamine deficiency, 115<br />

Thought blocking, 88<br />

Tolerance, 139, 149. See also Substance abuse<br />

Topographic mind theory, 25<br />

Tourette disorder, 72, 162–163<br />

Transference, 30<br />

Transvestic fetishism, 54<br />

Trazodone, 96t, 168<br />

Trichotillomania, 106<br />

Tricyclic antidepressants (TCAs), 80, 97–98,<br />

169–170<br />

commonly used, 98, 169<br />

mechanism of action, 168–169<br />

overdose, 169<br />

side effects, 97, 169<br />

tertiary, 169<br />

t-test, 214–215<br />

Turner syndrome, 51<br />

Twins. See Dizygotic twins; Monozygotic<br />

twins<br />

Type I (α) error, 212<br />

Type II (β) error, 212<br />

U<br />

Uprima. See Apomorphine<br />

V<br />

Valproic acid, 100t, 173<br />

Vanillylmandelic acid (VMA), 79<br />

Variable ratio reinforcement, 39<br />

Variables, 210<br />

Vascular dementia, 113<br />

Vasoactive intestinal peptide (VIP), 82<br />

Vasopressin, 82<br />

Venlafaxine, 97t, 104<br />

Viagra. See Sildenafil citrate<br />

VIP (vasoactive intestinal peptide), 82<br />

VMA (vanillylmandelic acid), 79<br />

Voyeurism, 54<br />

W<br />

Waxy flexibility, 90<br />

Wellbutrin. See Bupropion<br />

Wilcoxon test, 215<br />

Withdrawal. See also Substance abuse<br />

from alcohol, 141, 143<br />

from benzodiazepines, 166<br />

from drugs, 141–142<br />

Word salad, 88<br />

Z<br />

Zero-order elimination, of drugs,<br />

148, 148f<br />

Zoloft. See Sertraline

Hooray! Your file is uploaded and ready to be published.

Saved successfully!

Ooh no, something went wrong!